You are on page 1of 150

PHARMACOLOGY

A 56-year-old man calls his family physician complaining of severe diarrhea,


nausea, and vomiting, and muscle pain and weakness of approximately 18 hours
duration. The physician recalls that the patient is taking Drug C for the treatment
and prevention of recurrent acute flares of gout and asks the patient if he has
started taking any other drugs, nutritional supplements, or grapefruit juice. The
man explains that he is not taking any other drugs or supplements, but for the
last three days he has consumed “quite a lot of grapefruit juice” since his
brother sent him grapefruits from Florida. The doctor tells the man to stop taking
Drug C and go to the emergency department right away. She includes in her
differential grapefruit juice-induced drug toxicity caused most likely by inhibition
of:
A.CYP2D6
B.Organic anion transport protein
C.P-glycoprotein
D.Steroid hormone binding globulin
E.UDP-glucuronyltransferase
SOURCE: Dr. Goldstein, Handout Toxicities and DDIs p8,9) Cellular/Molecular.

Many drugs used in the treatment of cancer act by cross-linking DNA strands
and overwhelming the cellular repair mechanisms, which instigates a
coordinated cascade of reactions leading to the death of the cell. This
mechanism of cellular toxicity is called:
A. Apoptosis
B. Fibrosis
C. Free radical generation
D. Necrosis
E. Teratogenesis
SOURCE: Dr. Goldstein, Toxicities, and Drug Interactions p5 Cellular/Molecular
Thread.

A 46-year-old woman presents to the dermatologist with complaint of nail


fungus. The woman is upset by the “ugly” nails and chooses not to tell the
doctor that she takes simvastatin for dyslipidemia. The physician finds that the
patient’s liver function studies are within normal limits and prescribes
itraconazole, an oral antifungal agent. Five days later, the woman calls the
doctor complaining of new onset muscle soreness and dark colored urine
(symptom/sign of statin toxicity). She admits that she is taking the statin.
(Statins are oral drugs that are taken up into hepatocytes via carrier protein
transport and act in the hepatocytes to inhibit cholesterol synthesis; simvastatin
is metabolized by oxidation.) The mechanism of the drug interaction is most
likely inhibition of:
A. CYP3A4
B. Enterohepatic circulation
C. Organic anion transport protein OATP1B1
D. UDP-glucuronosyltransferase
E. Uridine diphosphate glucuronic acid
Rationale: As per Dr. Goldstein, “A is the one best answer. The students are expected
to learn the starred (*) drugs. On page 13 of the toxicities and drug interactions
handout, ketoconazole is starred (*) and listed as a moderate-to-strong CYP3A4
inhibitor. “When two co-administered drugs are both metabolized by a single CYP they
compete for binding to the enzyme's active site. This can result in the inhibition of
metabolism of one or both of the drugs, leading to elevated plasma levels.”
SOURCE: Dr. Goldstein, Toxicities and Drug Interactions p13
 Cellular/Molecular
Thread

 A patient is prescribed an oral cephalosporin antibiotic for the treatment of a
skin infection. Shortly after taking the drug, the patient develops a pruritic
(itchy) urticarial (blotchy, red welts) rash and angioedema (swelling around lips
and eyes). This immune-mediated reaction is most likely caused by:

A. Apoptosis
B. Hypersensitivity

C. Necrosis

D. Phototoxicity

E. Potentiation

SOURCE: Dr. Goldstein, Toxicities and Drug Interactions p Cellular/Molecular Thread

A 52-year-old man complains to his doctor that since he started taking


probenecid for gout, he has been feeling unusually drowsy and is having
cognitive difficulties. (Probenecid prevents transport of uric acid from the
proximal renal tubules back into the circulation.) The patient has a history of
anxiety treated chronically with a centrally active antianxiety medication that is
renally excreted unchanged. The doctor suspects a drug interaction as a result
of probenecid-mediated:
A. Alkalinization of urine pH

B. Competition at organic anion transport proteins
C. Inhibition of CYP3A4

D. Interference with absorption

E. Noncompetitive antagonism at receptor site
Rationale:
a.Incorrect. Alkalinization of urine may be used to enhance the excretion of weak acids
b. Correct. Inhibition of renal organic acid transporters (OATP) can increase the plasma
levels of other drugs that are also substrates of OATP, which, in turn, increases the risk
of adverse effects, as described in the stem of this question (dose-concentration-
response relationship, Pharmacodynamics handout p4).
c.Incorrect. The drug in the question is excreted unchanged (it is not metabolized). Thus,
CYP3A4 function is not relevant to this drug‘s elimination.
d.Incorrect. Interference with absorption would decrease plasma drug levels. Decreased
levels may result in a subtherapeutic response rather than drug toxicities.
e.Incorrect. The two drugs have different sites of action. The antianxiety medication acts
in the central nervous system and an antigout medication that acts in the kidney.
Item Description: Goldstein 2013; Pharmacology of Drug Toxicities and Drug
Interactions handout p14; slide 36
A patient with a history of breast cancer 8 years ago is found to have leukemia
secondary to the chemotherapy she received for the breast cancer. Which one of
the following is the most likely mechanism of toxicity that led to this patient‘s
current hematologic neoplasm?
A. Hypersensitivity reaction
B. Idiosyncratic response
C. Mutagenesis
D. Necrosis
E. Teratogenesis
Rationale:
a. Incorrect. A hypersensitivity reaction is an immune-mediated reaction and
appears within minutes to days (handout p3).
b. Incorrect. An idiosyncratic response is a rare, unpredictable response resulting
from abnormal reactivity to a chemical and is peculiar to a given individual. Leukemia is
a predictable, delayed toxicity of many chemotherapeutic agents (handout p2).
c. Correct. Many chemotherapeutic drugs kill cancer cells by inhibiting DNA
synthesis or directly damaging the DNA of the neoplastic cells. These agents also
harm certain host cells. Chemotherapeutic drugs that damage the DNA of bone
marrow cells can lead to hematologic cancers.
d. Incorrect. Necrosis is unregulated digestion of cellular components and disruption
of cellular membranes and inflammation that results from external damage to cells or
tissues (handout p5).
e. Incorrect. Teratogenesis is the drug- or chemical-induced toxicity to the fetus
(handout p6).
Item Description: Goldstein 2013; Pharmacology of Drug Toxicities and Drug
Interactions handout p2,5; slide 5, 7,11
A patient with neuropathic pain is given a prescription for an inhibitor of
norepinephrine and serotonin reuptake transporters. The doctor cautions the
patient that the drug can cause hypotension, which will resolve when the drug is
withdrawn. This effect occurs because of the drug‘s affinity for vascular alpha-
adrenergic receptors. This drug toxicity is an example of a(n):
A. Allergic reaction

B. Cardiac toxicity

C. Delayed response

D. Idiosyncratic response
E. Off-target effect
Rationale:
a. Incorrect. An allergic reaction is an immunogenic response.
b. Incorrect. The drug in the stem acts on the vasculature. Cardiac effects are not
indicated.
c. Incorrect. Hypotension is a can occur at any time based on its affinity for the
vascular receptors and is reversible when the drug is removed.
d. Incorrect. Hypotension is a predictable potential side effect.
e. Correct. The stem describes the different receptor effects of the prescribed drug.
The therapeutic action is related to the drug‘s action on norepinephrine and serotonin
reuptake, whereas the adverse effect of hypotension is due the drug‘s action on off-
target alpha-adrenergic receptors.
Item Description: Goldstein 2013; Pharmacology of Drug Toxicities and Drug
Interactions handout p2-4; slides 5-12
A patient presenting to the emergency department for an overdose of the μ
receptor agonist, heroin, is treated with the antidote, which is a reversible μ
receptor antagonist. This drug interaction is an example of:
A. Chemical antagonism
B. Competitive antagonism
C. Physiological antagonism
D. Sequestration

E. Synergy
Rationale:
a. Incorrect. Chemical antagonism is the reaction between two chemicals to
neutralize their effects (handout p8).

b. Correct. Both the agonist and the antagonist act at the mu receptor, thus, of the
listed options, the antidote is a competitive antagonist.
c. Incorrect. Physiological antagonism is the reaction between two chemicals in
which the antagonist negates the effects of the activator (handout p7).

d. Incorrect. Formation of a complex between to two drugs (sequestration) prevents
drug absorption from the gastrointestinal tract (handout p10).
e. Incorrect. Synergy between drugs is the combined effect exceeds the sum of the
effects of each agent given alone (handout p7).
Item Description: Goldstein 2013; Pharmacology of Drug Toxicities and Drug
Interactions handout p8; slide17
A patient being evaluated for antipsychotic therapy admits to smoking 2 packs
of cigarettes daily. The doctor recognizes that use of tobacco caused induction
of the enzyme that mediates the metabolism of the selected antipsychotic
medication. This enzyme is:
A. CYP1A2

B. CYP2C9

C. CYP2D6

D. Glucuronosyltransferase
E. Methyltransferase
Rationale:
a. Correct. Smoking and other types of tobacco use causes induction of CYP1A2
b. Incorrect. Smoking does not induce CYP2C9

c. Incorrect. Smoking does not induce CYP2D6

d. Incorrect. Smoking does not induce methyltransferase
e. Incorrect. Smoking does not induce glucuronosyltransferase

Item Description: Goldstein; Pharmacology of Drug Toxicities and Drug Interactions
handout p13; slide 27
A particular drug is a known teratogen and is, therefore, contraindicated in:
A. Adolescent patients
B. Cancer patients

C. Elderly patients
D. Pregnant patients
E. Professional athletes
Rationale:
a. Incorrect. Please see the explanation in option d.
b. Incorrect. Please see the explanation in option d.
c. Incorrect. Please see the explanation in option d.
d. Correct. A teratogen is a drug or other agent that causes malformation of the
fetus.
e. Incorrect. Please see the explanation in option d.
Item Description: Goldstein; Pharmacology of Drug Toxicities and Drug Interactions
handout p6, slide 11
A drug for the prevention of nausea and vomiting acts by inhibiting the action of
specific receptors in the chemoreceptor trigger zone in the brainstem. An
anticipated adverse effect of the drug is constipation by inhibition of the same
receptors in the gastrointestinal tract. Constipation in this case is categorized as
a:
A. Mutagenic effect
B. Pathological effect
C. Pharmacological effect
D. Teratogenic effect

E. Therapeutic effect

SOURCE: Dr. Goldstein, Drug Toxicity and Drug Interactions PowerPoint slide #5
Cellular/Molecular Thread.
A 4-year-old child presents to the emergency department with signs/symptoms of respiratory
depression. Physical examination is significant for shallow breathing, hypotension, weak pulse,
pinpoint pupils, and bluish lips and fingernails. Blood levels confirm acute opioid toxicity. The child
had outpatient surgery for a compound fracture of the right arm the previous day and has been
administered a standard initial pediatric dose of codeine every 4 hours as directed for the pain. She is
usually a healthy, active child. The physician suspects the child may have a genetic variant associated
with increased activity of the drug-metabolizing enzyme that converts codeine to the active morphine.
The toxic reaction is most likely the result of a(n):
A. Error in morphogenesis
B. Increased expression of solute carrier transporters
C. Single nucleotide polymorphism
D. Toxicity to the liver

E. Irreparable drug-induced damage to DNA

SOURCE: Dr. Goldstein, Drug toxicities and Drug interactions slide #12
Cellular/Molecular Thread
Based on the method of elimination of the large majority of beta-lactam
antibiotics, the predominant mechanism of drug interactions can be anticipated
to be mediated by inhibition of:
A. Biliary secretion
B. CYP3A4
C. N-acetylcysteine
D. Organic anion transporter
E. UDP-glucuronosyltransferase
Rationale:
a. Incorrect. The vast majority of beta-lactams are renally eliminated as active drug.
b. Incorrect. The vast majority of beta-lactams are not metabolized via the CYPs.
c. Incorrect. N-acetycysteine is the antidote to acetaminophen toxicity.
d. Correct. The vast majority of beta-lactams are renally eliminated via OAT as
active drug.
e. Incorrect. The vast majority of beta-lactams are not metabolized via
glucuronidation.
Item Description: Goldstein, 2013; General Principles of Antimicrobial Therapy;
Pharmacokinetic properties; Slide #14; handout p3,4,11,17 uestion #: 65
Using two antibiotics when only one is required unnecessarily increases the risk
of toxicity and:
A. Acquisition of hospital-acquired infection
B. Anaphylactic reaction
C. Damage to the microbiome
D. Development of resistance
E. Duration of drug therapy
Rationale:
a. Incorrect. Nosocomial infections are acquired from health care institutions by the
spread of pathogens from one patient to another, on inadequately cleaned hands of
hospital personnel, or by contaminated materials, and not as a direct result of antibiotic
therapy.
b. Incorrect. The number of antibiotics given does not increase the risk of allergic
reaction in the non-allergic patient.
c. Correct. The enteric flora protects the host from overgrowth of bacteria that are
innocuous when present in low numbers, but can cause disease if allowed to dominate
the flora, such as C. difficile, which often proliferates markedly when the normal flora
has been disrupted secondary to antibiotic treatment
d. Correct. The use of two antibiotics does not necessarily increase the risk of
resistance. In addition to damaging the microbiome, increasing the spectrum may
expose a greater variety of bacteria to selection pressure and increasing resistance
e. Incorrect. The duration of antimicrobial therapy is determined by the location and
nature of the infection.
Item Description: Goldstein, 2013; General Principles of Antimicrobial Therapy; Slide
#20; handout p7

Question #: 81
A patient taking a particular antiepileptic medication is given an antimicrobial agent that is metabolized by
CYP3A4. Because of drug interaction involving CYP3A4, the dose of the antimicrobial is increased, according
to the manufacturer’s recommendations. The CYP3A4 inducer is most likely:
✓A. Carbamazepine
B. Erythromycin
C. Itraconazole
D. Probenecid
E. Ritonavir
Rationale:
a- Correct. Carbamazepine is an antiepileptic drug and an inducer of CYP3A4; it was “starred” on the handout.
b- Incorrect. Erythromycin is an antibiotic, which is an inhibitor of CYP3A4; it was “starred” on the handout c-
Incorrect. Itraconazole is an antifungal agent, which is a strong inhibitor of CYP3A4; it was “starred” on the
handout.
d- Incorrect. Probenecid inhibits renal tubule transporters, which can lead to accumulation of renally excreted
drugs that use the same transporter (such as penicillin); it was “starred” on the handout.
e- Incorrect. Ritonavir is an antiretroviral agent, which is a strong inhibitor of CYP3A4; it was “starred” on the
handout.
Item Description: Goldstein, 2014; Drug Toxicities and Drug Interactions handout

Question #: 82
A patient presenting with a morphine overdose is given a rapid-acting antidote that acts at the same receptors
and reverses the effects of the morphine. (Morphine is a mu receptor agonist.) The mechanism of this antidote-
agonist drug action is:
D. Allosteric antagonism
E. Chemical antagonism
F. Partial antagonism
G. Physiologic antagonism
✓E. Receptor antagonism
Rationale:
a- Incorrect. An allosteric antagonist binds to a location near the agonist site of action and changes the
conformation of the agonist site, reducing the affinity of the receptor for the agonist.
b- Incorrect. A chemical antagonist directly binds the drug molecule (or other chemical), thereby neutralizing it
– preventing it from binding to its site of action.
c- Incorrect. The interaction of a partial agonist/antagonist with its receptor produces a submaximal response. At
the same time it prevents the full agonist from binding to the receptor and eliciting the maximum response.
d- Incorrect. Physiological antagonism occurs when an agonist and an antagonist acting at two independent
sites induce independent, but opposite effects.
e- Correct. The stem describes the antidote blocking the effects of the agonist, morphine, by competition for
the same site of action, the mu receptor.
Item Description: Goldstein, 2014; Drug Toxicities and Drug Interactions handout p7,8
Question #: 83
A patient’s sputum sample is sent to the microbiology laboratory for culture and susceptibility testing. The
pathogen is found to have intermediate resistance to the first-line antibiotic. As a result of this resistance, the
drug exhibits reduced:
A. Efficacy
B. Half-life
✓C. Potency
D. Selectivity
E. Spectrum
Rationale:
a- Incorrect. Intermediate resistance to particular antimicrobial agents can be overcome by higher drug
concentration – Emax is not reduced.
b- Incorrect. Half-life is a pharmacokinetic measure inversely proportional to clearance.
c- Correct. Antimicrobial agents are ligands that bind to their targets to produce effects. The relationship
between drug concentration and effect on a population of organisms is modeled using the standard
concentration-response curve for receptor and agonist. Intermediate resistance to particular antimicrobial
agents can be overcome by higher drug concentration. That is, giving a higher dose can achieve the maximum
effect. The dose-response curve is shifted to the right (reduced potency) and the Emax is not decreased.
d- Incorrect. Selectivity refers to a drug’s ability to preferentially produce a particular effect. Selective toxicity is
toxicity to the microbe, sparing the host cell.
e- Incorrect. The antimicrobial spectrum is the range of microorganisms the drug is effect against.
Item Description: Goldstein, 2014; General Principles of Antimicrobial Therapy handout p4
A pharmaceutical company investigating an experimental antibiotic finds that the drug produces a faster kill rate
and more extensive degree of killing as the drug concentrations increase. Additionally, the drug produces
prolonged, persistent antibacterial effects. This activity profile is best described by which one of the following?
A. EC50 right shift
B. Emax decrease
✓C. Peak / MIC
D. T >MIC
E. 24h-AUC / MIC
Rationale:
a- Incorrect. Right shift of EC50 indicates reduced potency of the drug. With antimicrobial therapy, it is an
indicator of increasing resistance.
b- Incorrect. Decreasing Emax indicates reduced efficacy of a drug. With antimicrobial therapy, it is an indicator
of resistance – the drug will not be effective against the particular organism.
c- Correct. Peak / MIC is the pharmacokinetic-pharmacodynamic (PK-PD) profile of drugs that produce faster
kill rate and more extensive killing with increasing drug concentrations. These drugs typically produce a
prolonged, persistent post-antibiotic effect.
d- Incorrect. T > MIC PK-PD profile is representative of antimicrobial agents that produce maximum killing when
the time (T) above MIC is at least 70% of the dosing interval. The ideal dosing regimen for these
antibiotics/antifungals maximizes the duration of exposure. These drugs typically do not produce a prolonged,
persistent post-antibiotic effect.
e- Incorrect. 24h-AUC / MIC correlates with efficacy. The ideal dosing regimen for these antibiotics maximizes
the amount of drug received. These drugs typically show a moderate to prolonged
post-antibiotic effect.
Item Description: Goldstein, 2014; General Principles of Antimicrobial Therapy handout p4-7

Question #: 1
The figure illustrates a circumstance in which: (Please see attachment)
✓A. A fraction of total receptors is needed to produce the maximal response
B. An agonist outcompetes the antagonist and produces the maximal response
C. Receptor downregulation causes reduced response to an agonist
D. Receptor affinity for an agonist is reduced over time
E. Receptors are constitutively active in the absence of an agonist and convert to
their inactive conformation over time
Rationale:
a. The correct answer. Curve A is the agonist response in the absence of an
antagonist. In curve B, maximal responsiveness to the agonist remains after
treatment with relatively low concentrations of an irreversible antagonist, indicating that remaining available
receptors are still in excess of the number required to produce the effect – they are spare receptors. Curve C
shows that the available receptors are adequate to produce the maximal response but they are no longer
“spare.” With higher concentrations of the antagonist, the maximal response to the agonist is depressed, as the
irreversible antagonist blocks the receptor from binding the agonist (Curves D and E).
b. Incorrect. In the presence of a competitive antagonist, an agonist in adequate concentrations would produce
the maximal effect (handout p8).
c. Incorrect. Option c describes receptor desensitization in which cells that are initially sensitive the binding of an
agonist become relatively insensitive to stimulation as a result of a reduction in the number of receptors
available to interact with the ligand. In this circumstance the curve would show initial response and then
decline but continue to show a lower level response (handout p10).
d. Incorrect. The affinity of an agonist for a receptor is an intrinsic property based on the physicochemical
characteristics of the agonist (handout p1).
e: Incorrect. Option e describes a circumstance in which receptor activity occurs without binding of a ligand. The
attached figure describes the effect of increasing concentrations of agonist (“Agonist concentration (C) (log scale))
(handout p7-8).
Source: Title: Goldstein 2013; Pharmacodynamics handout p6, slide 21.

Question # 62
An experimental drug is found to produce its maximum effect at concentrations in which all of its targeted
receptors are bound by the drug. This example describes the relationship between drug dose, receptor
occupancy, and:
✓A. Magnitude of effect
B. Physicochemical properties
C. Physiologic response
D. Therapeutic window
E. Variability in drug serum levels
Rationale:
a- The one best answer. The relationship between drug dose, receptor occupancy, and the magnitude of the
resulting physiologic effect can be demonstrated using dose-response curves where drug concentration is a
function of response and receptor occupancy.
b- Physicochemical properties of a drug are factors in the ability of a drug to cross physiologic barriers and
reach the site of action handout p1
c- The physiologic response is a function of the receptor‟s intrinsic activity handout p2
d- The therapeutic window is determined experimentally from the effect of a series of doses of drug on a
population of subjects.
e- Variability in drug serum levels is related to a drug‟s ability to be pass through the
Source: Goldstein 2013; Pharmacodynamics handout p4,5, and slides 18,20; dose receptor occupancy effect
relationship physiologic barriers, handout p1 and p12

Question #: 63
A researcher investigating the action of a new molecule finds that it inhibits the effect of the full agonist but that
higher doses of the agonist are able to overcome this antagonist‟s inhibitory effect and stimulate the maximal
effect. The researcher determines that the new molecule is a(n):
A. Allosteric antagonist
✓B. Competitive antagonist
C. Irreversible antagonist
D. Noncompetitive antagonist
E. Sensitizer
Rationale:
b- The correct answer. In the presence of a competitive antagonist, an agonist in adequate concentrations
would produce the maximal effect.
a, c, and d- Incorrect. In the presence of noncompetitive antagonists, such as irreversible,
pseudo-irreversible, and allosteric antagonists, the agonist is unable to produce its maximal effect even at high
concentrations.
e- Incorrect. Drug-mediated increased sensitivity to a ligand follows upregulation of receptors resulting from
chronic administration of an antagonist (handout p11).
Source: Goldstein 2013; Pharmacodynamics handout p8, slide 29.
Question #: 53
A hospitalized hypotensive patient is started on a continuous intravenous infusion of a drug that raises blood
pressure. The serum concentration of the drug reaches steady state when the dosing rate equals the clearance
multiplied by:
✓A. Concentration of drug in plasma (Cp)
B. Half-life (t½)
C. Hepatic blood flow (Qhepatic)
D. Maximum rate of drug metabolism (Vmax)
E. Volume of distribution (Vd)
Rationale:
a- Correct. CL = rate of drug elimination / Cp. At steady state, the rate of drug elimination equals the rate of drug
administration. Thus, rearranging the equation, the dosing rate = clearance x steady state concentration (CL x
Css)
b- Incorrect. CL x t½ = Vd x 0.693 Thus, the approximate half-life is related to the volume of distribution and the
clearance.
c- Incorrect. Hepatic blood flow determines the rate of delivery of drug to the liver. With respect to clearance by
the liver (or other organ), the rate of presentation of the drug to the organ is a product of the blood flow (Q), the
concentration of drug in the blood going into the organ, and the rate of exit of the drug from the organ. CLorgan
= (Q[CA – CV]) / CA
d- Incorrect. Vmax is the maximum rate of drug metabolism. The Michalis-Menten rate equation, CL = (Vmax x
C) / (Km + C), usually describes nonlinearity.
e- Incorrect. The volume of distribution relates the amount of drug in the body to the concentration of drug in the
blood (or measured fluid). Vd = Dose / Cp
Item Description: Goldstein, 2011; Clinical Pharmacokinetics handout p14
A series of increasing doses of an experimental drug in preclinical development is administered to a group of
experimental animals. The therapeutic index of this drug in this population is found to be large. This parameter
is indicative of:
A. Competitive antagonism
B. Hepatic blood flow
C. First-order elimination kinetics
D. Time to steady state
✓E. Variation of effect between subjects
Rationale:
a- Incorrect. Competitive antagonism is the mechanism of action. The stem describes effects related to
increasing doses of a drug (any drug) in a population of experimental animals.
b- Incorrect. Hepatic blood flow is a determinant of bioavailability of orally administered drugs.
c- Incorrect. In first-order elimination, the amount of drug eliminated per unit of time depends on the
concentration of drug in the compartment.
d- Incorrect. A drug’s half-life determines the time to steady state. It is a pharmacokinetic parameter and does
not reflect the effect of the drug.
e- Correct. The response elicited with each dose of a drug is described in terms of the cumulative percentage
of subjects exhibiting a defined all-or-none effect and is plotted against the log dose of the drug. The ratio
between the median lethal dose (LD50) and the median effective dose (ED50), LD50 / ED50, provides a
general indication of the margin of safety of a drug
Item Description: Goldstein, 2014; Pharmacodynamics handout p13

MICROBIOLOGY
Which of the following antigens is the most immunogenic?
A. Streptococcal polysaccharide
B. DNA from a hepatitis B virus
C. RNA from HIV
✓D. The protein toxoid from tetanus
Question
E. The lipids#:isolated
68 from the milk of a cow with an udder infection
Rationale:
a- Incorrect. Carbohydrates are antigens that produce an intermediate immune response.
b- Incorrect. Nucleic acids (RNA) are the least immunogenic antigens when compared with other
macromolecules.
c- Incorrect. Nucleic acids (DNA) are poor inducers of the immune response.
d- Correct. Proteins are the most immunogenic antigens of all the macromolecules. e- Incorrect. Lipids are also
poor immunogenic antigens.
Item Description: Dr. Martinez, 2014, Immune response, PowerPoint

Question #: 69
When a cell is infected and presents the antigen of the infection, in which molecule does it present the antigen
and which type of cell responds?
A. Class II molecule; CD4+ T cell
B. Class II molecule; CD8+ T cell
✓C. Class I molecule; CD8+ T cell
D. Class I molecule; CD4+ T cell
E. Toll-like receptors; macrophages
Rationale:
A- Incorrect. Intracellular antigens can’t be presented by an MHC class II molecule on the surface of the infected
cells to a CD4+ cell. MHC class I molecules are responsible to present intracellular antigens on infected cells.
B- Incorrect. MHC class II molecules only present extracellular antigens that can’t be recognized by CD8+ T
cells. CD4+ T cells recognize extracellular antigen bound to MHC II.
C- Correct. Intracellular antigens are presented by an MHC class I molecule on the surface of the infected cells
to a CD8+ cell.
D- Incorrect. CD4+ T cells can’t recognize antigen presented by MHC I instead they recognize antigen
presented only by MHC II.
E- Incorrect. Toll-like receptors on the surface of macrophages are part of innate immunity that recognizes
extracellular structurally conserved molecules derived from microbes.
Item Description: Dr. Martinez, 2014, Immune response, PowerPoint
Question #: 70
Compared to a secondary response, a primary response is:
A. Faster and higher in amount of IgG
✓B. Slower and lower in amount of IgG
C. Slower but higher in amount of IgG
D. Slower and lower in amount of IgM
E. Faster, more specific but lower in amount of IgG
Rationale: Rationale
a- Incorrect. A primary immune response is slower and lower in amount of IgG than a secondary response. b-
Correct. A primary immune response is slower and lower in amount of IgG than a secondary response. c-
Incorrect. A primary immune response is slower but lower in amount of IgG than a secondary response. d-
Incorrect. A primary immune response is slower but higher in amount of IgM than a secondary response. e-
Incorrect. A primary immune response is slower and lower in amount of IgG than a secondary response. Item
Description: Dr. Martinez, 2014, Immune response, PowerPoint

Question #: 71
The designated purpose of the “A” (bacitracin)-disc is to distinguish:
A. Alpha-hemolytic bacteria from beta-hemolytic bacteria
B. Staphylococcus aureus from other staphylococcal bacteria
C. Staphylococcus aureus from streptococcus pyogenes
✓D. Streptococcus pyogenes from other beta hemolytic streptococcus
E. Streptococcus pneumoniae from other alpha-hemolytic streptococcus
Rationale:
Question #:The
a- Incorrect. 42 A disc is not designed to distinguish these 2 different groups of bacteria. b- Incorrect. The A
disc is not designed to distinguish these 2 different groups of bacteria c- Incorrect. The A disc is not designed to
distinguish these 2 different groups of bacteria
d- Correct. For this group of Strept. bacteria, the “A” (bacitracin) disc is designed to distinguish S. pyogenes from
other beta-hemolytic Strept.
e- Incorrect. The A disc is not designed to distinguish these 2 different groups of bacteria
Item Description: Dr. Pavia, 2014, Detection of Microbes, Page 5 of lecture notes & PowerPoint slides

Question #: 43
The reverse transcriptase of HIV:
A. Transcribes viral DNA to cell DNA
✓B. Transcribes viral RNA to DNA
C. Transcribes cellular DNA to viral DNA
D. Is attached to the viral DNA
E. Translate viral RNA to DNA
Rationale:
A- Incorrect. The genome of HIV consist of RNA, therefore, reverse transcriptase cannot transcribe viral DNA.
Similarly, viral RNA is transcribed to viral DNA not cell DNA.
B- Correct. The reverse transcriptase of HIV transcribes viral RNA to DNA.
C- Incorrect. Viral RNA is converted to viral DNA which gets integrated to the cell DNA genome.
D- Incorrect. Although viral DNA is produced by reverse transcriptase the virus uses the DNA polymerases of
the host to replicate the viral genome.
E- Incorrect. Translation involves the conversion of RNA to proteins not to DNA.
Item Description: Dr. Martinez, 2014, Microbiology-Fungi and Viruses, PowerPoint
Which is the correct sequence (first to last) of events in viral replication?
A. Penetration, uncoating, synthesis, adsorption, assembly, release
B. Uncoating, penetration, synthesis, assembly, absorption, release
✓C. Adsorption, penetration, uncoating, synthesis, assembly, release
D. Assembly, synthesis, uncoating, release, penetration, adsorption
E. Synthesis, uncoating, release, penetration, adsorption, assembly
Rationale:
A- Incorrect. Adsorption is the first step of viral replication.
B- Incorrect. Uncoating and genome release occurs after penetration of the virus into the cell.
C- Correct. Viral replication involves the following sequence of events: adsorption or attachment to the cell,
penetration, uncoating and genome release, synthesis of genome and proteins, assembly of viral particles,
release from the host cell.
D- Incorrect. The assembly of viral particles occurs after the synthesis of genome and proteins. E- Incorrect.
The synthesis of genome and proteins cannot occur without the uncoating step.
Item Description: Dr. Martinez, 2014, Microbiology-Fungi and Viruses, PowerPoint

Question #: 45
A 53-year-old man with a renal transplant develops shortness of breath and fevers. His chest radiograph
reveals a dense left upper lobe infiltrate. A bronchoscopy with biopsy is performed. The pathologist calls to
inform you that encapsulated yeast cells ~5 micrometer (μm) in diameter are present in the biopsy specimen.
Which of the following is the cause of this patient’s disease?
A. Histoplasma capsulatum
B. Aspergillus fumigatus
✓C. Cryptococcus neoformans
D. Blastomyces dermatitidis
E. Candida albicans
Rationale:
a- Incorrect. Although the cell bodies of Histoplasma capsulatum appeared to have a capsule, the fungus
does not have a true capsular structure.
b- Incorrect. Aspergillus fumigatus is a mold.
c- Question
Correct. #: 44
Cryptococcus neoformans is the only eukaryotic encapsulated pathogen that protects the fungus in
both environment and inside the host.
d- Incorrect. Blastomyces dermatitidis is not encapsulated. e- Incorrect. Candida albicans is not encapsulated.
Item Description: Dr. Martinez, 2014, Microbiology-Fungi and Viruses, PowerPoint

Question #: 47
Which of the following is an accurate description for the pathogen that can cause encephalitis in
immunocompromised patients and congenital abnormalities, and can be acquired from contact with infected
cat feces?
✓A. Protozoan parasite
B. Acid-fast bacillus
C. Gram negative bacillus
D. Gram positive coccus
E. Dimorphic fungus
Rationale:
A- correct. The clinical and microbiologic information provided in the question is consistent for the protozoan
parasite Toxoplasma gondii. None of the other choices fit.
B- incorrect. This choice is not compatible with the information in the question C- incorrect. This choice is not
compatible with the information in the question D- incorrect. This choice is not compatible with the information in
the question E- incorrect. This choice is not compatible with the information in the question
Item Description: Dr. Pavia, 2014, Basic Microbiologic Principles; Page 4 of lecture notes.
Question #: 48
Your patient is a 9-year-old boy who presents with a fever and a severe sore throat. A rapid test for Group A
streptococcal antigen is done using a throat swab and it comes back positive. Which of the following tests
would confirm the likely diagnosis?
A. Giemsa stain of a patient sample
B. Growth on MacConkey agar
C. Presence of a peptidoglycan capsule
D. Alpha hemolysis on a blood agar plate
✓E. Inhibition of growth by bacitracin
Rationale:
A – incorrect. Giemsa stain is used for Chlamydia/Chlamydophila bacteria.
B – incorrect. MacConkey agar is for identifying gram-negative enteric bacilli & is not for Group A Strept. C –
incorrect. Goup A strept has a hyaluronic acid capsule
D – incorrect. alpha hemolysis would be a strong indicator for Strept. pneumoniae; there are no strains of
Group A strept that produce alpha hemolysis
E – correct. the stem of the question strongly suggests that Group A strept is the cause of this patient’s
problem. Group A strept. is sensitive to the inhibitory effects of bacitracin
Item Description: Dr. Pavia, 2014, Microbiology- Bacteria; Page 11 of lecture notes & ppt. slides.

ANATOMY
After recovery from surgery involving the temporomandibular joint, a patient complains that his food
tastes differently. From various tests you conclude that the patient retains general sensation on his
entire tongue, but has lost taste from the left side of his tongue anterior to the vallate papillae. What is
the most likely site of the nerve lesion he has experienced?

A. Lingual nerve proximal to chorda tympani


B. Lingual nerve distal to chorda tympani
C. Chorda tympani
D. Hypoglossal nerve
E. Glossopharyngeal nerve
Rationale:

a. Incorrect. Lingual nerve proximal to chorda tympani carries general somatic afferents from the tongue,
not taste.
b. Incorrect. Lingual nerve distal to chorda tympani carries general somatic afferents and taste from the
tongue, so general sensation would be lost as well if this were injured.
c. Correct. Chorda tympani carries taste sensation from the tongue, nothing else.
d. Incorrect. Hypoglossal nerve only carries general somatic efferents to intrinsic muscles of the tongue,
not general sensation or taste.
e. Incorrect. Glossopharyngeal nerve only carries taste sensations from a small region of the posterior 1/3
of the tongue, posterior to the vallate papillae.

Item Description: Dr. Beatty, Somatic Afferents & Efferents

A child in the hospital cafeteria is drinking milk and her friend causes her to suddenly laugh. The milk
accidentally passes through her nose. The timing of the elevation of the soft palate to close off the
nasopharynx was poor. What cranial nerve responsible for elevating the soft palate had this poor
reaction timing?

A. Trigeminal nerve (CN V)


B. Glossopharyngeal nerve (CN IX)
C. Vagus nerve (CN X)
D. Facial nerve (CN VII)
E. Olfactory nerve (CN I)

Rationale:

a. Incorrect. General somatic efferent innervation of the muscle that elevates the soft palate, levator veli
palatini, is provided by vagus nerve, not trigeminal nerve.
b. Incorrect. General somatic efferent innervation of the muscle that elevates the soft palate, levator veli
palatini, is provided by vagus nerve, not glossopharyngeal nerve.
c. Correct. General somatic efferent innervation of the muscle that elevates the soft palate, levator veli
palatini, is provided by vagus nerve.
d. Incorrect. General somatic efferent innervation of the muscle that elevates the soft palate, levator veli
palatini, is provided by vagus nerve, not facial nerve.
e. Incorrect. General somatic efferent innervation of the muscle that elevates the soft palate, levator veli
palatini, is provided by vagus nerve, not olfactory nerve.

Item Description: Dr. Beatty, Somatic Afferents & Efferents

A patient presents with the left side of his tongue completely atrophied, unable to change in shape. He
can still retract it. What cranial nerve is most likely affected?

A. Glossopharyngeal nerve (CN IX)


B. Facial nerve (CN VII)
C. Trigeminal nerve (CN V)
D. Hypoglossal nerve (CN XII)
E. Vagus nerve (CN X)

Rationale:

a. Incorrect. Intrinsic muscles of the tongue receive general somatic efferents from hypoglossal nerve, not
glossopharyngeal nerve.
b. Incorrect. Intrinsic muscles of the tongue receive general somatic efferents from hypoglossal nerve, not
facial nerve.
c. Incorrect. Intrinsic muscles of the tongue receive general somatic efferents from hypoglossal nerve, not
trigeminal nerve.
d. Correct. Intrinsic muscles of the tongue receive general somatic efferents from hypoglossal nerve.
e. Incorrect. Intrinsic muscles of the tongue receive general somatic efferents from hypoglossal nerve, not
vagus nerve.

Item Description: Dr. Beatty, Somatic Afferents & Efferents

After a surgical procedure that removed a tumor from the parotid gland, a patient presents with
difficulty moving his eyebrows and wrinkling the skin of his forehead. Which nerve is most likely to
have received a lesion during the surgical procedure and caused this disability?

A. Ophthalmic nerve
B. Supraorbital nerve
C. Cervical branch of facial nerve
D. Auriculotemporal nerve
E. Temporal branch of facial nerve

Rationale:

a. Incorrect. Ophthalmic nerve provides general somatic afferents to the skin of the forehead, not general
somatic efferents to muscles of facial expression there.
b. Incorrect. Supraorbital nerve provides general somatic afferents to the skin of the forehead, not general
somatic efferents to muscles of facial expression there.
c. Incorrect. Cervical branch of facial nerve provides general somatic efferents to muscles of facial
expression, but those of the neck, not the forehead region.
d. Incorrect. Auriculotemporal nerve provides general somatic afferents to skin of the temple, not general
somatic efferents to muscles of facial expression.
e. Correct. Temporal branch of facial nerve provides general somatic efferents to muscles of facial
expression of the forehead region.

Item Description: Dr. Beatty, Somatic Afferents & Efferents

A patient presents with his mandible depressed and is unable to close his mouth. His panicked friend
explains that he was just having a big yawn during a lecture, and it stuck that way. Upon inspection
you note that it isn’t the result of a dysfunctional muscle or nerve lesion, but that he seems to have
dislocated his temporomandibular joint. The mandibular condyles have protruded anterior to the
articular tubercles. Which of the following muscles is most likely to be responsible for depressing and
protracting the mandibular condyles so?

A. Anterior belly of digastric


B. Medial pterygoid
C. Buccinator
D. Temporalis
E. Lateral pterygoid

Rationale:

a. Incorrect. The lateral pterygoid protracts the mandibular condyle and would be most likely muscle
responsible here, not the anterior belly of digastric, which retracts the mandible.
b. Incorrect. The lateral pterygoid protracts the mandibular condyle and would be most likely muscle
responsible here, not the medial pterygoid, which protracts and translates the mandible.
c. Incorrect. The lateral pterygoid protracts the mandibular condyle and would be most likely muscle
responsible here, not the buccinator, which keeps food on the tooth surfaces.
d. Incorrect. The lateral pterygoid protracts the mandibular condyle and would be most likely muscle
responsible here, not the temporalis, which elevates and retracts the mandible
e. Correct. The lateral pterygoid protracts the mandibular condyle and would be most likely muscle
responsible here.

Item Description: Dr. Beatty, Somatic Afferents & Efferents

A patient presents with permanent numbness to his lower lip and lower teeth after oral surgery.
Branches of which cranial nerve were most likely injured?

A. Hypoglossal nerve (CN XII)


B. Oculomotor nerve (CN III)
C. Ophthalmic nerve (CN V1)
D. Facial nerve (CN VII)
E. Mandibular nerve (CN V3)

Rationale:

a. Incorrect. The hypoglossal nerve provides general somatic efferents to the intrinsic muscles of the
tongue, not general somatic afferents to the lower lip and teeth.
b. Incorrect. The oculomotor nerve provides somatic efferents to the some extraocular muscles, not
general somatic afferents to the lower lip and teeth.
c. Incorrect. The ophthalmic nerve provides general somatic afferents to a large part of the upper face and
orbit, not general somatic afferents to the lower lip and teeth.
d. Incorrect. The facial nerve provides general somatic efferents to muscles of facial expression, not
general somatic afferents to the lower lip and teeth.
e. Correct. The mandibular nerve provides general somatic afferents to the lower lip and teeth.

Item Description: Dr. Beatty, Somatic Afferents & Efferents

When tilting the head, a little bit of torsion of the eyeball is needed. Which of the following muscles
contributes to intorsion of the eyeball?

A. Inferior oblique
B. Inferior rectus
C. Lateral rectus
D. Medial rectus
E. Superior rectus

SOURCE: Dr. Geisler lecture on 11/30/2012; Moore, Dalley, and Agur pp. 898-903. Structural/Functional
Thread

A patient presents with an inability to abduct her right eye. Which of the following is the most likely
cause?

A. A tear in the left medial rectus muscle


B. Increased intracranial pressure on the optic nerve
C. Lesion on the right cranial nerve VI
D. Lesion on the right trochlear nerve
E. Lesion on the right oculomotor nerve

SOURCE: Dr. Geisler lecture on 11/30/2012; Moore, Dalley, and Agur pp. 898-900.
Structural/Functional Thread

A contestant on an obstacle course game show (like Wipeout) receives a severe blow to the head,
causing multiple fractures to his face around the orbits. One of these was a blowout fracture that led to
lesions to multiple branches of oculomotor nerve (CN III) within the orbit. The patient is exhibiting
difficulty constricting his pupil in response to having a light shined in his eye, yet indicates that he can
see with it just fine. What lesion is this most likely the result of?

A. Parasympathetics to constrictor pupillae


B. Sympathetics to dilator pupillae
C. Parasympathetics to the ciliary body
D. Sympathetics to Müller’s muscle
E. Parasympathetics to the lacrimal gland

Rationale:

a. correct. The difficulty constricting the pupil indicates that constrictor pupillae is affected, which is
controlled by parasympathetics.
b. Incorrect. Dilator pupillae does not constrict the pupil.
c. Incorrect. The ciliary body does not constrict the pupil.
d. Incorrect. Müller’s muscle does not constrict the pupil.
e. Incorrect. The lacrimal gland does not constrict the pupil.

Item Description: Dr. Beatty, Autonomics

A college student cycling to class gets hit by a car and receives lacerations to his face around his
cheeks. He is grateful to have avoided injury to branches of facial nerve, but needed stitches to deal
with arterial lesions. After recovery, he presents with a complaint that he seems to blush on his cheeks
unevenly. A lesion to which of the following nerves is most likely to be causing these symptoms?

A. Facial nerve
B. External carotid plexus
C. Ophthalmic nerve
D. Internal carotid plexus
E. Nerve of the pterygoid canal

Rationale:

a. Incorrect. Facial nerve provides efferents to muscles of facial expression, not sympathetic innervation to
skin.
b. Correct. The external carotid plexus includes the sympathetic nerves controlling blushing to the face
below the eye, including the cheeks.
c. Incorrect. Ophthalmic nerve provides general somatic afferents to the skin above the eye, not
sympathetics to the skin of the cheek
d. Incorrect. The internal carotid plexus includes the sympathetic nerves controlling blushing to the face
above the eye, excluding the cheeks.
e. Incorrect. The nerve of the pterygoid canal provides autonomics to structures in the eyeball, but not
sympathetics to the skin of the cheeks.

Item Description: Dr. Beatty, Autonomics

During a match, a mixed martial arts fighter receives a blow to the eye, resulting in a blowout fracture
of the floor of the orbit and lesion to the nerve lying there. This nerve is part of what cranial nerve?

A. Optic nerve (CN II)


B. Olfactory nerve (CN I)
C. Facial nerve (CN VII)
D. Glossopharyngeal nerve (CN IX)
E. Trigeminal nerve (CN V)

SOURCE: Dr. Beatty Structural/Functional Thread


After recovery from oral surgery, a patient complains that he keeps biting his tongue and food tastes
differently. From various tests you conclude that the patient has lost general sensation and taste from
the left side of his tongue anterior to the vallate papillae. What is the most likely site of the nerve lesion
he has experienced?

A. Lingual nerve proximal to chorda tympani


B. Lingual nerve distal to chorda tympani
C. Chorda tympani
D. Hypoglossal nerve
E. Glossopharyngeal nerve

Rationale: Answer choices A and B are accepted. As per Dr. Beatty, “Lingual nerve was described as proximal
and distal portions with respect to its origin, as proximal and distal terms always mean. The division between
those two lengths of the lingual nerves was defined by the connection of the chorda tympani, and this was
point out multiple times in lecture. If one suggests that the wording indicates that the answers provided were
describing being about their distance from the chorda tympani itself, neither answer would be right. Neither
would distinguish on what end of lingual nerve the lesion was found. I can accept that the wording of choices A
and B could be better. Therefore, choices A & B should be accepted.”

SOURCE: Dr. Beatty Structural/Functional Thread

A parent comes to you worried that his child seems to commonly choke on his food, often finding that
he has swallowed extremely large portions. You find that if you give the child small portions of food or
liquid, he swallows normally without problems. Upon inspection, you find that a tongue depressor
gently touching the back of the throat (oropharynx) results in no gag reflex. What cranial nerve is most
likely dysfunctional?

A. Vagus nerve (CN X)


B. Hypoglossal nerve (CN XII)
C. Trigeminal nerve (CN V)
D. Glossopharyngeal nerve (CN IX)
E. Facial nerve (CN VII)

Rationale: As per Dr. Beatty, “There are three clearly stated reasons why choice D is the only choice. 1) In
class, I pointed out that there are signs that vagus nerve may occasionally be involved, but I also stated that for
all testing reasons they will ever have, including my own, that they should remember this as glossopharyngeal
nerve as the afferent limb of the reflex. 2) The question asked about what cranial nerve was dysfunctional, but
if vagus was dysfunctional one would not be able to swallow as normal at all, as the motor control of muscles
involved would be affected. 3) The question asks for which cranial nerve is “most likely affected.” Even in the
context of the more subtle details of reality discussed in the classroom session about how vagus nerve may be
involved in some cases (as stated in the slide), the majority of times glossopharyngeal nerve is the afferent
limb, making it the most likely nerve affected.”

SOURCE: Dr. Beatty Structural/Functional Thread

You and your friends are having coffee, and someone causes you to suddenly laugh. The hot coffee
accidentally passes through your nose, causing it to burn. What structure moved that allowed the
coffee to enter your nose?

A. Soft palate
B. Larynx
C. Hard palate
D. Mandible
E. Pharyngotympanic tube
SOURCE: Dr. Beatty Structural/Functional Thread

An airline passenger’s newborn is screaming in pain during takeoff and landing, and the father
attempts to help ease the baby’s pain by having her nurse from her bottle. It works, and this is
primarily because the baby is opening her ________________ using her _______________.

A. Pharyngotympanic tube; tensor veli palatini


B. Pharynx; superior pharyngeal constrictor
C. Laryngopharynx; epiglottis
D. External auditory meatus; levator veli palatini
E. Mouth; orbicularis oris

SOURCE: Dr. Beatty Structural/Functional Thread

A private aircraft pilot has suffered a plane crash, which resulted in his safety harness damaging both
his right and left laryngeal nerves. He is now only able to make a few squeaking sounds when he tries
to speak. What is the action of the muscle whose innervation is left intact?

A. Abducting the vocal cords


B. Adducting the arytenoid cartilages
C. Adducting the vocal ligament
D. Relaxing the vocal ligament
E. Stretching or tensing the vocal ligament

Rationale: As per Dr. Beatty, “The question clearly states that both left and right recurrent laryngeal nerves are
injured and “Arytenoid cartilage help the vocal cords in being tense. The setup of the question requires one to
realize that all of the muscles of the larynx have lost innervation except for the one not getting innervated by
the recurrent laryngeal nerves – crocothyroid. Cricothyroid’s function is in rocking the thyroid cartilage
anteriorly to cause tension in the vocal ligament, not necessarily the adduction of the arytenoid cartilages (the
vocal cords can be pulled in tension without those being adducted). Therefore, choice B is not a proper
answer.”

SOURCE: Dr. Beatty Structural/Functional Thread

A patient presents without a pupillary light reflex, being unable to constrict his pupil. He also
complains about having trouble seeing things closely. You are concerned that he has disrupted nerves
going to his eye. What is most likely dysfunctional?

A. Parasympathetics to the ciliary muscles


B. Sympathetics to dilator pupillae
C. Somatic efferents to superior oblique
D. Sympathetics to Müller’s muscle
E. Somatic afferents to the cornea

SOURCE: Dr. Beatty Structural/Functional Thread

A 32-year-old woman has been repeatedly experiencing vertigo, suddenly becoming dizzy and unable
to determine up or down when her eyes are closed. What organ’s dysfunction is most likely
responsible for this sensation?

A. Vestibule
B. Malleus
C. Cochlea
D. Pharyngotympanic tube
E. Stapes
SOURCE: Dr. Beatty Structural/Functional Thread

A patient recovering from a virally induced Bell’s palsy presents with an unusual circumstance in
which he sheds tears when he is eating, known as “crocodile tears syndrome”. The improperly healed
nerve connections he has experienced have resulted in parasympathetics that normally go to
____________ to be redirected to _______________.

A. Lacrimal glands; salivary glands


B. Sweat glands; salivary glands
C. Salivary glands; lacrimal glands
D. Lacrimal glands; sweat glands
E. Sweat glands; lacrimal glands

SOURCE: Dr. Beatty Structural/Functional Thread

During surgery of the pituitary in which you pass through the sphenoid sinus to get to the lateral sides
of the gland, your proximity to the internal carotid artery results in an error that causes damage to the
fascia and nerves surrounding the internal carotid. After recovery, you test to see if the sympathetics
that may have been damaged are intact by testing for heat-induced swelling of skin on the
___________ .

A. Back of the head


B. Forehead
C. Cheek
D. Chin
E. Neck

SOURCE: Dr. Beatty Structural/Functional Thread

A 17-year-old male was involved in a gang related fight in which he was beaten with a baseball bat. He
suffered multiple fractures, one of which was to the temporal bone leading to a lesion to facial nerve
(CN VII) just proximal to where the chorda tympani branches. The patient is exhibiting facial paralysis
on the side of the fracture and a sensitivity to certain sounds. What most relevant autonomic function
is spared pertaining to this cranial nerve lesion?

A. Lacrimal secretion
B. Parotid secretion
C. Sublingual secretion
D. Submandibular secretion
E. Sweat secretion

Rationale: As per Dr. Beatty, “The question is asking what function is spared pertaining to this cranial nerve
lesion, NOT other cranial nerves. It was clearly explained in the classroom session and VPG how one could
think through the functions retained or lost due to lesions of the facial nerve at different points on its pathway.
Sweat secretion and the parotid are not controlled by facial nerve, and submandibular and sublingual are
controlled by parts of facial nerve that would be affected by the lesion, making lacrimal secretion the only
logical choice. The portions of lingual nerve that comes from trigeminal would still be travelling to the tongue,
but is uninvolved in the autonomic control of any of the choices given.”

SOURCE: Dr. Beatty Structural/Functional Thread

Which of the following groups of cranial nerves is composed only of special afferent function?

A. Optic nerve, vestibulocochlear nerve, and olfactory nerve


B. Oculomotor nerve, olfactory nerve, and optic nerve
C. Optic nerve, trochlear nerve, and olfactory nerve
D. Hypoglossal nerve, olfactory nerve, and abducens nerve
E. Optic nerve, spinal accessory nerve, vestibulocochlear nerve

SOURCE: Dr. Beatty Structural/Functional Thread

A cyclist that was the victim of a hit and run is brought into the emergency department unconscious.
Superficial injuries to his head make you concerned that he has experienced internal trauma, and you
need to rapidly identify whether he is experiencing elevated intracranial pressure. You inspect his eyes
with an ophthalmoscope and note signs of papilledema (swelling of the optic disc). Which of the
following is most likely directly involved in this symptom?

A. Optic nerve
B. Trochlear nerve
C. Abducens nerve
D. Ophthalmic nerve
E. Oculomotor nerve

Rationale:

a. Correct. Increased intracranial pressure is most likely causing pressure on the cranial nerve directly
connected to the optic disc, the optic nerve.
b. Incorrect. Increased intracranial pressure is most likely causing pressure on the cranial nerve directly
connected to the optic disc, the optic nerve. Trochlear nerve is shielded by the cavernous sinus and does not
have direct contact with the optic disc.
c. Incorrect. Increased intracranial pressure is most likely causing pressure on the cranial nerve directly
connected to the optic disc, the optic nerve. Abducens nerve is shielded by the cavernous sinus and does not
have direct contact with the optic disc.
d. Incorrect. Increased intracranial pressure is most likely causing pressure on the cranial nerve directly
connected to the optic disc, the optic nerve. Ophthalmic nerve is shielded by the cavernous sinus and does not
have direct contact with the optic disc.
e. Incorrect. Increased intracranial pressure is most likely causing pressure on the cranial nerve directly
connected to the optic disc, the optic nerve. Oculomotor nerve is shielded by the cavernous sinus and does not
have direct contact with the optic disc.

Item Description: Dr. Beatty, Special afferents

A college student at a concert enthusiastically dancing and shaking his head suddenly feels extremely
dizzy and drops to the floor. His friends joke and guess about whether he has ingested too much
alcohol or gotten injured by another person. After a moment the friend explained it was neither, and
simply explains that he was shaking his head and it happened suddenly. What organ’s dysfunction is
most likely responsible for this sensation?

A. Vestibule
B. Malleus
C. Cochlea
D. Pharyngotympanic tube
E. Stapes

Rationale:

a. Correct. The vestibule contains the organs balance, whose dysfunctions typically manifests as vertigo.
b. Incorrect. The vestibule contains the organs balance, whose dysfunctions typically manifests as vertigo.
A dysfunction of the malleus would result in hearing deficits.
c. Incorrect. The vestibule contains the organs balance, whose dysfunctions typically manifests as vertigo.
A dysfunction of the cochlea would result in hearing deficits.
d. Incorrect. The vestibule contains the organs balance, whose dysfunctions typically manifests as vertigo.
A dysfunction of the pharyngotympanic tube would not result in vertigo.
e. Incorrect. The vestibule contains the organs balance, whose dysfunctions typically manifests as vertigo.
A dysfunction of the stapes would result in hearing deficits.

Item Description: Dr. Beatty, Special afferents

A librarian wakes up one morning to find that he cannot smell his coffee, and upon examination from
his doctor finds that he has a brain tumor that has grown overnight enough to impinge on a cranial
nerve as it passes through the skull. Where is the tumor located?

A. Internal acoustic meatus


B. Superior orbital fissure
C. Cribriform plate
D. Inferior orbital fissure
E. Jugular foramen

Rationale:

a. Incorrect. The loss of olfactory sense indicates that the olfactory nerve is dysfunctional. It passes
through the cribriform plate of the ethmoid, not the internal acoustic meatus.
b. Incorrect. The loss of olfactory sense indicates that the olfactory nerve is dysfunctional. It passes
through the cribriform plate of the ethmoid, not the superior orbital fissure.
c. Correct. The loss of olfactory sense indicates that the olfactory nerve is dysfunctional. It passes through
the cribriform plate of the ethmoid.
d. Incorrect. The loss of olfactory sense indicates that the olfactory nerve is dysfunctional. It passes
through the cribriform plate of the ethmoid, not the inferior orbital fissure.
e. Incorrect. The loss of olfactory sense indicates that the olfactory nerve is dysfunctional. It passes
through the cribriform plate of the ethmoid, not the jugular foramen.

Item Description: Dr. Beatty, Special afferents

You test the extraocular function of a 32-year-old female by having her follow an “H” pattern you make
with your finger. Generally she is able to follow your finger, except that she is not able to elevate her
right eye after abduction. Which of the following is the most likely affected structure?

A. Right superior oblique muscle


B. Right superior rectus muscle
C. Right inferior rectus muscle
D. Right inferior oblique muscle
E. Right medial rectus muscle

Rationale:

a. Incorrect. The superior oblique is the primary depressor of the adducted eye, but the patient was able to
do this.
b. Correct. The superior rectus is the primary elevator of an abducted eye, thus failure to elevate the
abducted eye indicates that this muscle is affected.
c. Incorrect. The inferior rectus is the primary depressor of the abducted eye, but the patient was able to
do this.
d. Incorrect. The inferior oblique is the primary elevator of the adducted eye, but the patient was able to do
this.
e. Incorrect. The medial rectus is the primary adductor of the eye, but the patient was able to do this.
Item Description: Dr. Geisler, The Orbit and Extraocular Muscles, Clinically Oriented Anatomy 7e, pp. 898-899

A 60-year-old male presents with ptosis of the left eye. Two years ago he had a malignant brain tumor
removed. Upon opening the left eyelid you notice that the pupil is dilated and stationary in the
inferolateral corner of the orbit. What is the most likely affected structure?

A. Left trochlear nerve


B. Left abducens nerve
C. Only the superior division of the left oculomotor nerve
D. Left oculomotor nerve
E. Only the inferior division of the left oculomotor nerve

Rationale:

a. Incorrect. The trochlear nerve innervates the superior oblique, which depresses the eye. The eye is still
able to make this movement.
b. Incorrect. The abducens nerve innervates the lateral rectus, the primary abductor of the eye. The eye is
abducted, thus this muscle and its innervation are still functional.
c. Incorrect. If only the superior division is affected then the inferior division should be intact. Thus
muscles innervated by the inferior division (inferior rectus, medial rectus, and inferior oblique) should be
functioning. The medial rectus would adduct the eye and the inferior oblique would elevate it, but the eye is
abducted and depressed.
d. Correct. These are all classic symptoms of a complete palsy of the oculomotor nerve. This nerve
innervates the superior, inferior, and medial rectus; it innervates the inferior oblique; it innervates the levator
palpebrae superioris; and carries parasympathetic fibers that innervate the constrictor pupillae. The only
functioning extraocular muscles are the superior oblique (innervated by trochlear nerve) and lateral rectus
(innervated by the abducens nerve). The former depresses the eye and the latter abducts the eye, and both
movements are unopposed. Thus the pupil shifts to the inferolateral corner of the orbit. Gravity overcomes the
nonfunctioning levator palpebrae superioris, causing ptosis, and the sphincter pupillae cannot act as an
antagonist to the dilator pupillae. The dilator pupillae remains active because sympathetic fibers are conveyed
on a different nerve, specifically a sympathetic root of the otic ganglion that branches off of a plexus on the
internal carotid artery.
e. Incorrect. If only the inferior division was affected, then the superior division should be intact. Thus
muscles innervated by the superior division (superior rectus, levator palpebrae superioris) should be
functioning. The superior rectus would elevate the eye and the levator palpebrae superioris would elevate the
superior tarsus, but there is ptosis and the eye is depressed.

Item Description: Dr. Geisler, The Orbit and Extraocular Muscles, Clinically Oriented Anatomy 7e, pp. 898-899,
913

Which of the following is a correct statement?

A. The anterior pituitary is an ectodermal structure


B. The posterior pituitary originates from the epiphysis
C. Meckle’s cartilage subdivides to form the stapes
D. The tongue muscles are derived from pharyngeal muscle rudiments
E. The pharyngeal muscles are derived from postotic somites

Rationale:
a. Correct – the anterior pituitary derives from ectoderm and enters the head via Rathke’s pouch in the nasal cavity.
b. Incorrect- the posterior pituitary originates from the hypophysis of the brain
c. Incorrect- Meckel’s subdivides to from the malleus and incus
d. Incorrect- the tongue muscles are derived from the post otic head somites
e. Incorrect- the pharyngeal muscles re derived from branchial muscle rudiments

Item Description: Dr. Solounias, Head and Neck


A child is born with an external opening on the side of its neck (bilateral). Upon examination, pharyngeal
contents leak out of this opening. The ear of this patient is normal. This opening is most likely a defect with the:

A. Pharyngeal clefts
B. Otic placodes
C. Pharyngeal arch 1
D. Eustachian tubes
E. Foramen cecum

Rationale:
a. Correct – the pharyngeal clefts are open in all embryos but close to form the external neck surface
b. Incorrect- the otic placode forms the ear not the neck
c. Incorrect- Arch 1 is involved in the jaw- ear and it is not part of the neck
d. Incorrect- The Eustachian tubes form from pharyngeal pouch one and the ear was normal in this patient
e. Incorrect- foramen cecum is the place on the tongue where the thyroid passes through

Item Description: Dr. Solounias, Head and Neck

CN XII (hypoglossal nerve) provides efferent innervation to tissues derived from:

A. A postotic somite
B. A preotic somite
C. The base of arch 2
D. The base of arch 4
E. The otic capsule

Rationale: There is nothing pharyngeal (relating to the pharynx) for the hypoglossal nerve.

SOURCE: Dr. Solounias Structural/Functional Thread

A young girl is brought into the office with speech and swallowing problems caused by her tongue.Upon
examination, a benign mass in her tongue is identified. This mass is most likely:

A. Tissue from the thyroid


B. Remnants of the palatine tonsils
C. Intrinsic tongue muscle
D. Parathyroid tissue
E. A neurgenic placode

SOURCE: Dr. Solounias Structural/Functional Thread

Question #: 126
Which of the following structures is derived from an endochondral ossification?
A. Frontal bone
B. Meckel’s cartilage
C. Teeth
D. Nasal mucosa
✓E. Sphenoid bone
Rationale:
A- Incorrect – this is intramembranous
B- Incorrect- this is a branchial arch one cartilage C- Incorrect – this is ectodermal
D- Incorrect- this is derived from the nasal placode
E- Correct- this is derived from the neurocranium and is endochondral
Item Description: Dr. Solounias
Question #: 127
Which of the following structures is derived from a placode?
A. Mandibular jaw
B. Nasal bone
✓C. Inner ear cavity
D. Posterior pituitary
E. Genioglossus
Rationale:
A-Incorrect – this is intramembranous in origin B-Incorrect- this is intramembranous in origin
C- Correct – this originates from the otic placode D- Incorrect- this is part of the brain
E- Incorrect- this is derived from post otic somites
Item Description: Dr. Solounias

A patient presents with acute blockage of the submandibular duct resulting in inflammation that compresses
the lingual nerve nearby, in the floor of the oral cavity. What functional modality may be interfered with if the
lingual nerve is affected?
✓A. Taste to the anterior 2/3 of the tongue
B. Salivary production from the parotid gland
C. Taste to the posterior 1/3 of the tongue
D. Motor innervation to intrinsic tongue muscles
E. Motor innervation to extrinsic tongue muscles
Rationale:
a- Correct. taste to the anterior 2/3 of the tongue is provided by this portion of the lingual nerve.
b- Incorrect. Salivary production from the parotid gland is controlled by glossopharyngeal nerve via
auriculotemporal nerve.
c- Incorrect. Taste to the posterior 1/3 of the tongue is provided by glossopharyngeal and vagus nerves. d-
Incorrect. Intrinsic tongue muscles receive motor innervation from many hypoglossal nerve, ot this portion of
lingual nerve.
e- Incorrect. Extrinsic tongue muscles receive motor innervation from many nerves, but not this portion of
lingual nerve.
Item Description: Anatomy Lab Question, Dr. Beatty, 2014, Moore et al.

Question #: 114
When a patient of yours attempts to stick out his tongue, it deviates to the right side every time. You suspect
that his right hypoglossal nerve is injured. The unopposed protrusion from the left side is mediated by which
muscle?
A. Geniohyoid
B. Hyoglossus
C. Palatoglossus
D. Styloglossus
✓E. Genioglossus
Rationale:
a- Incorrect. Genioglossus is responsible for protruding the tongue b-Incorrect. Genioglossus is responsible for
protruding the tongue. c-Incorrect. Genioglossus is responsible for protruding the tongue. d-Incorrect.
Genioglossus is responsible for protruding the tongue. e-Correct. Genioglossus is responsible for protruding the
tongue.
Item Description: Anatomy Lab Question, Dr. Beatty, 2014, Moore et al.

A pituitary tumor has grown to the point where it is affecting a nearby cranial nerve. Which of the following
cranial nerves is closest to the pituitary and would be most likely affected in some way?
✓A. Optic nerve
B. Spinal accessory nerve
C. Olfactory nerve
D. Facial nerve
E. Glossopharyngeal nerve
Rationale:
a- Correct. The optic nerve, including the optic chiasm, lie superior to the pituitary gland and are often affected
by pituitary tumors.
b- Incorrect. Spinal accessory nerve is not found near the pituitary, it is restricted to the posterior cranial fossa
in the skull.
c- Incorrect. Olfactory nerve is not found near the pituitary, it is restricted to the anterior cranial fossa in the
skull.
d- Incorrect. Facial nerve is not found near the pituitary, it is restricted to the posterior cranial fossa in the
skull.
e- Incorrect. Glossopharyngeal nerve is not found near the pituitary, it is restricted to the posterior cranial
fossa in the skull.
Item Description: Anatomy Lab 16, Dr. Beatty

Question #: 86
Which of the following muscles is innervated by the external laryngeal nerve?
A. Thyrohyoid
B. Sternothyroid
C. Vocalis
D. Posterior cricoarytenoid
✓E. Cricothyroid
Rationale:
a- Incorrect. Thyrohyoid is innervated by the ansa cervicalis.
b- Incorrect. Sternothyroid is innervated by the ansa cervicalis.
c- Incorrect. Vocalis cricoarytenoid is innervated by internal laryngeal nerve.
d- Incorrect. Posterior cricoarytenoid is innervated by internal laryngeal nerve. e- Correct. Cricothyroid is
innervated by the external laryngeal nerve.
Item Description: Anatomy Lab 18, Dr. Beatty
Which of the following muscles is a longitudinal muscle of the pharynx?
A. Inferior pharyngeal constrictor
B. Superior pharyngeal constrictor
C. Middle pharyngeal constrictor
D. Longus colli
✓E. Stylopharyngeus
Rationale:
a- Incorrect. Inferior pharyngeal constrictor is a circumferential muscle of the pharynx. b- Incorrect. Superior
pharyngeal constrictor is a circumferential muscle of the pharynx. c- Incorrect. Middle pharyngeal constrictor is
a circumferential muscle of the pharynx.
d- Incorrect. Longus colli is a longitudinal muscle of the vertebral compartment. e- Correct. Stylopharyngeus is
a longitudinal muscle of the pharynx.
Item Description: Anatomy Lab 18, Dr. Beatty

A woman’s husband comments that she snores, and when you ask her to close her mouth and breathe she has
trouble. You squint your eye, and ask her to copy your action, but she cannot. You suspect that she lacks
control over the orbicularis oculi muscles. A portion of what cranial nerve is most likely dysfunctional?
A. Vagus nerve (CN X)
B. Hypoglossal nerve (CN XII)
C. Trigeminal nerve (CN V)
D. Glossopharyngeal nerve (CN IX)
✓E. Facial nerve (CN VII)
Rationale:
a- Incorrect. Vagus nerve does not provide general somatic efferents to muscles of facial expression.
b- Incorrect. Hypoglossal nerve does not provide general somatic efferents to muscles of facial expression. c-
Incorrect. Trigeminal nerve does not provide general somatic efferents to muscles of facial expression. d-
Incorrect. Glossopharyngeal nerve does not provide general somatic efferents to muscles of facial expression.
e- Correct. Facial nerve provides general somatic efferents to muscles of facial expression, such as nasalis.
Item Description: Dr. Beatty, 2014, Somatic Afferents & Efferents, Moore et al.

A 30-year-old female arrives at the hospital following an automobile accident. You discover that the palate and
upper teeth are moving separate from the rest of the face. The zygomatic arches and nose are not moving with
the palate. Where is the posterior end of the fracture most likely to be?
✓A. Pterygoid plates of sphenoid
B. Zygomatic/temporal suture
C. Nasal/frontal suture
D. Superior orbital fissure
E. Pterion
Rationale:
a- Correct. Le Fort fractures typically pass through the pterygoid plates of the sphenoid bone. This is a Le Fort
I fracture, which courses along the inferior border of the piriform aperture, passes along the floor of the maxillary
sinus just superior to the alveolar process of the maxilla, and then ends at the posterior margin of the pterygoid
plates of the sphenoid bone.
b- Incorrect. The zygomatic/temporal suture is typically fractured in a Le Fort III fracture, whereas this is a Le
Fort I fracture.
c- Incorrect. The nasal/frontal suture is typically fractured in Le Fort II and III fractures, whereas this is a Le Fort
I fracture.
d- Incorrect. A Le Fort III fracture often passes through the superior orbital fissure, whereas this is a Le Fort I
fracture.
e- Incorrect. Le Fort fractures separate all or part of the viscerocranium from the neurocranium, but the pterion
is situated entirely on the neurocranium
Item Description: Dr. Geisler, 2014, Skull and Cranial Fossae, Moore, p. 837

Question #: 57
The infraorbital nerve passes through which bone?
A. Frontal
B. Occipital
C. Nasal
✓D. Maxilla
E. Mandible
Rationale:
a- Incorrect. The frontal has the supraorbital nerve passing through it.
b- Incorrect. The occipital does not have the infraorbital nerve passing through it. c- Incorrect. The nasal does
not have the infraorbital nerve passing through it.
d- Correct. The infraorbital nerve passes through the maxilla via the infraorbital foramen.
e- Incorrect. The mandible does not have the infraorbital nerve passing through it, it has mental nerve passing
through it.
Item Description: Anatomy Lab Question- Dr. Beatty.
Session #96: Somatic Afferents and Efferents
2017
Question #: 44
After recovery from surgery involving the temporomandibular joint, a patient complains that his food tastes differently.
From various tests you conclude that the patient retains general sensation on his entire tongue, but has lost taste from
the left side of his tongue anterior to the vallate papillae. What is the most likely site of the nerve lesion he has
experienced?
A. Lingual nerve proximal to chorda tympani
B. Lingual nerve distal to chorda tympani
✓C. Chorda tympani
D. Hypoglossal nerve
E. Glossopharyngeal nerve
Rationale:
a-Incorrect. Lingual nerve proximal to chorda tympani carries general somatic afferents from the tongue, not taste.
b-Incorrect. Lingual nerve distal to chorda tympani carries general somatic afferents and taste from the tongue, so
general sensation would be lost as well if this were injured.
c-Correct. Chorda tympani carries taste sensation from the tongue, nothing else.
d-Incorrect. Hypoglossal nerve only carries general somatic efferents to intrinsic muscles of the tongue, not general
sensation or taste.
e-Incorrect. Glossopharyngeal nerve only carries taste sensations from a small region of the posterior 1/3 of the tongue,
posterior to the vallate papillae.
Item Description: Dr. Beatty, Somatic Afferents & Efferents

Question #: 45
A child in the hospital cafeteria is drinking milk and her friend causes her to suddenly laugh. The milk accidentally passes
through her nose. The timing of the elevation of the soft palate to close off the nasopharynx was poor. What cranial
nerve responsible for elevating the soft palate had this poor reaction timing?
A. Trigeminal nerve (CN V)
B. Glossopharyngeal nerve (CN IX)
✓C. Vagus nerve (CN X)
D. Facial nerve (CN VII)
E. Olfactory nerve (CN I)
Rationale:
a-Incorrect. General somatic efferent innervation of the muscle that elevates the soft palate, levator veli
palatini, is provided by vagus nerve, not trigeminal nerve.
b-Incorrect. General somatic efferent innervation of the muscle that elevates the soft palate, levator veli
palatini, is provided by vagus nerve, not glossopharyngeal nerve.
c-Correct. General somatic efferent innervation of the muscle that elevates the soft palate, levator veli
palatini, is provided by vagus nerve.
d-Incorrect. General somatic efferent innervation of the muscle that elevates the soft palate, levator veli
palatini, is provided by vagus nerve, not facial nerve.
e-Incorrect. General somatic efferent innervation of the muscle that elevates the soft palate, levator veli
palatini, is provided by vagus nerve, not olfactory nerve.
Item Description: Dr. Beatty, Somatic Afferents & Efferents

Question #: 46
A patient presents with the left side of his tongue completely atrophied, unable to change in shape. He can still retract it.
What cranial nerve is most likely affected?
A. Glossopharyngeal nerve (CN IX)
B. Facial nerve (CN VII)
C. Trigeminal nerve (CN V)
✓D. Hypoglossal nerve (CN XII)
E. Vagus nerve (CN X)
Rationale:
a-Incorrect. Intrinsic muscles of the tongue receive general somatic efferents from hypoglossal nerve, not
glossopharyngeal nerve.
b-Incorrect. Intrinsic muscles of the tongue receive general somatic efferents from hypoglossal nerve, not facial nerve.
c-Incorrect. Intrinsic muscles of the tongue receive general somatic efferents from hypoglossal nerve, not trigeminal
nerve.
d-Correct. Intrinsic muscles of the tongue receive general somatic efferents from hypoglossal nerve.
e-Incorrect. Intrinsic muscles of the tongue receive general somatic efferents from hypoglossal nerve, not vagus nerve.
Item Description: Dr. Beatty, Somatic Afferents & Efferents

Question #: 47
After a surgical procedure that removed a tumor from the parotid gland, a patient presents with difficulty moving his
eyebrows and wrinkling the skin of his forehead. Which nerve is most likely to have received a lesion during the surgical
procedure and caused this disability?
A. Ophthalmic nerve
B. Supraorbital nerve
C. Cervical branch of facial nerve
D. Auriculotemporal nerve
✓E. Temporal branch of facial nerve
Rationale:
a-Incorrect. Ophthalmic nerve provides general somatic afferents to the skin of the forehead, not general somatic
efferents to muscles of facial expression there.
b-Incorrect. Supraorbital nerve provides general somatic afferents to the skin of the forehead, not general somatic
efferents to muscles of facial expression there.
c-Incorrect. Cervical branch of facial nerve provides general somatic efferents to muscles of facial expression, but those
of the neck, not the forehead region.
d-Incorrect. Auriculotemporal nerve provides general somatic afferents to skin of the temple, not general somatic
efferents to muscles of facial expression.
e-Correct. Temporal branch of facial nerve provides general somatic efferents to muscles of facial
expression of the forehead region.
Item Description: Dr. Beatty, Somatic Afferents & Efferents

Question #: 48
A patient presents with his mandible depressed and is unable to close his mouth. His panicked friend explains that he
was just having a big yawn during a lecture, and it stuck that way. Upon inspection you note that it isn’t the result of a
dysfunctional muscle or nerve lesion, but that he seems to have dislocated his temporomandibular joint. The mandibular
condyles have protruded anterior to the articular tubercles. Which of the following muscles is most likely to be
responsible for depressing and protracting the mandibular condyles so?
A. Anterior belly of digastric
B. Medial pterygoid
C. Buccinator
D. Temporalis
✓E. Lateral pterygoid
Rationale:
a-Incorrect. The lateral pterygoid protracts the mandibular condyle and would be most likely muscle
responsible here, not the anterior belly of digastric, which retracts the mandible.
b-Incorrect. The lateral pterygoid protracts the mandibular condyle and would be most likely muscle
responsible here, not the medial pterygoid, which protracts and translates the mandible.
c-Incorrect. The lateral pterygoid protracts the mandibular condyle and would be most likely muscle
responsible here, not the buccinator, which keeps food on the tooth surfaces.
d-Incorrect. The lateral pterygoid protracts the mandibular condyle and would be most likely muscle
responsible here, not the temporalis, which elevates and retracts the mandible.
e-Correct. The lateral pterygoid protracts the mandibular condyle and would be most likely muscle responsible here.
Item Description: Dr. Beatty, Somatic Afferents & Efferents

Question #: 49
A patient presents with permanent numbness to his lower lip and lower teeth after oral surgery. Branches of which
cranial nerve were most likely injured?
A. Hypoglossal nerve (CN XII)
B. Oculomotor nerve (CN III)
C. Ophthalmic nerve (CN V1)
D. Facial nerve (CN VII)
✓E. Mandibular nerve (CN V3)
Rationale:
a-Incorrect. The hypoglossal nerve provides general somatic efferents to the intrinsic muscles of the
tongue, not general somatic afferents to the lower lip and teeth.
b-Incorrect. The oculomotor nerve provides somatic efferents to the some extraocular muscles, not general somatic
afferents to the lower lip and teeth.
c-Incorrect. The ophthalmic nerve provides general somatic afferents to a large part of the upper face and orbit, not
general somatic afferents to the lower lip and teeth.
d-Incorrect. The facial nerve provides general somatic efferents to muscles of facial expression, not
general somatic afferents to the lower lip and teeth.
e-Correct. The mandibular nerve provides general somatic afferents to the lower lip and teeth.
Item Description: Dr. Beatty, Somatic Afferents & Efferents

2016
Question #: 50
During a match, a mixed martial arts fighter receives a blow to the eye, resulting in a blowout fracture of the floor of the
orbit and lesion to the nerve lying there. This nerve is part of what cranial nerve?
A. Optic nerve (CN II)
B. Olfactory nerve (CN I)
C. Facial nerve (CN VII)
D. Glossopharyngeal nerve (CN IX)
✓E. Trigeminal nerve (CN V)
SOURCE: Dr. Beatty
Structural/Functional Thread

Question #: 51
After recovery from oral surgery, a patient complains that he keeps biting his tongue and food tastes
differently. From various tests you conclude that the patient has lost general sensation and taste from the left side of his
tongue anterior to the vallate papillae. What is the most likely site of the nerve lesion he has experienced?
✓A. Lingual nerve proximal to chorda tympani
✓B. Lingual nerve distal to chorda tympani
C. Chorda tympani
D. Hypoglossal nerve
E. Glossopharyngeal nerve
SOURCE: Dr. Beatty
Structural/Functional Thread
Rationale:
Answer choices A and B are accepted. As per Dr. Beatty, “Lingual nerve was described as proximal and distal portions
with respect to its origin, as proximal and distal terms always mean. The division between those two lengths of the
lingual nerves was defined by the connection of the chorda tympani, and this was point out multiple times in lecture. If
one suggests that the wording indicates that the answers provided were describing being about their distance from the
chorda tympani itself, neither answer would be right. Neither would distinguish on what end of lingual nerve the lesion
was found. I can accept that the wording of choices A and B could be better. Therefore, choices A & B should be
accepted.”
Question #: 52
A parent comes to you worried that his child seems to commonly choke on his food, often finding that he has swallowed
extremely large portions. You find that if you give the child small portions of food or liquid, he swallows normally
without problems. Upon inspection, you find that a tongue depressor gently touching the back of the throat
(oropharynx) results in no gag reflex. What cranial nerve is most likely dysfunctional?
A. Vagus nerve (CN X)
B. Hypoglossal nerve (CN XII)
C. Trigeminal nerve (CN V)
✓D. Glossopharyngeal nerve (CN IX)
E. Facial nerve (CN VII)
SOURCE: Dr. Beatty
Structural/Functional Thread
Rationale:
As per Dr. Beatty, “There are three clearly stated reasons why choice D is the only choice. 1) In class, I pointed out that
there are signs that vagus nerve may occasionally be involved, but I also stated that for all testing reasons they will ever
have, including my own, that they should remember this as glossopharyngeal nerve as the afferent limb of the reflex. 2)
The question asked about what cranial nerve was dysfunctional, but if vagus was dysfunctional one would not be able to
swallow as normal at all, as the motor control of muscles involved would be affected. 3) The question asks for which
cranial nerve is “most likely affected.” Even in the context of the more subtle details of reality discussed in the classroom
session about how vagus nerve may be involved in some cases (as stated in the slide), the majority of times
glossopharyngeal nerve is the afferent limb, making it the most likely nerve affected.”

Question #: 53
You and your friends are having coffee, and someone causes you to suddenly laugh. The hot coffee
accidentally passes through your nose, causing it to burn. What structure moved that allowed the coffee to enter your
nose?
✓A. Soft palate
B. Larynx
C. Hard palate
D. Mandible
E. Pharyngotympanic tube
SOURCE: Dr. Beatty
Structural/Functional Thread

Question #: 54
An airline passenger’s newborn is screaming in pain during takeoff and landing, and the father attempts to help ease the
baby’s pain by having her nurse from her bottle. It works, and this is primarily because the baby is opening her
________________ using her _______________.
✓A. Pharyngotympanic tube; tensor veli palatini
B. Pharynx; superior pharyngeal constrictor
C. Laryngopharynx; epiglottis
D. External auditory meatus; levator veli palatini
E. Mouth; orbicularis oris
SOURCE: Dr. Beatty
Structural/Functional Thread

Question #: 55
A private aircraft pilot has suffered a plane crash, which resulted in his safety harness damaging both his
right and left laryngeal nerves. He is now only able to make a few squeaking sounds when he tries to
speak. What is the action of the muscle whose innervation is left intact?
A. Abducting the vocal cords
B. Adducting the arytenoid cartilages
C. Adducting the vocal ligament
D. Relaxing the vocal ligament
✓E. Stretching or tensing the vocal ligament
SOURCE: Dr. Beatty
Structural/Functional Thread
Rationale:
As per Dr. Beatty, “The question clearly states that both left and right recurrent laryngeal nerves are injured and
“Arytenoid cartilage help the vocal cords in being tense. The setup of the question requires one to realize that all of the
muscles of the larynx have lost innervation except for the one not getting innervated by the recurrent laryngeal nerves –
crocothyroid. Cricothyroid’s function is in rocking the thyroid cartilage anteriorly to cause tension in the vocal ligament,
not necessarily the adduction of the arytenoid cartilages (the vocal cords can be pulled in tension without those being
adducted). Therefore, choice B is not a proper answer.”

2015
5. Which of the following cranial nerves provides general somatic efferent innervation to a structure in the orbit?

A. ✓Abducens nerve (CN VI)


B. Hypoglossal nerve (CN XII)
C. Spinal accessory nerve (CN XI)
D. Trigeminal nerve (CN V)
E. Vestibulocochlear nerve (CN VIII)

SOURCE: Dr. Beatty’s lecture on 11/15/2011. Assigned reading: Moore et. al., pages 1053-1082; Sadler, pages 265-291
Structural/Functional Thread

5. A boxer’s jaw is dislocated during a fight, and significant lesions to muscles of mastication are apparent. Though
he has trouble protracting and elevating his mandible, he has no trouble retracting it. Which of the following muscles is
most likely to be fully functional and allowing this movement?

A. Anterior belly of digastric


B. Masseter
C. Medial pterygoid
D. Orbicularis oris
E. ✓Posterior portion of temporalis

SOURCE: Dr. Beatty’s lecture on 11/17/2011. Assigned reading: Moore et. al., pages 849-853, 903-905, 914-965.
Structural/Functional Thread
Rationale: The question actually asks which is MOST LIKELY to be fully functional of all the muscles listed. None of the
others are capable of retraction, and the patient is only listed as having trouble elevating the mandible, not that it is
completely unable to do so. The choice of the posterior portion of the temporalis is specific enough. The posterior
portion of temporalis is not significantly functional in jaw elevation.

5. A child presents with a crayon inserted far into his nose. After its removal and recovery, you test his
nasal cavity for any functional losses and find that the inferior and posterior portions of his nasal septum have lost
general somatic afferent sensation. This is most likely due to damage to branches of the:

A. Facial nerve (CN VII)


B. Mandibular nerve (CN V3)
C. ✓Maxillary nerve (CN V2)
D. Olfactory nerve (CN II)
E. Ophthalmic nerve (CN V1)

SOURCE: Dr. Beatty’s lecture on 11/17/2011. Assigned reading: Moore et. al., pages 849-853, 903-905, 914-965.
Structural/Functional Thread
Rationale:The question clearly states that the portion of the septum injured is the posterior and inferior portion, not the
portion innervated by the ethmoidal nerves. This was fully explained in the lecture on trigeminal nerve. Olfactory nerve
or (CN II), neither is composed of general somatic afferents. Olfactory is a special afferent nerve, not a general somatic
afferent.

6. A patient presents soon after recovering from a car accident in which his face was severely lacerated by broken
glass. His forehead was lacerated badly, and after recovery the patient has some asymmetry in some of his facial
expressions. Injury to which of the following nerves is most likely to have caused this?

A. Buccal branch of facial nerve (CN VII)


B. Cervical branch of facial nerve (CN VII)
C. Mandibular branch of facial nerve (CN VII)
D. ✓Temporal branch of facial nerve (CN VII)
E. Zygomatic branch of facial nerve (CN VII)

SOURCE: Dr. Beatty’s lecture on 11/22/2011. Assigned reading: Moore et. al., pages 966-980. Structural/Functional
Thread
Rationale: As per Dr. Beatty,"All of the branches of facial nerve are listed in the question, and All can affect facial
expressions. The question clearly states that a laceration occurred in one region of the face. The laceration would be in
the branch of facial nerve in THAT location.
As I repeatedly corrected in lecture and in lab – the orbicularis oculi is innervated by both branches, zygomatic and
temporal, depending on whether the portion being referred to is above or below the eyelid (approximately). The
zygomatic branch of facial nerve does NOT extend to the eyebrow, so it is NOT the one lacerated."

2014

5. Which of the following muscles is capable of assisting in depressing (opening) the mandible?

A. Medial pterygoid
B. ✓Lateral pterygoid
C. Orbicularis oris
D. Masseter
E. Posterior portion of temporalis

SOURCE: Dr. Beatty’s lecture on 11/18/2010. Assigned reading: Drake 2010 pp. 858-898, 920-947, 1013-1034, 1055-1060
Structural/Functional Thread

7. A Major League Soccer player gets kicked in the side of the face, lacerating his cheek near the ear
during a match. The injury is deep, but does not cause any bones to be broken. It is later is noticed that the player has
some asymmetry in some of his facial expressions. Injury to which of the following nerves is most likely to have caused
this?

A. Maxillary nerve (CN V2)


B. ✓Mandibular branch of facial nerve (CN VII)
C. Mandibular nerve (CN V3)
D. Glossopharyngeal nerve (CN IX)
E. Ophthalmic nerve (CN V1)

SOURCE: Dr. Beatty’s lecture on 11/23/2010. Assigned reading: Drake 2010 pp. 858-878, 902-919
Structural/Functional Thread

5. A soldier in Afghanistan is injured by a piece of shrapnel that enters the posterior triangle of the neck.
Examination shows normal functions in the arm and breathing, but she shows weakness in shrugging/elevating the
shoulder and bending her neck. Which structure was most likely injured in this scenario?

A. Internal carotid artery


B. Anterior belly of digastric
C. ✓Spinal accessory nerve (CN XI)
D. Thyroid gland
E. Hypoglossal nerve (CN XII)

SOURCE: Dr. Beatty’s lecture on 11/30/2010. Assigned reading: Drake 2010 pp. 947-984.
Structural/Functional Thread

5. A patient presents with stab wounds to the neck that, though healed and no longer critical, appear to have
resulted in some dysfunctions that you suspect relate to cranial nerve lesions. He is unable to move his tongue properly,
and has great difficulty swallowing. The most likely explanation for the loss of motor function of the tongue is a lesion to
the _. The most likely explanation for the trouble swallowing is a lesion to the .

A. Vagus nerve; glossopharyngeal nerve


B. Glossopharyngeal nerve; vagus nerve
C. ✓Hypoglossal nerve; glossopharyngeal nerve
D. ✓Hypoglossal nerve; vagus nerve
E. Glossopharyngeal nerve; hypoglossal nerve

SOURCE: Dr. Beatty’s lecture on 11/30/2010. Assigned reading: Drake 2010 pp. 985-997, 1034-1054.
Structural/Functional Thread
Rationale: Answer choices C and D are being accepted

Session #97: The Orbit and Extraocular Muscles


2017
Question #: 25
You test the extraocular function of a 32-year-old female by having her follow an “H” pattern you make with your finger.
Generally she is able to follow your finger, except that she is not able to elevate her right eye after abduction. Which of
the following is the most likely affected structure?
A. Right superior oblique muscle
✓B. Right superior rectus muscle
C. Right inferior rectus muscle
D. Right inferior oblique muscle
E. Right medial rectus muscle
Rationale:
a-Incorrect. The superior oblique is the primary depressor of the adducted eye, but the patient was able to do this.
b-Correct. The superior rectus is the primary elevator of an abducted eye, thus failure to elevate the abducted eye
indicates that this muscle is affected.
c-Incorrect. The inferior rectus is the primary depressor of the abducted eye, but the patient was able to do this.
d-Incorrect. The inferior oblique is the primary elevator of the adducted eye, but the patient was able to do this.
e-Incorrect. The medial rectus is the primary adductor of the eye, but the patient was able to do this.
Item Description: Dr. Geisler, The Orbit and Extraocular Muscles, Clinically Oriented Anatomy 7e, pp.
898-899

Question #: 26
A 60-year-old male presents with ptosis of the left eye. Two years ago he had a malignant brain tumor removed. Upon
opening the left eyelid you notice that the pupil is dilated and stationary in the inferolateral corner of the orbit. What is
the most likely affected structure?
A. Left trochlear nerve
B. Left abducens nerve
C. Only the superior division of the left oculomotor nerve
✓D. Left oculomotor nerve
E. Only the inferior division of the left oculomotor nerve
Rationale:
a-Incorrect. The trochlear nerve innervates the superior oblique, which depresses the eye. The eye is still able to make
this movement.
b-Incorrect. The abducens nerve innervates the lateral rectus, the primary abductor of the eye. The eye is abducted, thus
this muscle and its innervation are still functional.
c-Incorrect. If only the superior division is affected then the inferior division should be intact. Thus muscles innervated
by the inferior division (inferior rectus, medial rectus, and inferior oblique) should be functioning.
The medial rectus would adduct the eye and the inferior oblique would elevate it, but the eye is abducted and
depressed.
d-Correct. These are all classic symptoms of a complete palsy of the oculomotor nerve. This nerve innervates the
superior, inferior, and medial rectus; it innervates the inferior oblique; it innervates the levator palpebrae superioris; and
carries parasympathetic fibers that innervate the constrictor pupillae. The only functioning extraocular muscles are the
superior oblique (innervated by trochlear nerve) and lateral rectus (innervated by the abducens nerve). The former
depresses the eye and the latter abducts the eye, and both movements are unopposed. Thus the pupil shifts to the
inferolateral corner of the orbit. Gravity overcomes the nonfunctioning levator palpebrae superioris, causing ptosis, and
the sphincter pupillae cannot act as an antagonist to the dilator pupillae. The dilator pupillae remains active because
sympathetic fibers are conveyed on a different nerve, specifically a sympathetic root of the otic ganglion that branches
off of a plexus on the internal carotid artery.
e-Incorrect. If only the inferior division was affected, then the superior division should be intact. Thus muscles
innervated by the superior division (superior rectus, levator palpebrae superioris) should be functioning. The superior
rectus would elevate the eye and the levator palpebrae superioris would elevate the superior tarsus, but there is ptosis
and the eye is depressed.
Item Description: Dr. Geisler, The Orbit and Extraocular Muscles, Clinically Oriented Anatomy 7e, pp.
898-899, 913

2016
Question #: 37
When tilting the head, a little bit of torsion of the eyeball is needed. Which of the following muscles contributes to
intorsion of the eyeball?
A. Inferior oblique
B. Inferior rectus
C. Lateral rectus
D. Medial rectus
✓E. Superior rectus
SOURCE: Dr. Geisler lecture on 11/30/2012; Moore, Dalley, and Agur pp. 898-903.
Structural/Functional Thread

Question #: 38
A patient presents with an inability to abduct her right eye. Which of the following is the most likely cause?
A. A tear in the left medial rectus muscle
B. Increased intracranial pressure on the optic nerve
✓C. Lesion on the right cranial nerve VI
D. Lesion on the right trochlear nerve
E. Lesion on the right oculomotor nerve
SOURCE: Dr. Geisler lecture on 11/30/2012; Moore, Dalley, and Agur pp. 898-900.
Structural/Functional Thread
2014
5. You test the extraocular function of a patient by having them follow an “H” pattern you make with a small light.
You notice that the patient is not able to place his pupil in the inferomedial corner of the eye. Otherwise all seems
normal. Which of the following is most likely the problem?

A. Damage to the medial rectus muscle


B. Damage to the superior rectus muscle
C. Lesion on the oculomotor nerve within the subarachnoid space
D. Lesion on the superior division of the oculomotor nerve
E. ✓Lesion on the trochlear nerve

SOURCE: Dr. Geisler’s lecture on 11/23/2010. Assigned reading: Drake pp.890-891. Structural/Functional Thread

6. Which of the following structures passes though the superior orbital fissure but external to the common
tendinous ring?

A. Abducent nerve
B. Inferior division of oculomotor nerve
C. ✓Lacrimal nerve
D. Ophthalmic artery
E. Nasociliary nerve
SOURCE: Dr. Geisler’s lecture on 11/23/2010. Assigned reading: Drake pg. 890.
Structural/Functional Thread.
Session #104 and 105: Special Afferents and Autonomics in the Head and Neck
2017
Question #: 1
A contestant on an obstacle course game show (like Wipeout) receives a severe blow to the head, causing
multiple fractures to his face around the orbits. One of these was a blowout fracture that led to lesions to
multiple branches of oculomotor nerve (CN III) within the orbit. The patient is exhibiting difficulty constricting his
pupil in response to having a light shined in his eye, yet indicates that he can see with it just fine. What lesion
is this most likely the result of?
✓A. Parasympathetics to constrictor pupillae
B. Sympathetics to dilator pupillae
C. Parasympathetics to the ciliary body
D. Sympathetics to Müller’s muscle
E. Parasympathetics to the lacrimal gland
Rationale:
a-correct. The difficulty constricting the pupil indicates that constrictor pupillae is affected, which is controlled
by parasympathetics.
b-Incorrect. Dilator pupillae does not constrict the pupil.
c-Incorrect. The ciliary body does not constrict the pupil.
d-Incorrect. Müller’s muscle does not constrict the pupil.
e-Incorrect. The lacrimal gland does not constrict the pupil.
Item Description: Dr. Beatty, Autonomics

Question #: 50
A cyclist that was the victim of a hit and run is brought into the emergency department unconscious.
Superficial injuries to his head make you concerned that he has experienced internal trauma, and you need
to rapidly identify whether he is experiencing elevated intracranial pressure. You inspect his eyes with an
ophthalmoscope and note signs of papilledema (swelling of the optic disc). Which of the following is most
likely directly involved in this symptom?
✓A. Optic nerve
B. Trochlear nerve
C. Abducens nerve
D. Ophthalmic nerve
E. Oculomotor nerve
Rationale:
a-Correct. Increased intracranial pressure is most likely causing pressure on the cranial nerve directly
connected to the optic disc, the optic nerve.
b-Incorrect. Increased intracranial pressure is most likely causing pressure on the cranial nerve directly
connected to the optic disc, the optic nerve. Trochlear nerve is shielded by the cavernous sinus and does
not have direct contact with the optic disc.
c-Incorrect. Increased intracranial pressure is most likely causing pressure on the cranial nerve directly
connected to the optic disc, the optic nerve. Abducens nerve is shielded by the cavernous sinus and does
not have direct contact with the optic disc.
d-Incorrect. Increased intracranial pressure is most likely causing pressure on the cranial nerve directly
connected to the optic disc, the optic nerve. Ophthalmic nerve is shielded by the cavernous sinus and does
not have direct contact with the optic disc.
e-Incorrect. Increased intracranial pressure is most likely causing pressure on the cranial nerve directly
connected to the optic disc, the optic nerve. Oculomotor nerve is shielded by the cavernous sinus and does
not have direct contact with the optic disc.
Item Description: Dr. Beatty, Special afferents

Question #: 51
A college student at a concert enthusiastically dancing and shaking his head suddenly feels extremely
dizzy and drops to the floor. His friends joke and guess about whether he has ingested too much alcohol or
gotten injured by another person. After a moment the friend explained it was neither, and simply explains that
he was shaking his head and it happened suddenly. What organ’s dysfunction is most likely
responsible for this sensation?
✓A. Vestibule
B. Malleus
C. Cochlea
D. Pharyngotympanic tube
E. Stapes
Rationale:
a-Correct. The vestibule contains the organs balance, whose dysfunctions typically manifests as vertigo.
b-Incorrect. The vestibule contains the organs balance, whose dysfunctions typically manifests as vertigo.
A dysfunction of the malleus would result in hearing deficits.
c-Incorrect. The vestibule contains the organs balance, whose dysfunctions typically manifests as vertigo.
A dysfunction of the cochlea would result in hearing deficits.
d-Incorrect. The vestibule contains the organs balance, whose dysfunctions typically manifests as vertigo.
A dysfunction of the pharyngotympanic tube would not result in vertigo.
e-Incorrect. The vestibule contains the organs balance, whose dysfunctions typically manifests as vertigo.
A dysfunction of the stapes would result in hearing deficits.
Item Description: Dr. Beatty, Special afferents

Question #: 52
A librarian wakes up one morning to find that he cannot smell his coffee, and upon examination from his doctor
finds that he has a brain tumor that has grown overnight enough to impinge on a cranial nerve as it passes
through the skull. Where is the tumor located?
A. Internal acoustic meatus
B. Superior orbital fissure
✓C. Cribriform plate
D. Inferior orbital fissure
E. Jugular foramen
Rationale:
a-Incorrect. The loss of olfactory sense indicates that the olfactory nerve is dysfunctional. It passes through the
cribriform plate of the ethmoid, not the internal acoustic meatus.
b-Incorrect. The loss of olfactory sense indicates that the olfactory nerve is dysfunctional. It passes through the
cribriform plate of the ethmoid, not the superior orbital fissure.
c-Correct. The loss of olfactory sense indicates that the olfactory nerve is dysfunctional. It passes through the
cribriform plate of the ethmoid.
d-Incorrect. The loss of olfactory sense indicates that the olfactory nerve is dysfunctional. It passes through the
cribriform plate of the ethmoid, not the inferior orbital fissure.
e-Incorrect. The loss of olfactory sense indicates that the olfactory nerve is dysfunctional. It passes through the
cribriform plate of the ethmoid, not the jugular foramen.
Item Description: Dr. Beatty, Special afferents

Question #: 53
A college student cycling to class gets hit by a car and receives lacerations to his face around his cheeks.
He is grateful to have avoided injury to branches of facial nerve, but needed stitches to deal with arterial
lesions. After recovery, he presents with a complaint that he seems to blush on his cheeks unevenly. A lesion
to which of the following nerves is most likely to be causing these symptoms?
A. Facial nerve
✓B. External carotid plexus
C. Ophthalmic nerve
D. Internal carotid plexus
E. Nerve of the pterygoid canal
Rationale:
a-Incorrect. Facial nerve provides efferents to muscles of facial expression, not sympathetic innervation to
skin.
b-Correct. The external carotid plexus includes the sympathetic nerves controlling blushing to the face below
the eye, including the cheeks.
c-Incorrect. Ophthalmic nerve provides general somatic afferents to the skin above the eye, not sympathetics
to the skin of the cheek.
d-Incorrect. The internal carotid plexus includes the sympathetic nerves controlling blushing to the face above
the eye, excluding the cheeks.
e-Incorrect. The nerve of the pterygoid canal provides autonomics to structures in the eyeball, but not
sympathetics to the skin of the cheeks.
Item Description: Dr. Beatty, Autonomics

Question #: 56
A patient presents without a pupillary light reflex, being unable to constrict his pupil. He also complains
about having trouble seeing things closely. You are concerned that he has disrupted nerves going to his
eye. What is most likely dysfunctional?
✓A. Parasympathetics to the ciliary muscles
B. Sympathetics to dilator pupillae
C. Somatic efferents to superior oblique
D. Sympathetics to Müller’s muscle
E. Somatic afferents to the cornea
Title: SOURCE: Dr. Beatty
Structural/Functional Thread

Question #: 57
A 32-year-old woman has been repeatedly experiencing vertigo, suddenly becoming dizzy and unable to
determine up or down when her eyes are closed. What organ’s dysfunction is most likely responsible for
this sensation?
✓A. Vestibule
B. Malleus
C. Cochlea
D. Pharyngotympanic tube
E. Stapes
Title: SOURCE: Dr. Beatty
Structural/Functional Thread

Question #: 58
A patient recovering from a virally induced Bell’s palsy presents with an unusual circumstance in which
he sheds tears when he is eating, known as “crocodile tears syndrome”. The improperly healed nerve
connections he has experienced have resulted in parasympathetics that normally go to ____________ to
be redirected to _______________.
A. Lacrimal glands; salivary glands
B. Sweat glands; salivary glands
✓C. Salivary glands; lacrimal glands
D. Lacrimal glands; sweat glands
E. Sweat glands; lacrimal glands
SOURCE: Dr. Beatty
Structural/Functional Thread

Question #: 59
During surgery of the pituitary in which you pass through the sphenoid sinus to get to the lateral sides of
the gland, your proximity to the internal carotid artery results in an error that causes damage to the fascia
and nerves surrounding the internal carotid. After recovery, you test to see if the sympathetics that may
have been damaged are intact by testing for heat-induced swelling of skin on the ___________ .
A. Back of the head
✓B. Forehead
C. Cheek
D. Chin
E. Neck
SOURCE: Dr. Beatty
Structural/Functional Thread

Question #: 60
A 17-year-old male was involved in a gang related fight in which he was beaten with a baseball bat. He
suffered multiple fractures, one of which was to the temporal bone leading to a lesion to facial nerve (CN
VII) just proximal to where the chorda tympani branches. The patient is exhibiting facial paralysis on the
side of the fracture and a sensitivity to certain sounds. What most relevant autonomic function is spared
pertaining to this cranial nerve lesion?
✓A. Lacrimal secretion
B. Parotid secretion
C. Sublingual secretion
D. Submandibular secretion
E. Sweat secretion
SOURCE: Dr. Beatty
Structural/Functional Thread
Rationale:
As per Dr. Beatty, “The question is asking what function is spared pertaining to this cranial nerve lesion,
NOT other cranial nerves. It was clearly explained in the classroom session and VPG how one could
think through the functions retained or lost due to lesions of the facial nerve at different points on its
pathway. Sweat secretion and the parotid are not controlled by facial nerve, and submandibular and
sublingual are controlled by parts of facial nerve that would be affected by the lesion, making lacrimal
secretion the only logical choice. The portions of lingual nerve that comes from trigeminal would still be
travelling to the tongue, but is uninvolved in the autonomic control of any of the choices given.”

Question #: 61
Which of the following groups of cranial nerves is composed only of special afferent function?
✓A. Optic nerve, vestibulocochlear nerve, and olfactory nerve
B. Oculomotor nerve, olfactory nerve, and optic nerve
C. Optic nerve, trochlear nerve, and olfactory nerve
D. Hypoglossal nerve, olfactory nerve, and abducens nerve
E. Optic nerve, spinal accessory nerve, vestibulocochlear nerve
SOURCE: Dr. Beatty
Structural/Functional Thread

OMM
What motion is the most prominent in the atlanto-axial joint?

Side bending
Rotation
Flexion
Extension
Translation

Rationale:

Incorrect. The most prominent motion in the AA is rotation not side bending
Correct. The most prominent motion in the AA is rotation
Incorrect. The most prominent motion in the AA is rotation not flexion, that motion is more prominent in the OA.
incorrect The most prominent motion in the AA is rotation not extension which is more prominent in the OA.
Incorrect. The most prominent motion in the AA is rotation not translation which is what we do in the
Osteopathic exam when diagnosing the cervical spine.

Item Description: Dr. Caron, 11.20 2013, Osteopathic Considerations of Head & Neck DiGiovanna 3rd ed Ch.
24
A 50-year-old male presents to your office with new onset, progressive left temporal headaches. He
describes them as 8/10 intermittent, dull, worse in the morning and with changing positions. Over the
counter Aleve 500 mg helps a little. He denies any aura, history of trauma or specific neurologic
complaints. On the funduscopic exam you note bilateral papilledema. What is on the top of your
differential diagnosis?

Meningitis
Carotid artery dissection
Migraine headache
Intracranial space occupying lesion
Cluster phenomenon

Rationale:

Incorrect. Meningitis would have fever & neck stiffness


Incorrect. Carotid artery dissection would present w/ acute onset with change in pupil size, drooping eyelid and
neck pain.
Incorrect. Usually has an aura and is not aggravated by position change, no papilledema
Correct. Intracranial lesion would cause papilledema with symptom changes with differing positions
Incorrect. Shorter duration, no papilledema, no change w/ position

Item Description: Dr. Caron, 11.20 2013, Osteopathic Considerations of Head & Neck, Reference Karim
Abdollahi, Virchel E. Wood

What motion is the most prominent in the atlanto-axial joint?

Side bending
Rotation
Flexion
Extension
Translation

SOURCE: Caron, 2012, Osteopathic Considerations of Head & Neck: Integument System 2012, cervical
biomechanicsStructural/Functional Thread

When the head goes into flexion the occipital condyles go in what direction?

Laterally
Rotation left
Rotation right
Anteriorly
Posteriorly

SOURCE: Caron, 2012, Osteopathic Considerations of Head & Neck: Integument System 2012, cervical
biomechanics Structural/Functional Thread

An administrative assistant presents to your office with bilateral shoulder and arm pain which is worse
when she is sitting at her desk. What three tests might you perform to help in your differential
diagnosis of thoracic outlet syndrome?

Spurling, FABRE and Tinel


Tinel, Roos and Thomson
Tomas, Spring and Mc’Murray
Appley, Neer and Wright
Wright, Roos and Adson
SOURCE: Caron, 2012, Osteopathic Considerations of Head & Neck: Integument System 2012, Thoracic
Outlet Syndrome Structural/Functional Thread

A 50-year-old male presents to your office with new onset, progressive left temporal headaches. He
describes them as 8/10 intermittent, dull, worse in the morning and with changing positions. Over the
counter Aleve 500 mg helps a little. He denies any aura, history of trauma or specific neurologic
complaints. On the funduscopic exam you note bilateral papilledema. What is the most likely
diagnosis?

Meningitis
Carotid artery dissection
Migraine headache
Intracranial space occupying lesion
Cluster phenomenon

Rationale: As per Dr. Caron, “Although it is possible to get papilledema from meningitis, none of the other
symptoms point towards meningitis. The more classic meningitis signs are nuchal rigidity & fever. The
headaches from meningitis also tend not to be localized to the temporal area. So the most likely answer is a
space occupying lesion. The hints of new onset, age and change of position should have pointed the student in
the right direction.”

SOURCE: Caron, 2012, Osteopathic Considerations of Head & Neck: Integument System 2012, headache red
flags Structural/Functional Thread

A 41-year-old male seeks care for left ankle pain after a long hike. During your physical examination, you note
that the left foot is freer in plantar flexion and internal rotation compared with the right foot. Which of the following
would be a correct diagnosis of the left lower limb?
✓A. Posterior fibular head
B. Anterior fibular head
C. Pronation of the foot
D. Eversion of the foot
E. External rotation of the tibia
Rationale:
a- Correct. The findings would be consistent with a supinated foot, which is associated with a posterior fibular
head.
b- Incorrect. The findings would be consistent with a supinated foot, which is associated with a posterior
fibular head, not an anterior fibular head, since fibular head motion occurs in the opposite direction.
c- Incorrect. Plantar flexion and internal rotation of the foot would be consistent with foot supination, not foot
pronation.
d- Incorrect. Plantar flexion and internal rotation of the foot would be associated also with foot inversion.
These three motions together constitute foot supination.
e- Incorrect. External rotation of the tibia is not an associated finding of the other described findings in the
case. There is reciprocal relationship between the foot/ankle and the fibular head. In this case, plantarflexion of
the foot is associated with a posterior fibular head.
Item Description: Dr. Li, Biomechanics, Evaluation, and Osteopathic Perspectives of the Knee, Lecture
powerpoint and presentation

Question #: 59
A 65-year-old female patient with peripheral diabetic neuropathy presents to your office with a chief complaint
of lower extremity weakness bilaterally for the past two months. The patient cannot stand and walk on her toes.
What nerve root is most likely affected?
D. L2
E. L3
F. L4
G. L5
✓E. S1
Rationale:
a- L2 – Incorrect. Likely affects Thigh adduction via innervation of the adductor longus and brevis. b- L3 –
Incorrect. Overlaps with Knee extension and thigh adduction
c- L4 – Incorrect. Affects ankle dorsiflexion innervation of the tibialis anterior. The patient would have her feet
in plantarflexion during walking / standing on her toes.
d- L5 – Incorrect. Effects largely extension of the great toes via innervation of the extensor hallusis longus. e-
S1 – Correct. Affects plantar flexion via innervation of the Gastrocnemius and soleus.
Item Description: Dr. Cheriyan, 2014, Osteopathic considerations of the Foot and Ankle; lecture PPT slide.
Session # 115: Osteopathic Considerations of the Head & Neck
2017
Question #: 29
What motion is the most prominent in the atlanto-axial joint?
A. Side bending
✓B. Rotation
C. Flexion
D. Extension
E. Translation
Rationale:
a- Incorrect. The most prominent motion in the AA is rotation not side bending
b- Correct. The most prominent motion in the AA is rotation
c- Incorrect. The most prominent motion in the AA is rotation not flexion, that motion is more prominent in the
OA.
d- incorrect The most prominent motion in the AA is rotation not extension which is more prominent in the OA.
e- Incorrect. The most prominent motion in the AA is rotation not translation which is what we do in the
Osteopathic exam when diagnosing the cervical spine.
Item Description: Dr. Caron, 11.202013, Osteopathic Considerations of Head & Neck DiGiovanna 3rd
ed Ch. 24

Question #: 30
A 50-year-old male presents to your office with new onset, progressive left temporal headaches. He describes
them as 8/10 intermittent, dull, worse in the morning and with changing positions. Over the counter Aleve 500
mg helps a little. He denies any aura, history of trauma or specific neurologic complaints.
On the funduscopic exam you note bilateral papilledema. What is on the top of your differential diagnosis?
A. Meningitis
B. Carotid artery dissection
C. Migraine headache
✓D. Intracranial space occupying lesion
E. Cluster phenomenon
Rationale:
a- Incorrect. Meningitis would have fever & neck stiffness
b- Incorrect. Carotid artery dissection would present w/ acute onset with change in pupil size, drooping eyelid
and neck pain.
c- Incorrect. Usually has an aura and is not aggravated by position change, no papilledema
d- Correct. Intracranial lesion would cause papilledema with symptom changes with differing positions
e- Incorrect. Shorter duration, no papilledema, no change w/ position
Item Description: Dr. Caron, 11.202013, Osteopathic Considerations of Head & Neck, Reference Karim
Abdollahi, Virchel E. Wood

2016
Question #: 43
What motion is the most prominent in the atlanto-axial joint?
A. Side bending
✓B. Rotation
C. Flexion
D. Extension
E. Translation
SOURCE: Caron, 2012, Osteopathic Considerations of Head & Neck: Integument System 2012, cervical
biomechanics
Structural/Functional Thread

Question #: 44
When the head goes into flexion the occipital condyles go in what direction?
A. Laterally
B. Rotation left
C. Rotation right
D. Anteriorly
✓E. Posteriorly
SOURCE: Caron, 2012, Osteopathic Considerations of Head & Neck: Integument System 2012, cervical
biomechanics
Structural/Functional Thread

Question #: 45
An administrative assistant presents to your office with bilateral shoulder and arm pain which is worse when
she is sitting at her desk. What three tests might you perform to help in your differential diagnosis of thoracic
outlet syndrome?
A. Spurling, FABRE and Tinel
B. Tinel, Roos and Thomson
C. Tomas, Spring and Mc’Murray
D. Appley, Neer and Wright
✓E. Wright, Roos and Adson
SOURCE: Caron, 2012, Osteopathic Considerations of Head & Neck: Integument System 2012, Thoracic
Outlet Syndrome
Structural/Functional Thread

Question #: 46
A 50-year-old male presents to your office with new onset, progressive left temporal headaches. He describes
them as 8/10 intermittent, dull, worse in the morning and with changing positions. Over the counter Aleve 500
mg helps a little. He denies any aura, history of trauma or specific neurologic complaints. On the funduscopic
exam you note bilateral papilledema. What is the most likely diagnosis?
A. Meningitis
B. Carotid artery dissection
C. Migraine headache
✓D. Intracranial space occupying lesion
E. Cluster phenomenon
SOURCE: Caron, 2012, Osteopathic Considerations of Head & Neck: Integument System 2012, headache red
flags
Structural/Functional Thread
Rationale:
As per Dr. Caron, “Although it is possible to get papilledema from meningitis, none of the other symptoms point
towards meningitis. The more classic meningitis signs are nuchal rigidity & fever. The headaches from
meningitis also tend not to be localized to the temporal area. So the most likely answer is a space occupying
lesion. The hints of new onset, age and change of position should have pointed the student in the right
direction.”

2015
99. A patient presents to your office with a chief complaint of right sided pain of his arm, scapula and neck
following a motor vehicle accident two weeks ago. On physical exam, he has no edema or color changes on
his affected side. Your rationale for diagnosing him with a non-specific Thoracic Outlet Syndrome is because
his symptoms are consistent with:
A. Brachial plexus involvement
B. Subclavian artery involvement
C. Subclavian vein involvement
D. Subcoracoid area involvement
E. ✓Traumatic event involvement
Source: 1/3/12 lecture – Dr. Flaum
Structural/Functional Thread
Rationale: Choice A: Brachial Plexus involvement (Neurogenic) - patients present with painless atrophy of the
intrinsic muscles of the hand, and athletes who perform overhead arm activities may report difficulty grasping a
racket or ball as a result of intrinsic muscle weakness. The described patient complained of: right sided pain of
his arm, scapula and neck following a motor vehicle accident two weeks ago. Therefore, only accept answer
choice E: traumatic event involvement.

PATHOLOGY
Question #: 72
The accompanying photo represents a microscopic section of an unidentified organ with an area of necrosis
which shows preserved cellular architecture with loss of nuclei, resulting from loss of blood supply. What does
this most likely represent? (Please see attachment)
✓A. A kidney infarct
B. Acute pancreatitis
C. Pulmonary tuberculosis
D. Brain abscess
E. Vasculitis
Rationale:
a- Correct. An area of necrosis which results from ischemia (loss of blood supply) that microscopically shows preservation of the cellular
architecture with loss or disappearance of nuclei is an infarct. This pattern is coagulative necrosis and this photo represents a kidney infarct
with coagulative necrosis. The description of the pattern of necrosis in this question provides the answer without the need to identify the
specific organ.
b- Incorrect. An area of necrosis which results from ischemia (loss of blood supply) that microscopically shows preservation of the
cellular architecture with loss or disappearance of nucleus is an infarct. This pattern is coagulative necrosis and this photo represents a
kidney infarct with coagulative necrosis. The description of the pattern of necrosis in this question provides the answer without the need to
identify the specific organ. The necrosis of acute pancreatitis is most likely to be fatty necrosis because of the release of enzymes from
the pancreas and the fatty destruction which follows.
c- Incorrect. An area of necrosis which results from ischemia (loss of blood supply) that microscopically shows preservation of the
cellular architecture with loss or disappearance of nucleus is an infarct. This pattern is coagulative necrosis and this photo represents a
kidney infarct with coagulative necrosis. The description of the pattern of necrosis in this question provides the answer without the need
to identify the specific organ. Pulmonary tuberculosis is most likely to be represented by necrotizing or caseating granulomas which are
examples of caseous necrosis.
d- Incorrect. An area of necrosis which results from ischemia (loss of blood supply) that microscopically shows preservation of the
cellular architecture with loss or disappearance of nucleus is an infarct. This pattern is coagulative necrosis and this photo represents a
kidney infarct with coagulative necrosis. The description of the pattern of necrosis in this question provides the answer without the need to
identify the specific organ. Brain tissue and abscess both would undergo liquefactive necrosis.
e- Incorrect. An area of necrosis which results from ischemia (loss of blood supply) that microscopically shows preservation of the
cellular architecture with loss or disappearance of nucleus is an infarct. This pattern is coagulative necrosis and this photo represents a
kidney infarct with coagulative necrosis. The description of the pattern of necrosis in this question provides the answer without the need to
identify the specific organ. Vasculitis, or inflammation of blood vessels, is most likely immune mediated and generally shows fibrinoid
necrosis.
Item Description: Dr. Plummer, 2014, Introduction to Path-Cellular Adaptation and Injury, Robbins Ch. 2
Question #: 73
A pathologist is examining tissue under the microscope and sees an area where the cells are necrotic and the
nuclei of the cells are pyknotic and have undergone karyorrhexis and karyolysis. The enzymes responsible for
these changes were activated by a molecule which enters the cell because of:
A. Loss of intracellular Na+
B. Increased pH
C. Increased ATP production
✓D. Failure of the Ca+2 pump
E. Decrease in anaerobic glycolysis
Rationale:
a- Incorrect. Pyknosis, karyorrhexis, and karyolysis are irreversible nuclear changes caused by
endonucleases which are activated by Ca+2 which enters an irreversibly damaged cell because of failure of
the Ca+2 pump. Therefore, an influx of calcium into the cell occurs, activating many enzymes including
endonucleases which cause pyknosis, karyorrhexis, and karyolysis. Because of injury in the first place, there is
decreased oxidative phosphorylation and decreased ATP production and the Na+/K+ pump is reduced which
results in Na+ entering the cells causing a gain in intracellular Na+ which draws in water to the cell and causes
edema. This is reversible and happens before the calcium activated enzyme destruction.
b- Incorrect. Pyknosis, karyorrhexis, and karyolysis are irreversible nuclear changes caused by
endonucleases which are activated by Ca+2 which enters an irreversibly damaged cell because of failure of
the Ca+2 pump. Therefore, an influx of calcium into the cell occurs, activating many enzymes including
endonucleases which cause pyknosis, karyorrhexis, and karyolysis . The decreased oxidative phosphorylation
and decreased ATP production because of mitochondrial damage due to cell injury causes an increase in
anaerobic glycolysis which, in turn, DECREASES pH.
c- Incorrect. Pyknosis, karyorrhexis, and karyolysis are irreversible nuclear changes caused by
endonucleases which are activated by Ca+2 which enters an irreversibly damaged cell because of failure of
the Ca+2 pump. Therefore, an influx of calcium into the cell occurs, activating many enzymes including
endonucleases which cause pyknosis, karyorrhexis, and karyolysis. When injury occurs to a cell, the
mitochondria are damaged and ATP production is DECREASED.
d- Correct. Pyknosis, karyorrhexis, and karyolysis are irreversible nuclear changes caused by endonucleases
which are activated by Ca+2 which enters an irreversibly damaged cell because of failure of the Ca+2 pump.
Therefore, an influx of calcium into the cell occurs, activating many enzymes including endonucleases which
cause pyknosis, karyorrhexis, and karyolysis
e- Incorrect. Pyknosis, karyorrhexis, and karyolysis are irreversible nuclear changes caused by
endonucleases which are activated by Ca+2 which enters an irreversibly damaged cell because of failure of
the Ca+2 pump. Therefore, an influx of calcium into the cell occurs, activating many enzymes including
endonucleases which cause pyknosis, karyorrhexis, and karyolysis. The decreased oxidative phosphorylation
and decreased ATP production because of mitochondrial damage due to cell injury causes an INCREASE in
anaerobic glycolysis.
Item Description: Dr. Plummer, 2014, Introduction to Path-Cellular Adaptation and Injury, Robbins Ch. 2 T.

A 20-year-old woman has two days of low grade fever with malaise, cough, and nasal congestion. Physical
examination reveals clear breath sounds. A white blood cell count shows mild to moderate elevation in
lymphocytes. The most likely etiology for this woman’s condition is:
A. Acute, bacterial infection
B. Parasitic infection
C. Allergic reaction
✓D. Viral infection
E. Anaphylaxis
Rationale:
a- Incorrect. This patient’s signs and symptoms and the predominance of lymphocytes in her peripheral blood
point to a viral infection. Though lymphocytes are often seen in chronic inflammation, they may also be the first
cells that appear in viral infections. Acute, bacterial infections are associated with increased neutrophils
b- Incorrect. This patient’s signs and symptoms and the predominance of lymphocytes in her peripheral blood
point to a viral infection. Though lymphocytes are often seen in chronic inflammation, they may also be the first
cells that appear in viral infections. Parasitic infections are associated with increased eosinophils
c- Incorrect. This patient’s signs and symptoms and the predominance of lymphocytes in her peripheral blood
point to a viral infection. Though lymphocytes are often seen in chronic inflammation, they may also be the first
cells that appear in viral infections. Allergic reactions are associated with increased eosinophils.
d- Correct. This patient’s signs and symptoms and the predominance of lymphocytes in her peripheral blood
point to a viral infection. Though lymphocytes are often seen in chronic inflammation, they may also be the first
cells that appear in viral infections
e- Incorrect. This patient’s signs and symptoms and the predominance of lymphocytes in her peripheral blood
point to a viral infection. Though lymphocytes are often seen in chronic inflammation, they may also be the first
cells that appear in viral infections. Anaphylaxis is life-threatening and acute and would not show increased
lymphocytes in the blood. The patient would be extremely short of breath.
Item Description: Dr. Plummer, 2014, Robbins Ch. 3 pg. 69-100
A 33-year-old woman undergoes successful surgery for a ruptured appendix. Approximately five to seven days
Question #: 75 the healing skin suture site predominantly contains:
post-operatively,
E. Neutrophils
F. Collagen
G. Granulomas
H. Keloid
✓E. Granulation tissue
Rationale:
a- Incorrect. Five to seven days after a normally healing injury (surgery), the wound site would predominantly
contain granulation tissue consisting mainly of new blood vessels, fibroblasts, and macrophages. Neutrophils
are predominant at about 1 day after injury (surgery).
b- Incorrect. Five to seven days after a normally healing injury (surgery), the wound site would predominantly
contain granulation tissue consisting mainly of new blood vessels, fibroblasts, and macrophages. Collagen is
the major component laid down in fibrosis or scar which may deposit around two weeks post injury (surgery) but
becomes prominent scar tissue by one or two months.
c- Incorrect. Five to seven days after a normally healing injury (surgery), the wound site would predominantly
contain granulation tissue consisting mainly of new blood vessels, fibroblasts, and macrophages. Granulomas
are inflammatory responses of a chronic nature especially found in tuberculosis, fungal infections, and foreign
body reactions.
d- Incorrect. Five to seven days after a normally healing injury (surgery), the wound site would predominantly
contain granulation tissue consisting mainly of new blood vessels, fibroblasts, and macrophages. A keloid is
excessive deposition of scar tissue beyond the boundaries of the wound which would occur much later.
e- Correct. Five to seven days after a normally healing injury (surgery), the wound site would predominantly
contain granulation tissue consisting mainly of new blood vessels, fibroblasts, and macrophages
Item Description: Dr. Plummer, 2014, Robbins Ch. 3, pg. 100-110
A 69-year-old man has guaiac positive stools and weight loss. He undergoes a colonoscopy and a large
ulcerating mass is discovered in his sigmoid colon. A biopsy of the mass reveals invasive, irregular, crowded
Question
glands #: 76
with pleomorphic nuclei. There is invasion into blood vessels and lymphatics. What is the tumor best
classified as?
A. Leiomyosarcoma
B. Melanoma
C. Fibroadenoma
D. Papilloma
✓E. Adenocarcinoma
Rationale:
a- Incorrect. Invasive, irregular, crowded glands with pleomorphic nuclei make up an adenocarcinoma which
is a malignant epithelial tumor. A leiomyosarcoma is a malignant mesenchymal tumor which originates from
smooth muscle not epithelial tissue.
b- Incorrect. Invasive, irregular, crowded glands with pleomorphic nuclei make up an adenocarcinoma which
is a malignant epithelial tumor. Melanoma is a malignant epithelial tumor but specifically it is a malignant
neuroeptihelial tumor. It does not form glands.
c- Incorrect. Invasive, irregular, crowded glands with pleomorphic nuclei make up an adenocarcinoma which
is a malignant epithelial tumor. Fibroadenoma is a benign tumor consisting both of glandular (adeno) and fibrous
elements. It is commonly found in the breast.
d- Incorrect. Invasive, irregular, crowded glands with pleomorphic nuclei make up an adenocarcinoma which
is a malignant epithelial tumor. Papilloma is a benign tumor of epithelial origin which is papillary in architecture.
e- Correct. Invasive, irregular, crowded glands with pleomorphic nuclei make up an adenocarcinoma which is a
malignant epithelial tumor.
Item Description: Dr. Plummer, 2014, Robbins Ch. 7.

PHYSIOLOGY
Given the normal intracellular vs. extracellular ion concentration in neurons, if current through K+
channels contributed the sole ionic conductance in neurons at rest, then the resting membrane
potential would equal to which of the following:

A. Equilibrium potential of sodium


B. Equilibrium potential of potassium
C. Equilibrium potential of chloride
D. Equilibrium potential of calcium
E. Equilibrium potential of magnesium

Rationale:

a. Incorrect: The membrane potential would reach the equilibrium potential of sodium only if sodium
contributed the sole ionic conductance in neurons at rest
b. Correct: Because the potassium channels are the only channels open
c. Incorrect. The membrane potential would reach the equilibrium potential of chloride only if chloride
contributed the sole ionic conductance in neurons at rest
d. Incorrect. The membrane potential would reach the equilibrium potential of calcium only if calcium
contributed the sole ionic conductance in neurons at rest
e. Incorrect. The membrane potential would reach the equilibrium potential of magnesium only if
magnesium contributed the sole ionic conductance in neurons at rest

Item Description: Dr. Ramos, Physiology, Resting Membrane Potential

If calcium channels were the only channels open at rest in neurons, and the concentration of
intracellular vs. extracellular calcium were the same, then the recorded membrane potential would be
which of the following:

A. 40mV
B. 0mV
C. -40mV
D. Question
-65mV#: 77
E. -80mV

Rationale:

a. Incorrect: This is the membrane potential generally observed at the peak of the action potential
b. Correct: Because the concentrations of intra vs. extracellular calcium are the same then the equilibrium
potential of calcium is 0mV.
c. Incorrect. This is the membrane potential generally needed for voltage-gated sodium channel activation
d. Incorrect. This is the normal resting membrane potential
e. Incorrect. This is the normal equilibrium potential for potassium
Item Description: Dr. Ramos, Physiology, Resting Membrane Potential

A drug that causes membrane potential depolarization in neurons might act via which of the following
mechanisms:

A. Opening of voltage-gated potassium channels


B. Opening of voltage-gated chloride channels
C. Closing of sodium-potassium pumps
D. Opening of “leak” potassium channels
E. Closing of “leak” sodium channels

Rationale:

a. Incorrect. Due to the intracellular concentration of potassium, opening of voltage-gated potassium


channels will cause hyperpolarization
b. Incorrect. Due to the concentration of extracellular chloride, opening of voltage-gated chloride channels
will cause hyperpolarization
c. Correct. Due to the extracellular concentration of sodium, closing of sodium-potassium pump will cause
depolarization
d. Incorrect. Due to the intracellular concentration of potassium, opening of “leak” potassium channels will
cause hyperpolarization.
e. Incorrect. Due to the extracellular concentration of sodium, closing of “leak” sodium channels will cause
hyperpolarization

Item Description: Dr. Ramos, Physiology, Resting Membrane Potential

If current through Na+ channels contributed the sole ionic conductance in neurons at rest then the
resting membrane potential would equal which of the following:

A. Equilibrium potential of sodium


B. Equilibrium potential of potassium
C. Equilibrium potential of chloride
D. Equilibrium potential of calcium
E. Equilibrium potential of magnesium

SOURCE: R. Ramos, Physiology, Resting Membrane Potential Cellular/Molecular Thread

If ion channels selectively permeable to potassium were open, and if the intracellular and extracellular
concentration of all ions were equal, then the resting membrane potential of a neuron would equal
which of the following:

A. The equilibrium potential for potassium


B. The equilibrium potential for calcium
C. The equilibrium potential for chloride
D. -60mv
E. Question
Zero#: 78

Rationale: Answer choices A and E are accepted as correct. As per Dr. Ramos, “Given that the stem states
that the concentrations of all ions are equal (which includes K+) then both “A” and “E” are correct because the
“equilibrium potential for K+” will equal “Zero” and this will be the resting membrane potential.”

SOURCE: R. Ramos, Physiology, Resting Membrane Potential Cellular/Molecular Thread

Which of the following scenarios would result in membrane potential depolarization:

A. Opening of voltage-gated potassium channels


B. Opening of voltage-gated chloride channels
C. Closing of sodium-potassium pumps
D. Opening of leak potassium channels
E. Closing of leak sodium channels

SOURCE: R. Ramos, Physiology, Resting Membrane Potential Cellular/Molecular Thread

Given the concentrations of ions, which of the following ions would have an equilibrium potential

equal to 0mV?
A. Calcium (Ca2+)
B. Chloride (Cl-)
C.Magnesium (Mg2+)
D.Potassium (K+)
E. Sodium (Na+)
SOURCE: R.Ramos PhD, Lecture: Physiology-Resting Membrane Potential. January 3, 1:00pm –
2:20pm
 Cellular/Molecular Thread

If Na+ channels contributed solely to the ionic conductance in neurons, then the resting membrane
potential would be approximately which of the following values?

A. -80mV

B. -65mV
C. -40mV

D. +40mV

E. +65mV

SOURCE: R.Ramos PhD, Lecture: Physiology-Resting Membrane Potential. January 3, 1:00pm – 2:20pm
Cellular/Molecular Thread
The resting membrane potential is more depolarized than that predicted for the equilibrium potential
for potassium due to the contribution of current flow through which of the following?
A. ‘Leak’ potassium channels
B. ‘Leak’ sodium channels
C.V oltage-gated potassium channels
D.V oltage-gated sodium channels
E. Sodium-potassium pump
SOURCE: R.Ramos PhD, Lecture: Physiology-Resting Membrane Potential. January 3, 1:00pm – 2:20pm
Question #: 79 Thread
Cellular/Molecular

The cell bodies of norepinephrine-containing neurons are located in the brain stem, mainly the:

A. Obex
B. Locus coeruleus
C. Vagal trigones
D. Lateral lemniscus
E. Tectum

Rationale:

a. Incorrect. The obex is the apex of the fourth ventricle


b. Correct. The locus coeruleus contains most of the cell bodies that produce norepinephrine
c. Incorrect. Vagal trigons are triangular swellings formed by dorsal motor and hypoglossal nuclei
d. Incorrect. Lateral lemniscus forms a band of fibers covering the superior cerebellar peduncle
e. Incorrect. The tectum is a posterior area of the ventricular space covering the superior and inferior
colliculi

Item Description: Dr. Torres, Power-Point Slides, Reading Assignments

Significant amounts of serotonin (5-HT)-containing neurons are found in the upper brain stem,
particularly in the pons and medulla (areas that are collectively called the):

A. Septal region
B. Ventral tegmental region
C. Meso-striatal pathway
D. Raphe nuclei
E. Supraoptic nucleus

Rationale:

a. Incorrect. The septal region does not contain 5-HT-containing neurons


b. Incorrect. The ventral tegmental region contains DA-containing neurons
c. Incorrect. The meso-striatal pathway links the substantia nigra to the striatum
d. Correct. The raphe nuclei contain an abundance of 5-HT-containing neurons
e. Incorrect. The supraoptic nucleus does not contain 5-HT-containing neurons

Item Description: Dr. Torres, Power-Point Slides, Reading Assignments

Within the nerve terminal, catecholamines can be inactivated by enzymes such as:

A. Dopamine beta-hydroxylase
B. Monoamine oxidase
C. Tyrosine hydroxylase
D. N-acetyltransferase
E. Aromatic amino acid decarboxylase (AAD)

Rationale:

a. Incorrect. Dopamine beta-hydroxylase converts dopamine into norepinephrine


b. Correct. Monoamine oxidase metabolizes catecholamines (e.g., NE and DA)
c. Incorrect. Tyrosine hydroxylase converts tyrosine into DOPA
d. Question #: 80 N-acetyltransferase is an enzyme responsible for the synthesis of melatonin
Incorrect.
e. Incorrect. AAD is the rate-limiting step of dopamine synthesis

Item Description: Dr. Torres, Power-Point Slides, Reading Assignments

The dissociative anesthetic ketamine and the psychedelic drug phencyclidine block receptors for the
neurotransmitter:

A. Dopamine
B. Norepinephrine
C. Glutamate
D. Serotonin
E. Acetylcholine

SOURCE: Dr. Torres 2013, Nervous System and Behavior, Neurochemistry, Lecture and Reading
Assignments Cellular/Molecular Thread

The benzodiazepine anxiolytics and barbiturate sedatives bind to the ligand-gated ion channel for the
neurotransmitter:

A. Glutamate
B. Serotonin
C. Epinephrine
D. Acetylcholine
E. GABA

SOURCE: Dr. Torres 2013, Nervous System and Behavior, Neurochemistry, Lecture and Reading
AssignmentsCellular/Molecular Thread

The most common inhibitory and excitatory neurotransmitters in the brain are:

A. Norepinephrine and epinephrine


B. Dopamine and serotonin
C. GABA and glutamate
D. Dynorphin and glycine
E. Anandamide and nitric oxide

SOURCE: Dr. Torres 2013, Nervous System and Behavior, Neurochemistry, Lecture and Reading
Assignments Cellular/Molecular Thread

The neurotransmitter thought to be involved in a variety of processes including sleep, feeding,


affective disorders and pain is:

A. Serotonin
B. Nitric oxide
C. Dopamine
D. Epinephrine
E. Norepinephrine

Rationale: As per Dr. Torres, “Serotonin is the (best) appropriate answer choice.”

SOURCE: Dr. Torres 2013, Nervous System and Behavior, Neurochemistry, Lecture and Reading
AssignmentsCellular/Molecular Thread
Cerebrospinal
Question #: 81fluid (CSF) is produced by the __________ and absorbed by the ___________ that
protrude into the venous sinuses

A. Subarachnoid space; dura mater


B. Epidural space; pia mater
C. Fourth ventricle; central canal
D. Interventricular foramen of Monro; third ventricle
E. Choroid plexus; arachnoid granulations or villi

Rationale:

a. Incorrect. Subarachnoid space does not produce CSF and CSF is not absorbed by the dura mater
b. Incorrect. Epidural space is a meningeal space and pia mater is part of the meninges
c. Incorrect. The fourth ventricle is part of the ventricular system and CSF is not absorbed by the central
canal
d. Incorrect. Interventricular foramen of Monro is a conduit within the ventricular system and the third
ventricle is part of the ventricular system
e. Correct. Choroid plexus produces CSF and CSF is absorbed by arachnoid granulations or villi

Item Description: Dr. Torres, Power-Point Slides, Reading Assignments

Which of the following regions lack the blood-brain barrier?

A. Amygdala
B. Organum vasculosum of the laminal terminalis
C. Hippocampus
D. Habenula
E. Nucleus accumbens

Rationale:

a. Incorrect. The amygdala is surrounded by the blood-brain barrier


b. Correct. Organum vasculosum of the laminal terminalis is devoid of a BBB
c. Incorrect. The BBB is present in the hippocampus
d. Incorrect. The BBB is present in the habenula
e. Incorrect. The BBB is present in the nucleus accumbens

Item Description: Dr. Torres, Power-Point Slides, Reading Assignments

The third ventricle communicates with the fourth ventricle through the:

A. Cerebral aqueduct
B. Interventricular foramina
C. Central cana
D. Subfornical organ
E. Median eminence

Rationale:

a. Correct. The cerebral aqueduct is the conduit between the third and fourth ventricle
b. Incorrect. Interventricular foramina is the conduit between lateral and third ventricle(s)
c. Incorrect. The central canal is an extension of the fourth ventricle to the medulla and spinal cord
d. Incorrect. The subfornical organ is devoid of a blood-brain barrier
e. Incorrect. The median eminence is part of the ventral hypothalamus
Item Description: Dr. Torres, Power-Point Slides, Reading Assignments
Question #: 82
The specialized structure that projects into the lateral, third and fourth ventricles of the brain is:

A. The choroid plexus


B. Astrocyte end-feet
C. Crus cerebri
D. Optic chiasm
E. Median foramen (of Magendie)

SOURCE: Dr. Torres, 2013, Nervous System and Behavior, Blood-Brain Barrier, CSF and Ventricles, Lecture,
Reading Assignment Cellular/Molecular Thread

Which of the following is a circumventricular organ:

A. Hippocampus
B. Amygdala
C. Area postrema
D. Caudate putamen
E. Septum

SOURCE: Dr. Torres, 2013, Nervous System and Behavior, Blood-Brain Barrier, CSF and Ventricles, Lecture,
Reading Assignment Cellular/Molecular Thread

Blood-brain barrier disruption is present in:

A. Multiple sclerosis
B. Serotonin syndrome
C. Grave‟s disease
D. Addison‟s disease
E. Autism spectrum disorders

SOURCE: Dr. Torres, 2013, Nervous System and Behavior, Blood-Brain Barrier, CSF and Ventricles, Lecture,
Reading Assignment Cellular/Molecular Thread

Detection of red blood cells in the cerebrospinal fluid could indicate:


A. Aseptic meningitis
B. Herniation or coning
C.Meningeal tumor(s)
D.Rhinorrhea
E. Subarachnoid hemorrhage
SOURCE: Dr. German Torres Power-Point Lecture 01-10-12 Cellular/ Molecular Thread
These tumors are benign, well-circumscribed, slow growing tumors. They account for 15% of primary
intracranial tumors and are more commonly diagnosed in females than males (3:2). In addition, ninety
percent of these tumors are supratentorial:
A. Astrocytoma
B. Ependymoma
C.Glioblastoma multiforme
D.Medulloblastoma
E. Meningioma
SOURCE: Dr. German Torres Power-Point Lecture 01-10-12 Cellular/ Molecular Thread
A 5-year-old boy presents to your office with fever, headache, nuchal rigidity and positive Kernig’s
sign. Cerebral spinal fluid profiles show numerous polymorphonuclear leukocytes, decreased glucose
levels and increased protein levels. What is the most likely diagnosis?
A. Bacterial meningitis
B. Septic encephalopathy
C.Subfalcine herniation
Question #: 83 herniation
D.Transforaminal
E. Viral meningitis
SOURCE: Dr. German Torres Power-Point Lecture 01-10-12 Cellular/ Molecular Thread

In the attached schematic illustration of an action potential, the changes in membrane potential
highlighted by the arrow are caused primarily by which of the following mechanisms:

A. Closing of ‘leak’ potassium channels


B. Opening of voltage-gated potassium channels
C. Opening of chloride channels
D. Opening of voltage-gated sodium channels
E. Closing of sodium-potassium pump

The emergency department at a local hospital runs out of Lidocaine. After the arrival of a patient with a
small, open wound in need of several stitches, the attending physician asks the staff pharmacist to
find a comparable analgesic with similar mechanisms of action. The pharmacist will be looking for a
blocker of which of the following:

A. ‘Leak’ potassium channels


B. Voltage-gated potassium channels
C. ‘Leak’ sodium channels
D. Voltage-gated sodium channels
E. Sodium-potassium pump

Which of the following might result in action potentials with a peak amplitude of only 0mV?
A. Decreases in extracellular chloride concentration
B. Increases in extracellular potassium concentration
C. Question #: 84 in intracellular potassium concentration
Decreases
D. Increases in extracellular sodium concentration
E. Equal concentration of intracellular and extracellular sodium concentration

The transition between the undershoot and the return of the membrane potential back to rest is
mediated by which of the following:

A. Closing of voltage-gated potassium channels


B. Closing of voltage-gated chloride channels
C. Opening of sodium-potassium pumps
D. Closing of leak potassium channels
E. Closing of leak sodium channels

Which of the following conditions would result in neurons emitting action potentials with larger
amplitudes and faster rising phases than usual:

A. Decreases in extracellular sodium concentration


B. Increases in extracellular sodium concentration
C. Decreases in extracellular chloride concentration
D. Decreases in extracellular potassium concentration
E. Decreases in extracellular sodium and potassium concentration

The analgesic effects of Lidocaine come from its potent blocking of which of the following:

A. „Leak‟ potassium channels


B. Voltage-gated potassium channels
C. „Leak‟ sodium channels
D. Voltage-gated sodium channels
E. Sodium-potassium pump

Which of the following best approximates the value of the membrane potential of a neuron during the
time indicated by the arrow in the figure above?

A. 
 -65mV

B. -40mV

C. 0mV

D. +40mV

E. Question
+65mV#: 85
Which of the following best describes ion flow in a neuron during the time indicated by the arrow in the

figure above?
A. Ion flow through calcium channels toward the equilibrium potential of calcium
B. Ion flow through chloride channels toward the equilibrium potential of potassium
C.Ion flow through potassium channels toward the equilibrium potential of potassium
D.Ion flow through potassium channels toward the equilibrium potential of sodium
E. Ion flow through sodium channels toward the midpoint between the equilibrium potential of sodium
Which of the following pharmacological approaches would be most beneficial in treating hyperactivity
of action potential firing such as in epilepsy?
A. Blocking ion flow through ‘leak’ potassium channels
B. Decreasing reuptake
C.Increasing voltage-gated sodium channel inactivation
D.Stimulating voltage-gated calcium channels
E. Stimulating voltage-gated sodium channels
You prescribe Lidocaine patches to a patient with shoulder pain and explain to her that these patches
will help relieve her pain by affecting which of the following channels?
A. Calcium channels
B. Chloride channels
C.Magnesium channels
D.Potassium channels
E. Sodium channels
Mutation of the scn1a gene which encodes a voltage-gated sodium channel has been linked to which
of the following disorders?
A. Alzheimer’s disease
B. Epilepsy
C.Huntington’s disease
D.Parkinson’s disease
E. Schizophrenia

Which of the following is responsible for the transition between the action potential undershoot and the return
of the membrane potential back to rest:
A. Opening of voltage-gated K+ channels
B. Activation of glutamate receptors
C. Opening of voltage-gated Na+ channels
✓D. Closing#:of86voltage-gated K+ channels
Question
E. Opening of voltage-gated Ca+ channels
Rationale:
a- Incorrect. The opening of voltage gated K+ channels would keep the membrane potential below the resting
membrane potential
b- Incorrect. Glutamate receptors do not contribute to the action potential
c- Incorrect. The opening of Na+ channels does not take place during the falling phase or undershoot.
d- Correct. Closing of voltage-gated K+ channels cause the membrane potential to return to resting levels. e-
Incorrect. Voltage gated Ca+ channels are not active.
Item Description: Dr. Ramos, 2014, Action Potential, Lecture and readings.

Question #: 109
An expansion of the ventricles due to tissue loss, as in the case of Alzheimer’s disease, is termed:
✓A. Hydrocephalus ex vacuo
B. Focal cerebral ischemia
C. Cytotoxic cerebral edema
D. Traumatic brain injury
E. Epidural hematoma
Rationale:
a-Correct. Hydrocephalus ex vacuo is the correct term that describes expansion of the ventricles b-Incorrect.
No expansion of the ventricles is normally seen in focal cerebral ischemia
c- Incorrect. No expansion of the ventricles is normally seen in cerebral edema
d- Incorrect. No expansion of the ventricles in normally seen in traumatic brain injury e-Incorrect. No
expansion of the ventricles is normally seen in epidural hematoma Item Description: Dr. Torres, 2014, Power-
Point Presentation, Reading Assignment.
Session # 99, 100 and Virtual: Resting Potential, Action Potential, and Synaptic Transmission
2017
Question #: 45
In the attached schematic illustration of an action potential, the changes in membrane potential highlighted by the
arrow are caused primarily by which of the following mechanisms:
A. Closing of ‘leak’ potassium channels
✓B. Opening of voltage-gated potassium channels
C. Opening of chloride channels
D. Opening of voltage-gated sodium channels
E. Closing of sodium-potassium pump
Rationale:
a- Incorrect. Closing of ‘leak’ potassium channels doesn’t occur during the action potential
b- Correct. Opening of voltage-gated potassium channels causes the falling phase of the action potential due to K+
leaving the neuron
c- Incorrect. Opening of voltage chloride channels doesn’t occur during the action potential
d- Incorrect. Opening of voltage-gated sodium channels is responsible for the rising phase of the action potential.
e- Incorrect. Closing of sodium-potassium pump doesn’t occur during the action potential
Attachment:
Question #: 87

Item Description: Dr. Ramos, Physiology, Action Potential TAGS Resting membrane potential

Question #: 46
The emergency department at a local hospital runs out of Lidocaine. After the arrival of a patient with a small, open
wound in need of several stitches, the attending physician asks the staff pharmacist to find a comparable analgesic with
similar mechanisms of action. The pharmacist will be looking for a blocker of which of the following:
A. ‘Leak’ potassium channels
B. Voltage-gated potassium channels
C. ‘Leak’ sodium channels
✓D. Voltage-gated sodium channels
E. Sodium-potassium pump
Rationale:
a- Incorrect. Lidocaine is a voltage-gated sodium channel blocker not a ‘leak’ potassium channel blocker
b- Incorrect. Lidocaine is a voltage-gated sodium channel blocker not a voltage-gated potassium channel blocker
c- Incorrect. Lidocaine is a voltage-gated sodium channel blocker not a ‘leak’ sodium channel blocker
d- Correct. Lidocaine is a voltage-gated sodium channel blocker
e- Incorrect. Lidocaine is a voltage-gated sodium channel blocker not a sodium-potassium pump blocker
Item Description: Dr. Ramos, Physiology, Action Potential

Question #: 47
Which of the following might result in action potentials with a peak amplitude of only 0mV?
A. Decreases in extracellular chloride concentration
B. Increases in extracellular potassium concentration
C. Decreases in intracellular potassium concentration
D. Increases in extracellular sodium concentration
✓E. Equal concentration of intracellular and extracellular sodium concentration
Rationale:
a- Incorrect. Because the peak of the action potential is regulated by sodium concentration, changes in chloride
concentration will have no effect
b- Incorrect. Because the peak of the action potential is regulated by sodium concentration, changes in chloride
concentration will have no effect
c- Incorrect. Because the peak of the action potential is regulated by sodium concentration, changes in chloride
concentration will have no effect
d- Incorrect. Increasing the extracellular concentration of sodium will increase the driving force and equilibrium
potential of potassium leading to a larger peak amplitude
e- Correct. Equal concentration of intracellular and extracellular sodium will result in an equilibrium potential for sodium
of 0mV
Item Description: Dr. Ramos, Physiology, Action Potential
Question #: 48
Question
Given #: 88intracellular vs. extracellular ion concentration in neurons, if current through K+ channels contributed
the normal
the sole ionic conductance in neurons at rest, then the resting membrane potential would equal to which of the
following:
A. Equilibrium potential of sodium
✓B. Equilibrium potential of potassium
C. Equilibrium potential of chloride
D. Equilibrium potential of calcium
E. Equilibrium potential of magnesium
Rationale:
a- Incorrect: The membrane potential would reach the equilibrium potential of sodium only if sodium contributed the
sole ionic conductance in neurons at rest
b- Correct: Because the potassium channels are the only channels open
c- Incorrect. The membrane potential would reach the equilibrium potential of chloride only if chloride contributed the
sole ionic conductance in neurons at rest
d- Incorrect. The membrane potential would reach the equilibrium potential of calcium only if calcium contributed the
sole ionic conductance in neurons at rest
e- Incorrect. The membrane potential would reach the equilibrium potential of magnesium only if magnesium
contributed the sole ionic conductance in neurons at rest
Item Description: Dr. Ramos, Physiology, Resting Membrane Potential

Question #: 49
If calcium channels were the only channels open at rest in neurons, and the concentration of intracellular vs.
extracellular calcium were the same, then the recorded membrane potential would be which of the following:
A. 40mV
✓B. 0mV
C. -40mV
D. -65mV
E. -80mV
Rationale:
a- Incorrect: This is the membrane potential generally observed at the peak of the action potential
b- Correct: Because the concentrations of intra vs. extracellular calcium are the same then the equilibrium potential of
calcium is 0mV.
c- Incorrect. This is the membrane potential generally needed for voltage-gated sodium channel activation
d- Incorrect. This is the normal resting membrane potential
e- Incorrect. This is the normal equilibrium potential for potassium
Item Description: Dr. Ramos, Physiology, Resting Membrane Potential

Question #: 50
A drug that causes membrane potential depolarization in neurons might act via which of the following mechanisms:
A. Opening of voltage-gated potassium channels
B. Opening of voltage-gated chloride channels
✓C. Closing of sodium-potassium pumps
D. Opening of “leak” potassium channels
E. Closing of “leak” sodium channels
Rationale:
a- Incorrect. Due to the intracellular concentration of potassium, opening of voltage-gated potassium channels will cause
hyperpolarization
b- Incorrect. Due to the concentration of extracellular chloride, opening of voltage-gated chloride channels will cause
hyperpolarization
c- Correct. Due to the extracellular concentration of sodium, closing of sodium-potassium pump will cause
depolarization
d- Incorrect. Due to the intracellular concentration of potassium, opening of “leak” potassium channels will cause
hyperpolarization.
Question Due
e- Incorrect. #: 89to the extracellular concentration of sodium, closing of “leak” sodium channels will cause
hyperpolarization
Item Description: Dr. Ramos, Physiology, Resting Membrane Potential

Question #: 51
Invasion of the action potential into the axon terminal results in opening of this type of ion channel which is responsible
for initiating vesicular release:
✓A. Calcium channels
B. Magnesium channels
C. Sodium channels
D. Chloride channels
E. Potassium channels
Rationale:
a- Correct. Calcium channels are responsible for initiating vesicular release
b- Incorrect. Magnesium channels are not responsible for initiating vesicular release
c- Incorrect. Sodium channels are not responsible for initiating vesicular release
d- Incorrect. Chloride channels are not responsible for initiating vesicular release
e- Incorrect. Potassium channels are not responsible for initiating vesicular
Item Description: Dr. Ramos, Physiology, Synaptic Transmission

Question #: 52
Given the concentrations of ions shown, synaptic transmission at an ionotropic glutamate synapse would result in which
of the following: (Please see attachment)
A. Sodium (Na+) movement into the postsynaptic neuron
B. Chloride (Cl-) movement into the postsynaptic neuron
C. Chloride (Cl-) movement out of the postsynaptic neuron
✓D. Potassium (K+) movement out of the postsynaptic neuron
✓E. Sodium (Na+) movement out of the postsynaptic neuron
Rationale:
a- Incorrect. Glutamate receptors allow sodium/potassium channels to open. Given the concentrations in the table,
sodium will rush out of the neuron
b- Incorrect. Glutamate receptors allow sodium/potassium channels to open. Chloride is not a permeant ion at
ionotropic glutamate receptors
c- Incorrect. Glutamate receptors allow sodium/potassium channels to open. Chloride is not a permeant ion at
ionotropic glutamate receptors
d- Correct. Glutamate receptors allow sodium/potassium channels to open. Potassium is a permeant ion at
ionotropic glutamate receptors
e- Correct. Glutamate receptors allow sodium/potassium channels to open. Given the concentrations in the table,
sodium will rush out of the neuron
Attachment:

Item Description: Dr. Ramos, Physiology, Synaptic Transmission

Question #: 53
Transmission at the neuromuscular junction results in post-synaptic depolarization because of which of the following:
✓A. The driving force of sodium is greater than the driving force of potassium
B. Sodium flows out of muscle through nicotinic receptors
Question #:
C. Potassium 90 out of muscle through nicotinic receptors
flows
D. Sodium flows out of glutamate receptors
E. Potassium flows out of glutamate receptors
Rationale:
a- Correct. The driving force of sodium is greater than the driving force of potassium so ion flow through nicotinic
receptors is depolarizing.
b- Incorrect. Sodium does not flow out of muscle through nicotinic receptors
c- Incorrect. Potassium does flow out of muscle through nicotinic receptors but this effect is hyperpolarizing
d- Incorrect. Sodium does not flow out of glutamate receptors
e- Incorrect. Potassium does not flow out of glutamate receptors
Item Description: Dr. Ramos, Physiology, Synaptic Transmission

Question #: 54
The binding of the neurotransmitter gamma-amino butyric acid (GABA) to its receptor on the postsynaptic membrane
results in which of the following:
✓A. Hyperpolarization in postsynaptic neurons via chloride flows into the neuron
B. Depolarization in postsynaptic neurons via potassium flows into the neuron
C. Hyperpolarization in postsynaptic neurons via potassium flows out of the neuron
D. Depolarization in postsynaptic neurons via sodium flows into the neuron
E. Depolarization in postsynaptic neurons via calcium flows out of the neuron
Rationale:
a- Correct. GABA receptor activation results in chloride flow into neurons which causes hyperpolarization
b- Incorrect. GABA receptor activation results in chloride flow into neurons
c- Incorrect. GABA receptor activation results in chloride flow into neurons
d- Incorrect. GABA receptor activation results in chloride flow into neurons
e- Incorrect. GABA receptor activation results in chloride flow into neurons
Item Description: Dr. Ramos, Physiology, Synaptic Transmission

2016
Question #: 60
The transition between the undershoot and the return of the membrane potential back to rest is mediated by which of
the following:
✓A. Closing of voltage-gated potassium channels
B. Closing of voltage-gated chloride channels
C. Opening of sodium-potassium pumps
D. Closing of leak potassium channels
E. Closing of leak sodium channels
SOURCE: R. Ramos, Physiology, Action Potential
TAGS: Action Potential
Rationale: As per Dr. Ramos, “The undershoot is established by the opening of voltage-gated K+ channels and the flow of
K+ out of the cell which drives the membrane potential to the equilibrium potential of K+ (-80mV). Therefore the
transition between the undershoot (-80mV) and the return to the resting membrane potential (-65mV) corresponds to a
15mV depolarization of the membrane. This depolarization is due to the closing of voltage-gated K+. The reason the
neuron returns to a resting potential of -65mV and not some other value (e.g. more depolarized or more hyperpolarized)
is because the “leak” K+ channels, Na+/K+ pumps, and Na+ leak channels establish the -65mV resting membrane
potential value. The question asks specifically asks about the mechanisms underlying the transition from the undershoot
to resting potential.”

Question #: 61
Which of the following conditions would result in neurons emitting action potentials with larger amplitudes and faster
rising phases than usual:
A. Decreases in extracellular sodium concentration
✓B. Increases in extracellular sodium concentration
Question #:
C. Decreases in 91
extracellular chloride concentration
D. Decreases in extracellular potassium concentration
E. Decreases in extracellular sodium and potassium concentration
SOURCE: R. Ramos, Physiology, Action Potential
TAGS: Action Potential

Question #: 62
The analgesic effects of Lidocaine come from its potent blocking of which of the following:
A. „Leak‟ potassium channels
B. Voltage-gated potassium channels
C. „Leak‟ sodium channels
✓D. Voltage-gated sodium channels
E. Sodium-potassium pump
SOURCE: R. Ramos, Physiology, Action Potential
TAGS: Action Potential

Question #: 63
If current through Na+ channels contributed the sole ionic conductance in neurons at rest then the resting membrane
potential would equal which of the following:
✓A. Equilibrium potential of sodium
B. Equilibrium potential of potassium
C. Equilibrium potential of chloride
D. Equilibrium potential of calcium
E. Equilibrium potential of magnesium
SOURCE: R. Ramos, Physiology, Resting Membrane Potential
TAGS: Resting membrane potential

Question #: 64
If ion channels selectively permeable to potassium were open, and if the intracellular and extracellular concentration of
all ions were equal, then the resting membrane potential of a neuron would equal which of the following:
✓A. The equilibrium potential for potassium
B. The equilibrium potential for calcium
C. The equilibrium potential for chloride
D. -60mv
✓E. Zero
SOURCE: R. Ramos, Physiology, Resting Membrane Potential
TAGS: Resting membrane potential
Rationale: Answer choices A and E are accepted as correct. As per Dr. Ramos, “Given that the stem states that the
concentrations of all ions are equal (which includes K+) then both “A” and “E” are correct because the “equilibrium
potential for K+” will equal “Zero” and this will be the resting membrane potential.”

Question #: 65
Which of the following scenarios would result in membrane potential depolarization:
A. Opening of voltage-gated potassium channels
B. Opening of voltage-gated chloride channels
✓C. Closing of sodium-potassium pumps
D. Opening of leak potassium channels
E. Closing of leak sodium channels
SOURCE: R. Ramos, Physiology, Resting Membrane Potential
TAGS: Resting membrane potential
Question #: 82
Question
Which of the #: 92
following is the least common type(s) of chemical synapses found in the brain:✓A. Axo-axonic
B. Axo-dendritic
C. Axo-somatic
D. Motorend plate
E. Dendro-dendritic
SOURCE: R. Ramos, Physiology, Synaptic Transmission
TAGS: Synaptic Transmission
Rationale: As per Dr. Ramos, “Motorend plates/Neuromuscular junctions are not found in the brain.”

Question #: 83
Propagation of the action potential into the axon terminal results in opening of this type of ion channel which is
responsible for initiating vesicular release:
✓A. Calcium channels
B. Magnesium channels
C. Sodium channels
D. Chloride channels
E. Potassium channels
SOURCE: R. Ramos, Physiology, Synaptic Transmission
TAGS: Synaptic Transmission
Rationale: As per Dr. Ramos, “Calcium channels are responsible for initiating vesicular release of neurotransmitter.”

Question #: 84
Synaptic activity at the synapse between the 1a afferent and the motor neuron would result in which of the following:
A. Hyperpolarization in the postsynaptic neuron via chloride flow into the cell
B. Depolarization in the postsynaptic neuron via potassium flow into the cell
C. Hyperpolarization in the postsynaptic neuron via potassium flow out of the cell
✓D. Depolarization in the postsynaptic neuron via sodium flow into the cell
E. Depolarization in the postsynaptic neuron via chloride flow out of the cell
SOURCE: R. Ramos, Physiology, Synaptic Transmission
TAGS: Synaptic Transmission

Question #: 94
During testing of experimental drugs for use as novel therapies, you record the following changes in the membrane
potential of a neuron shown in response to application of a drug. Which of the following do you suspect takes place in
response to the drug? (Please see attached)
A. Calcium channels close
B. Gamma amino-butyric acid (GABA) receptors open
✓C. Glutamate receptors open
D. “Leak” potassium channels open
E. Voltage-gated sodium channels close
Attachment:

SOURCE: SOURCE: R. Ramos, Physiology, Synaptic Transmission


TAGS: Synaptic Transmission
Question #: 95
Given the concentrations of ions shown, synaptic transmission at the neuromuscular synapse would result in which of
theQuestion
following:#:(Please
93 see attached)
A. Calcium (Ca2+) movement out of postsynaptic neurons
B. Chloride (Cl-) movement into postsynaptic neurons
C. Chloride (Cl-) movement out of postsynaptic neurons
✓D. Potassium (K+) movement out of postsynaptic neurons
E. Sodium (Na+) movement into postsynaptic neurons
Attachment:

SOURCE: SOURCE: R. Ramos, Physiology, Synaptic Transmission


TAGS: Synaptic Transmission
Cellular/Molecular Thread
Rationale: As per Dr. Ramos, “This question is designed to test knowledge about 1] the direction of flow of ions at
unequal concentrations across a cell membrane and 2] the properties of synaptic transmission at the neuromuscular
synapse. Therefore, given 1] the ion concentrations in the attached chart and 2] the fact that sodium and potassium can
flow through nicotinic receptors at the neuromuscular synapse, the only correct answer choice is “D.” Calcium and
chloride do not flow through nicotinic receptors. Given the concentrations in the attached chart, sodium will move out
of the cell which makes answer choice “E” incorrect.”

Question #: 96
Given the concentrations of ions shown, the equilibrium potential of potassium would equal which of the following:
(Please see attached)
✓A. -80mV
B. -65mV
C. -40mV
D. +40mV
E. +65mV
Attachment:

SOURCE: SOURCE: R. Ramos, Physiology, Synaptic Transmission


TAGS: Synaptic Transmission

Question #: 97
Which of the following best describes the cell-type shown in the picture: (Please see attached)A. Bipolar neuron
B. Astrocyte
C. Pseudo-unipolar neuron
✓D. Multipolar neuron
E. Oligodendrocyte
Attachment:
TAGS: Neurons

Question
Question #: 94
#: 99
Blockade of voltage gated calcium channels at the axon terminals of motor neurons will result in which of the following?
A. Increased acetylcholine release
✓B. Decreased acetylcholine release
C. Increased glutamate release
D. Decreased glutamate release
E. Decreased gamma amino butyric acid release
SOURCE: SOURCE: R. Ramos, Physiology, Synaptic Transmission
TAGS: Synaptic Transmission

2015

57.
Given the concentrations of ions shown above, gamma amino-butyric acid (GABA) neurotransmission
through ionotropic receptors in adult neurons would result in which of the following?
A. Calcium (Ca2+) movement out of postsynaptic neurons
B. Chloride (Cl-) movement into postsynaptic neurons
C. ✓Chloride (Cl-) movement out of postsynaptic neurons
D. Potassium (K+) movement out of postsynaptic neurons
E. Sodium (Na+) movement into postsynaptic neurons
SOURCE: R.Ramos PhD, Lecture: Physiology-Synaptic Transmission. January 4, 2:00pm – 3:50pm
Cellular/Molecular Thread
Rationale: GABA receptors open and chloride (Cl-) flows into the cell only because Cl- is normally at high
concentration outside the neuron. Therefore, GABA is inhibitory only because of the Cl- concentration
differences normally found in vs. outside the neuron. This question is designed to test whether students
understand that the flow of a given ion through a channel or receptor is governed by the concentration
gradients found in vs. outside a neuron. The chart that accompanies the question indicates a higher
concentration of Cl- inside vs outside the neuron. Therefore, upon opening of GABA receptors, Cl- would rush
out of the neuron due to the concentration difference indicated by the chart. In order to point the test-taker’s
attention to the chart, the following words were included to the question stem: “Given the concentrations of
ions shown above…” Thus the only acceptable answer to this question is “Chloride (Cl-) movement out of
postsynaptic neurons.”

58.
Given the concentrations of ions shown above, which of the following ions would have an equilibrium
potential equal to 0mV?
A. Calcium (Ca2+)
B. Chloride (Cl-)
C. Magnesium (Mg2+)
D. Potassium (K+)
E. ✓Sodium (Na+)
SOURCE: R.Ramos PhD, Lecture: Physiology-Resting Membrane Potential. January 3, 1:00pm – 2:20pm
Cellular/Molecular Thread
Question #: 95

59.
Which of the following best approximates the value of the membrane potential of a neuron during the
time indicated by the arrow in the figure above?
A. -65mV
B. -40mV
C. 0mV
D. ✓+40mV
E. +65mV
SOURCE: R.Ramos PhD, Lecture: Physiology-Action Potential. January 3, 2:30pm – 3:50pm Cellular/Molecular
Thread
Rationale: The peak of the action potential drives the membrane potential to approximately +40mV. This is
illustrated in numerous slides in the “Action Potential” lecture and is the most appropriate answer to this
question.

60.
Which of the following best describes ion flow in a neuron during the time indicated by the arrow in the
figure above?
A. Ion flow through calcium channels toward the equilibrium potential of calcium
B. Ion flow through chloride channels toward the equilibrium potential of potassium
C. ✓Ion flow through potassium channels toward the equilibrium potential of potassium
D. Ion flow through potassium channels toward the equilibrium potential of sodium
E. Ion flow through sodium channels toward the midpoint between the equilibrium potential of sodium
SOURCE: R.Ramos PhD, Lecture: Physiology-Action Potential. January 3, 2:30pm – 3:50pm
Cellular/MolecularThread
QuestionThe
Rationale: #: 96 arrow clearly points to the undershoot of the action potential which results from open
potassium channels which drive the membrane potential towards the potassium equilibrium potential.
Therefore the only acceptable answer for this question is “Ion flow through potassium channels toward the
equilibrium potential of potassium.”

5. If Na+ channels contributed solely to the ionic conductance in neurons, then the resting membrane
potential would be approximately which of the following values?
A. -80mV
B. -65mV
C. -40mV
D. +40mV
E. ✓ +65mV
SOURCE: R.Ramos PhD, Lecture: Physiology-Resting Membrane Potential. January 3, 1:00pm – 2:20pm
Cellular/Molecular Thread

6. You explain to your patient that the drug you are prescribing will result in more neurotransmitter to be
present at the synapse. Which of the following is the mechanism of action of that drug?
A. Blocks voltage-gated sodium channels
B. Increases ion flow through ‘leak’ potassium channels
C. ✓Increases reuptake
D. Increases voltage-gated sodium channel inactivation
E. Stimulates voltage-gated calcium channels

SOURCE: R.Ramos PhD, Lecture: Physiology-Synaptic Transmission. January 4, 2:00pm – 3:50pm


Cellular/Molecular Thread

7. Which of the following pharmacological approaches would be most beneficial in treating hyperactivity of
action potential firing such as in epilepsy?
A. Blocking ion flow through ‘leak’ potassium channels
B. Decreasing reuptake
C. ✓Increasing voltage-gated sodium channel inactivation
D. Stimulating voltage-gated calcium channels
E. Stimulating voltage-gated sodium channels
SOURCE: R.Ramos PhD, Lecture: Physiology-Action Potential. January 3, 2:30pm – 3:50pm
Cellular/Molecular Thread

64.
During testing of experimental drugs for use as novel therapies, you record the following changes in the
membrane potential of a neuron shown above in response to application of a drug. Which of the
following do you suspect takes place in response to the drug?
A. Calcium channels open
B. ✓Gamma amino-butyric acid (GABA) receptors open
C. Glutamate receptors open
D. “Leak” potassium channels close
E. Voltage-gated sodium channels open
SOURCE: R.Ramos PhD, Lecture: Physiology-Synaptic Transmission. January 4, 2:00pm – 3:50pm
Rationale: The figure that accompanies the question illustrates a recording of hyperpolarizing membrane
potential changes. Based on the understanding that GABA receptors cause hyperpolarization via Cl- flow through
the receptor, the only acceptable answer is “Gamma amino-butyric acid (GABA) receptors open.” Receptors
can indeed open as is the case with GABA receptors which have a Cl- pore.
Question #: 97
5. The synapse between the muscle spindle-associated 1A dorsal root neuron and the alpha motor neuron
utilizes which of the following neurotransmitters?
A. Acetylcholine
B. Dopamine
C. Gamma amino-butyric acid (GABA)
D. ✓Glutamate
E. Serotonin

SOURCE: R.Ramos PhD, Lecture: Physiology-Synaptic Transmission. January 4, 2:00pm – 3:50pm


Cellular/Molecular Thread
6. You prescribe Lidocaine patches to a patient with shoulder pain and explain to her that these patches will
help relieve her pain by affecting which of the following channels?

A. Calcium channels
B. Chloride channels
C. Magnesium channels
D. Potassium channels
E. ✓Sodium channels

SOURCE: R.Ramos PhD, Lecture: Physiology-Action Potential. January 3, 2:30pm – 3:50pm Cellular/Molecular
Thread

7. The resting membrane potential is more depolarized than that predicted for the equilibrium potential for
potassium due to the contribution of current flow through which of the following?

A. ‘Leak’ potassium channels


B. ✓ ‘Leak’ sodium channels
C. Voltage-gated potassium channels
D. Voltage-gated sodium channels
E. Sodium-potassium pump
SOURCE: R.Ramos PhD, Lecture: Physiology-Resting Membrane Potential. January 3, 1:00pm – 2:20pm
Cellular/Molecular Thread

8. Mutation of the scn1a gene which encodes a voltage-gated sodium channel has been linked to which of the
following disorders?

A. Alzheimer’s disease
B. ✓Epilepsy
C. Huntington’s disease
D. Parkinson’s disease
E. Schizophrenia

SOURCE: R.Ramos PhD, Lecture: Physiology-Action Potential. January 3, 2:30pm – 3:50pm


Cellular/Molecular Thread

2014
5. You prescribe Lidocaine patches to a pregnant patient with back pain and explain to her that these
patches will help relieve her pain by affecting which of the following:
A. Calcium channels
B. Chloride channels
C. Magnesium channels
D. Potassium channels
E. ✓Sodium
Question #: 98 channels
SOURCE: Dr. Ramos, Lecture and readings: Action Potential Cellular/Molecular Thread

6. Voltage-gated sodium channels are found preferentially in which part of neurons?


A. ✓Axon hillock
B. Axon terminal
C. Dendrites
D. Soma
E. Spines
SOURCE: Dr. Ramos, Lecture and readings: Action Potential Cellular/Molecular Thread

7. Mutation of the scn1a gene which encodes a voltage-gated sodium channel has been linked to which of the following
disorders?
A. Alzheimer’s disease
B. ✓Epilepsy
C. Huntington’s disease
D. Parkinson’s disease
E. Schizophrenia
SOURCE: Dr. Ramos, Lecture and readings: Action Potential Cellular/Molecular Thread

5. Which one of the following ions is found at high intracellular concentration compared to that of
extracellular fluid?
A. Calcium
B. Chloride
C. Magnesium
D. ✓Potassium
E. Sodium
SOURCE: Dr. Ramos, Lecture and readings: Resting Membrane Potential Cellular/Molecular Thread

6. You are the attending physician in the emergency department and treat a patient with “redtide” shellfish
poisoning. You explain to the patient’s family that the cause of the patient’s near death was due to a toxin
produced by micro-organisms that were present in the shellfish which affect:
A. Calcium channels
B. Chloride channels
C. Magnesium channels
D. Potassium channels
E. ✓Sodium channels
SOURCE: Dr. Ramos, Lecture and readings: Action Potential Cellular/Molecular Thread

7. Neurons that make synapses on the soma of postsynaptic neurons in the central nervous system typically
use which of the following neurotransmitters:
A. Acetylcholine
B. Epinephrine
C. ✓GABA
D. Glutamate
E. Serotonin
SOURCE: Dr. Ramos, Lecture and readings: Synaptic transmission Cellular/Molecular Thread

8. You explain to your patient that the drug you are prescribing will result in more neurotransmitter to be
present at the synapse. The mechanism of action of that drug is which of the following:
A. ✓Blocks reuptake
B. Blocks voltage-gated calcium channels
C. Blocks voltage-gated sodium channels
D. Increases ion flow through ‘leak’ potassium channels
E. Question #: 99 voltage-gated sodium channel inactivation
Increases
SOURCE: Dr. Ramos, Lecture and readings: Synaptic transmission
Cellular/Molecular Thread.
Session #108 and 109: Neurochemistry and Blood-Brain Barrier, CSF, Ventricles
2017
Question #: 55
Cerebrospinal fluid (CSF) is produced by the __________ and absorbed by the ___________ that protrude into the
venous sinuses
A. Subarachnoid space; dura mater
B. Epidural space; pia mater
C. Fourth ventricle; central canal
D. Interventricular foramen of Monro; third ventricle
✓E. Choroid plexus; arachnoid granulations or villi
Rationale:
a- Incorrect. Subarachnoid space does not produce CSF and CSF is not absorbed by the dura mater
b- Incorrect. Epidural space is a meningeal space and pia mater is part of the meninges
c- Incorrect. The fourth ventricle is part of the ventricular system and CSF is not absorbed by the central canal
d- Incorrect. Interventricular foramen of Monro is a conduit within the ventricular system and the third ventricle is part
of the ventricular system
e- Correct. Choroid plexus produces CSF and CSF is absorbed by arachnoid granulations or villi
Item Description: Dr. Torres, Power-Point Slides, Reading Assignments

Question #: 56
Which of the following regions lack the blood-brain barrier?
A. Amygdala
✓B. Organum vasculosum of the laminal terminalis
C. Hippocampus
D. Habenula
E. Nucleus accumbens
Rationale:
a- Incorrect. The amygdala is surrounded by the blood-brain barrier
b- Correct. Organum vasculosum of the laminal terminalis is devoid of a BBB
c- Incorrect. The BBB is present in the hippocampus
d- Incorrect. The BBB is present in the habenula
e- Incorrect. The BBB is present in the nucleus accumbens
Item Description: Dr. Torres, Power-Point Slides, Reading Assignments

Question #: 57
The third ventricle communicates with the fourth ventricle through the:
✓A. Cerebral aqueduct
B. Interventricular foramina
C. Central canal
D. Subfornical organ
E. Median eminence
Rationale:
a- Correct. The cerebral aqueduct is the conduit between the third and fourth ventricle
b- Incorrect. Interventricular foramina is the conduit between lateral and third ventricle(s)
c- Incorrect. The central canal is an extension of the fourth ventricle to the medulla and spinal cord
d- Incorrect. The subfornical organ is devoid of a blood-brain barrier
e- Incorrect. The median eminence is part of the ventral hypothalamus
Item Description: Dr. Torres, Power-Point Slides, Reading Assignments

Question
Question #: 100
#: 58
The cell bodies of norepinephrine-containing neurons are located in the brain stem, mainly the:
A. Obex
✓B. Locus coeruleus
C. Vagal trigones
D. Lateral lemniscus
E. Tectum
Rationale:
a- Incorrect. The obex is the apex of the fourth ventricle
b- Correct. The locus coeruleus contains most of the cell bodies that produce norepinephrine
c- Incorrect. Vagal trigons are triangular swellings formed by dorsal motor and hypoglossal nuclei
d- Incorrect. Lateral lemniscus forms a band of fibers covering the superior cerebellar peduncle
e- Incorrect. The tectum is a posterior area of the ventricular space covering the superior and inferior
colliculi
Item Description: Dr. Torres, Power-Point Slides, Reading Assignments

Question #: 59
Significant amounts of serotonin (5-HT)-containing neurons are found in the upper brain stem, particularly in the pons
and medulla (areas that are collectively called the):
A. Septal region
B. Ventral tegmental region
C. Meso-striatal pathway
✓D. Raphe nuclei
E. Supraoptic nucleus
Rationale:
a- Incorrect. The septal region does not contain 5-HT-containing neurons
b- Incorrect. The ventral tegmental region contains DA-containing neurons
c- Incorrect. The meso-striatal pathway links the substantia nigra to the striatum
d- Correct. The raphe nuclei contain an abundance of 5-HT-containing neurons
e- Incorrect. The supraoptic nucleus does not contain 5-HT-containing neurons
Item Description: Dr. Torres, Power-Point Slides, Reading Assignments

Question #: 60
Within the nerve terminal, catecholamines can be inactivated by enzymes such as:
A. Dopamine beta-hydroxylase
✓B. Monoamine oxidase
C. Tyrosine hydroxylase
D. N-acetyltransferase
E. Aromatic amino acid decarboxylase (AAD)
Rationale:
a- Incorrect. Dopamine beta-hydroxylase converts dopamine into norepinephrine
b- Correct. Monoamine oxidase metabolizes catecholamines (e.g., NE and DA)
c- Incorrect. Tyrosine hydroxylase converts tyrosine into DOPA
d- Incorrect. N-acetyltransferase is an enzyme responsible for the synthesis of melatonin
e- Incorrect. AAD is the rate-limiting step of dopamine synthesis
Item Description: Dr. Torres, Power-Point Slides, Reading Assignments

Question #: 61
Which of the following pituitary gland hormones is regulated by the neurotransmitter dopamine?
A. Luteinizing hormone
B. Follicle-stimulating hormone
C. Growth hormone
D. Adrenocorticotropic hormone
✓E.Question
Prolactin#: 101
hormone
Rationale:
a- Incorrect. Luteinizing hormone is not affected by dopamine
b- Incorrect. Follicle-stimulating hormone is not regulated by dopamine
c- Incorrect. Growth hormone secretion is not regulated by brain dopamine
d- Incorrect. Adrenocorticotropic secretion is not affected by dopamine levels
e- Correct. Prolactin is regulated by the neurotransmitter dopamine
Item Description: Dr. Torres, Power-Point Slides, Reading Assignments

Question #: 62
Deficiencies in acetylcholine-releasing neurons have classically been associated with dysfunctions often seen in:
A. Parkinson’s disease
B. Huntington’s disease
C. Amyotrophic lateral sclerosis
✓D. Alzheimer’s disease
E. Lambert-Eaton myasthenic syndrome (LEMS)
Rationale:
a- Incorrect. Parkinson’s disease is associated with deficiencies in dopamine
b- Incorrect. Huntington’s disease is associated with deficiencies in GABA
c- Incorrect. Amyotrophic lateral sclerosis is an autoimmune disease
d- Correct. Alzheimer’s disease is associated with deficiencies in acetylcholine
e- Incorrect. LEMS is an autoimmune disorder of neuromuscular transmission
Item Description: Dr. Torres, Power-Point Slides, Reading Assignments

2016
Question #: 8
The dissociative anesthetic ketamine and the psychedelic drug phencyclidine block receptors for the neurotransmitter:
A. Dopamine
B. Norepinephrine
✓C. Glutamate
D. Serotonin
E. Acetylcholine
SOURCE: Dr. Torres 2013, Nervous System and Behavior, Neurochemistry, Lecture and Reading
Assignments
Cellular/Molecular Thread

Question #: 9
The benzodiazepine anxiolytics and barbiturate sedatives bind to the ligand-gated ion channel for the neurotransmitter:
A. Glutamate
B. Serotonin
C. Epinephrine
D. Acetylcholine
✓E. GABA
SOURCE: Dr. Torres 2013, Nervous System and Behavior, Neurochemistry, Lecture and Reading
Assignments
Cellular/Molecular Thread

Question #: 10
The most common inhibitory and excitatory neurotransmitters in the brain are:
A. Norepinephrine and epinephrine
B. Dopamine and serotonin
✓C. GABA and glutamate
Question #:and
D. Dynorphin 102glycine
E. Anandamide and nitric oxide
SOURCE: Dr. Torres 2013, Nervous System and Behavior, Neurochemistry, Lecture and Reading
Assignments
Cellular/Molecular Thread

Question #: 11
The neurotransmitter thought to be involved in a variety of processes including sleep, feeding, affective disorders and
pain is:
✓A. Serotonin
B. Nitric oxide
C. Dopamine
D. Epinephrine
E. Norepinephrine
SOURCE: Dr. Torres 2013, Nervous System and Behavior, Neurochemistry, Lecture and Reading
Assignments
Cellular/Molecular Thread
Rationale: As per Dr. Torres, “Serotonin is the (best) appropriate answer choice.”

Question #: 16
The specialized structure that projects into the lateral, third and fourth ventricles of the brain is:
✓A. The choroid plexus
B. Astrocyte end-feet
C. Crus cerebri
D. Optic chiasm
E. Median foramen (of Magendie)
SOURCE: Dr. Torres, 2013, Nervous System and Behavior, Blood-Brain Barrier, CSF and Ventricles,
Lecture, Reading Assignment
Cellular/Molecular Thread

Question #: 17
Which of the following is a circumventricular organ:
A. Hippocampus
B. Amygdala
✓C. Area postrema
D. Caudate putamen
E. Septum
SOURCE: Dr. Torres, 2013, Nervous System and Behavior, Blood-Brain Barrier, CSF and Ventricles,
Lecture, Reading Assignment
Cellular/Molecular Thread

Question #: 18
Blood-brain barrier disruption is present in:
✓A. Multiple sclerosis
B. Serotonin syndrome
C. Grave‟s disease
D. Addison‟s disease
E. Autism spectrum disorders
SOURCE: Dr. Torres, 2013, Nervous System and Behavior, Blood-Brain Barrier, CSF and Ventricles,
Lecture, Reading Assignment
Cellular/Molecular Thread

Question #: 103
2015
92. Detection of red blood cells in the cerebrospinal fluid could indicate:
A. Aseptic meningitis
B. Herniation or coning
C. Meningeal tumor(s)
D. Rhinorrhea
E. ✓Subarachnoid hemorrhage
SOURCE: Dr. German Torres Power-Point Lecture 01-10-12
Cellular/ Molecular Thread

93. These tumors are benign, well-circumscribed, slow growing tumors. They account for 15% of primary
intracranial tumors and are more commonly diagnosed in females than males (3:2). In addition, ninety percent of these
tumors are supratentorial:
A. Astrocytoma
B. Ependymoma
C. Glioblastoma multiforme
D. Medulloblastoma
E. ✓Meningioma
SOURCE: Dr. German Torres Power-Point Lecture 01-10-12
Cellular/ Molecular Thread

94. A 5-year-old boy presents to your office with fever, headache, nuchal rigidity and positive Kernig’s sign.Cerebral
spinal fluid profiles show numerous polymorphonuclear leukocytes, decreased glucose levels and increased protein levels.
What is the most likely diagnosis?
A. ✓Bacterial meningitis
B. Septic encephalopathy
C. Subfalcine herniation
D. Transforaminal herniation
E. Viral meningitis

64. Which of the following neurotransmitters is classified as a “gaseous” molecule?


A. Adenosine
B. Anandamine
C. Dynorphin
D. Glutamate
E. ✓Nitric oxide
SOURCE: Dr. German Torres, Power-point Lecture 2/07/2012 Cellular and Molecular Thread

65. The therapeutic index is defined as:


A. The ratio between an agonist and an antagonist drug
B. The ratio of ED50 to LD50
C. The ratio of efficacy to potency
D. ✓The ratio of LD50 to ED50
E. The ratio of potency to efficacy
SOURCE: Dr. German Torres, Power-point Lecture 2/07/2012
Cellular and Molecular Thread

66. The catecholamines include the following neurotransmitters:


A. Acetylcholine and serotonin
B. Cocaine- and amphetamine-regulated transcript and dynorphin
C. Gamma-aminobutyric acid and adenosine
D. Glutamate and histamine
E. ✓Norepinephrine and dopamine
SOURCE: Dr. German Torres, Power-point Lecture 2/07/2012
Question
Cellular #: 104
and Molecular Thread

67. Antipsychotic medications chiefly affect the neurotransmitter:


A. Acetylcholine
B. ✓Dopamine
C. Glutamate
D. Norepinephrine
E. Serotonin
SOURCE: Dr. German Torres, Power-point Lecture 2/07/2012
Cellular and Molecular Thread
Rationale: The question clearly stated: the primary/main neurotransmitter system. And that is dopamine
which is chiefly affected by ALL antipsychotic drugs

68. Cocaine’s major mechanism of action in the brain is:


A. Enzyme inhibition
B. Receptor blockade
C. Receptor stimulation
D. Retrograde inhibition
E. ✓Reuptake blockade
SOURCE: Dr. German Torres, Power-point Lecture 2/07/2012
Cellular and Molecular Thread

74. Which of the following drugs is a dissociative anesthetic?


A. Etomidate
B. ✓Ketamine
C. Lidocaine
D. Phenobarbital
E. Propofol
SOURCE: Dr. German Torres, Power-point Lecture 2/07/2012
Cellular and Molecular Thread

75. Atrophy of the caudate nucleus and putamen, and dilation of the anterior horns of the lateral ventricles are
characteristics of which of the following neurodegenerative states?
A. Alzheimer’s disease
B. ✓Huntington’s disease
C. Multiple sclerosis
D. Narcolepsy
E. Pick’s disease
SOURCE: Dr. German Torres, Power-point Lecture 2/07/2012
Cellular and Molecular Thread

2014
95. A 68-year-old man presents to the emergency department with a head injury. Cerebrospinal fluid profiles show the
presence of red blood cells; protein is normal to slightly elevated (<45 mg/dL); and glucose levels are normal (80-120
mg/dL). Computed tomography scan of the head shows a crescent-shaped hypodense area between the cortex and
skull. What is the most likely diagnosis?
A. ✓Subdural hematoma
B. Transtentorial herniation
C. Benign adenoma
D. Hydrocephalus
E. Hydrocephalus ex vacuo
SOURCE: Dr. German Torres Power-Point Lecture 1-6-11
Cellular/ Molecular Thread
Question #: 105
96. A 5-year-old boy presents to your office with fever, headache, nuchal rigidity and Kernig’s sign.
Cerebral spinal fluid profiles show numerous polymorphonuclear leukocytes, decreased glucose levels and increased
protein levels. What is the most likely diagnosis?
A. Viral meningitis
B. Subfalcine herniation
C. Transforaminal herniation
D. ✓Bacterial meningitis
E. Septic encephalopathy
SOURCE: Dr. German Torres Power-Point Lecture 1-6-11
Cellular/ Molecular Thread

97. A 55-year-old male patient with bone metastases from lung cancer experienced nausea before treatment, despite
regular use of prochlorperazine (a dopamine receptor antagonist with anti-muscarinic action). He received radiotherapy
to the lumbar spine which produced emesis almost immediately, with 4 episodes of emesis in the first 2 hrs. Which of
the following brain regions is responsible for the aforementioned emetic response?
A. ✓Area postrema
B. Periaqueductal gray matter
C. Amygdala
D. Retro trapezoid nucleus
E. Nucleus accumbens
SOURCE: Dr. German Torres Power-Point Lecture 1-6-11
Cellular/ Molecular Thread

98. For many years, the brain had been regarded as an immunologically privileged site, where no immunosurveillance
takes place. Further, the blood-brain barrier had been assumed to be an effective barrier for blood cells since only a few
leukocytes have been found in the normal brain. However, in the following disease, the recruitment of inflammatory
cells has been readily detected both in the brain parenchyma and in the cerebrospinal fluid:
A. Bipolar disorder
B. Parkinson’s disease
C. ✓Multiple sclerosis
D. Schizophrenia
E. Huntington’s disease
SOURCE: Dr. German Torres Power-Point Lecture 1-6-11, Reading Assignment: Choroid Plexus:
Biology and Pathology. Acta Neuropathology (2010) 119-75-88
Cellular/Molecular Thread

41. This pediatric anesthetic produces a relatively sustained and rapid antidepressant effect in adult patients with major
depression, particularly in those individuals with difficult to treat recurrent depression:
A. Chlorpromazine
B. Felbamate
C. ✓Ketamine
D. Lamotrigine
E. Valproate
SOURCE: Dr. German Torres Power-Point Lecture 1-27-11
Cellular/ Molecular Thread

42. A 19-year-old man is diagnosed with epilepsy. One approach in treating this brain disorder is to give the patient
valproate which has a selective action upon neurotransmitter systems. Which of the following neurotransmitter systems
would valproate primarily act upon to relieve his focal seizures?
A. ✓Gamma-aminobutyric acid system
B. Glutamate system
C. Glycine system
D. Question #: 106
Nicotinic system
E. Serotonin system
SOURCE: Dr. German Torres Power-Point Lecture 1-27-11
Cellular/ Molecular Thread

43. An American child of Pakistani origin presents to your office with severe obesity (body mass index =
(BMI) > 40) and hyperphagia accompanied by metabolic, neuroendocrine and immune dysfunction. As
there are no additional clinical features suggestive of any recognized syndromic form of obesity, you
order the measurement of serum leptin using solid phase sandwich ELISA. Results from the ELISA test
indicate undetectable levels of leptin in the child. You next order direct sequencing of the ob gene for
both the proband (i.e., child) and her parents. Direct sequencing of the ob gene reveals the child to be
homozygous for deletion of a single guanine nucleotide in codon 133 of the open reading frame. The
deletion is present in the heterozygous state in both parents. Based upon these laboratory findings you
immediately place the child on recombinant methionyl human leptin therapy. What would be the effects of the above
drug therapy on the child’s BMI, plasma triglycerides and high density lipoprotein (HDL)?
A. ✓BMI decreases, plasma triglycerides are normalized and HDL increases
B. BMI increases, plasma triglycerides stay the same and HDL stays the same
C. BMI increases, plasma triglycerides increase and HDL decreases
D. BMI stays the same, plasma triglycerides increase and HDL decreases
E. BMI stays the same, plasma triglycerides stay the same and HDL stays the same

44. A 25-year-old man with epileptic seizures is treated with carbamazepine (Tegretol) in a monotherapy
dose of 10 mg/kg/day. Which of the following is the principal mechanism of action of carbamazepine?
A. Blocks calcium channels
B. ✓Blocks sodium channels
C. Blocks the N-Methyl-D-Aspartate receptor subtype
D. Decreases levels of gamma-aminobutyric acid
E. Increases potassium channel permeability
SOURCE: Dr. German Torres Power-Point Lecture 1-27-11
Cellular/ Molecular Thread

45. A 20-year-old man is found dead in his bed by his sisters. Although the young man had been suffering from recurring
seizures, the death was not the direct cause of a seizure or status epilepticus. There were signs of arrested apnea and
asystole. Postmortem examination did not reveal an anatomical or toxicological cause for the death. Which of the
following is the most likely cause of death in this young man?
A. Dysthymia depression
B. Focal seizure in epilepsy
C. Narcolepsy
D. ✓Sudden unexpected death in epilepsy
E. Systemic lupus erythematosus
SOURCE: Dr. German Torres Power-Point Lecture 1-27-11
Cellular/ Molecular Thread

46. A postmortem examination of brain tissue from an elderly male patient reveals characteristic pathological
abnormalities including severe atrophy of the caudate nucleus, putamen and cerebral cortex. On microscopic
examination, there is severe neuronal loss (especially in gamma-aminobutyric acid-containing medium spiny neurons)
and gliosis in the striatum, with the caudate nucleus being more affected than the putamen. Based upon the above
pathology, what was the disease phenotype of this individual?
A. Gilles de la Tourette syndrome
B. ✓Huntington’s disease
C. Narcolepsy
D. Parkinson’s disease
E. Schizophrenia
SOURCE: Dr. German Torres Power-Point Lecture 1-27-11.
Question #: 107

Which of the following is an example of conductive hearing loss?

A. Inner hair cell degeneration


B. Acoustic neuroma
C. Malformation of the ossicles
D. Mondini malformation
E. Presbycusis

Rationale:

a. Incorrect. Inner hair cell degeneration will be described as sensorineural hearing loss.
b. Incorrect. Acoustic neuroma would be classified as sensorineural hearing loss.
c. Correct. Because the ossicles are part of the middle ear, this is an example of conductive hearing loss.
d. Incorrect. Mondini malformations involve the cochlea so this would be an example of sensorineural hearing loss.
e. Incorrect. Presbycusis is an age-related hearing loss of high frequency sounds due to hair cell death which would
be classified as sensorineural hearing loss.
Item Description: Dr. Ramos, Auditory system

Which of the following is an example of sensorineural hearing loss?

A. Mondini malformation
B. Otitis media infection
C. Middle ear barotraumas
D. Otosclerosis of the stapes
E. Cerumen impaction

Rationale:

a. Correct. This is an example of sensorineural hearing loss.


b. Incorrect. This is an example of coductive hearing loss.
c. Incorrect. This is an example of coductive hearing loss.
d. Incorrect. This is an example of coductive hearing loss.
e. Incorrect. This is an example of coductive hearing loss.
Item Description: Dr. Ramos, Auditory system

Outer hair cells release which of the following neurotransmitters?

A. GABA
B. Dopamine
C. Acetylcholine
D. Serotonin
E. Glutamate

Rationale:

a. Incorrect. The inner and outer hair cells release glutamate.


b. Incorrect. The inner and outer hair cells release glutamate.
c. Incorrect. The inner and outer hair cells release glutamate.
d. Incorrect. The inner and outer hair cells release glutamate.
e. Correct. The inner and outer hair cells release glutamate.
Item Description: Dr. Ramos, Auditory system

Question #: 108
Which of the following structures makes direct contact with the oval window of the cochlea?

A. Malleus
B. Stapes
C. Incus
D. Basilar membrane
E. Tectorial membrane

Rationale:

a. Incorrect. The malleus makes contact with the tympanic membrane.


b. Correct. The stapes makes contact with the oval window.
c. Incorrect. The incus makes contact with the malleus and stapes.
d. Incorrect. The basilar membrane does not make contact with the oval window.
e. Incorrect. The tectorial membrane does not make contact with the oval window.
Item Description: Dr. Ramos, Auditory system

Which of the following structures makes direct contact with the oval window of the cochlea?

A. Malleus
B. Stapes
C. Incus
D. Tympanic membrane
E. Inner hair cells
SOURCE: R. Ramos, Auditory system Cellular/Molecular Thread

Inner hair cells release which of the following neurotransmitters?

A. GABA
B. Dopamine
C. Acetylcholine
D. Serotonin
E. Glutamate
SOURCE: R. Ramos, Auditory system Cellular/Molecular Thread

Which of the following structures project to the medial geniculate nucleus?

A. Inferior colliculus
B. Superior colliculus
C. Dorsal cochlear nucleus
D. Ventral cochlear nucleus
E. Superior olivary nucleus

SOURCE: R. Ramos, Auditory system Cellular/Molecular Thread

Which of the following is an example of sensorineural hearing loss?

A. Cerumen impaction
B. Otitis media
C. Middle ear barotraumas
D. Otosclerosis of the stapes
E. Presbycusis
Question #: 109
SOURCE: R. Ramos, Auditory system Cellular/Molecular Thread

Which anatomical feature found on the external ear is most commonly


Nervous the&cause
System for “protruding
Behavior Part II Examears” when A (Written)
#2- Version
misfolded?
10. Which anatomical feature found on the external ear is most
when misfolded?
A. Antihelix A. Antihelix
B. Concha
B. Concha
C. Helix C. Helix
D. Intertragic notch D. Intertragic notch
E. Tragus E. Tragus
SOURCE: Dr. Leheste, Auditory Powerpoint Lecture & Bar
SOURCE: Dr. Leheste, Auditory Powerpoint Lecture & Barr’s, Chapter Cellular/Molecular Thread
21 Cellular/Molecular Thread
11. Which cellular structure labeled in the hematoxyin/eosin-sta
Which cellular structure labeled in the hematoxyin/eosin-stained section of the human inner ear produces
produces endolymph?
endolymph?

A. 1

B. 2

C. 3

D. 4

E. 5


SOURCE: Dr. Leheste, Auditory Powerpoint Lecture & Barr’s,


Chapter 21 Cellular/Molecular Thread A. 1
B. 2
Which compartments/structures in the auditory system are filled with
C. 3 perilymph?
D. 4
E. 5
A. Internal acoustic meatus and pharyngotympanic tube
B. Middle ear and eustachian tube SOURCE: Dr. Leheste, Auditory Powerpoint Lecture & Bar
Cellular/Molecular Thread
C. Scala media and cochlear duct
D. Scala typmani and scala vestibuli 12. Which compartments/structures in the auditory system are f
E. Tympanic cavity and cochlear duct A. Internal acoustic meatus and pharyngotympanic tube
B. Middle ear and eustachian tube
SOURCE: Dr. Leheste, Auditory Powerpoint Lecture & Barr’s, Chapter C.
21 Cellular/Molecular Thread duct
Scala media and cochlear
D. Scala typmani and scala vestibuli
What is the function of the outer hair cells in the inner ear? E. Tympanic cavity and cochlear duct
SOURCE: Dr. Leheste, Auditory Powerpoint Lecture & Bar
Cellular/Molecular Thread
A. Alteration of excitation threshold
B. Determination of the direction of sound 13. What is the function of the outer hair cells in the inner ear?
C. Frequency processing A. Alteration of excitation threshold
D. Generation of an electrical auditory signal going to the auditoryB.
association cortex of the direction of sound
Determination
E. Production of endolymph C. Frequency processing
D. Generation of an electrical auditory signal going to the
Production of endolymph
SOURCE: Dr. Leheste, Auditory Powerpoint Lecture & Barr’s, Chapter E.
21 Cellular/Molecular Thread
SOURCE: Dr. Leheste, Auditory Powerpoint Lecture & Bar
Cellular/Molecular Thread
In high tone deafness, which cell type(s) is (are) most likely compromised?

A. Inner and outer hair cells at the apex of the organ of Corti
B. Inner hair cells at the apex of the organ of Corti
C. Inner hair cells at the base of the organ of Corti
4
D. Outer hair cells at the apex of the organ of Corti
E. Outer hair cells at the base of the organ of Corti

SOURCE: Dr. Leheste, Auditory Powerpoint Lecture & Barr’s, Chapter 21 Cellular/Molecular Thread
Olivocochlear feedback neurons involved in sensitivity regulation of the organ of Corti:

A. Are inhibitory
Question #: 110 to inner hair cells and excitatory to outer hair cells
B. Are inhibitory to outer hair cells and excitatory to inner hair cells
C. Can be either inhibitory or excitatory on inner and outer hair cells
D. Have an excitatory effect on inner and outer hair cells
E. Have an inhibitory effect on inner and outer hair cells

SOURCE: Dr. Leheste, Auditory Powerpoint Lecture & Barr’s, Chapter 21 Cellular/Molecular Thread

A 45-year-old male who has worked for many years as tractor operator on a farm has suffered persistent
exposure to loud sound caused by farm equipment. He is presenting to his physician with a dramatically reduced
hearing ability which appears to be limited to a high frequency range. What is the most likely cause for his
ailment?

A. Blockage of sound due to protective ear wax buildup


B. Degenerative damage to the base of the cochlea
C. Degenerative damage to the cochlear apex
D. Osteodegenerative arthritis of the ossicles
E. Sound-induced misalignment of the ossicles

SOURCE: Dr. Leheste, Powerpoint Lecture on 02.07.2011 & Barr’s, The Human Nervous System, 9th ed. 2009, p.
323/324
 Cellular/Molecular Thread

The malleus, one of three ossicles located in the middle ear, functions in the conduction of sound. Which of the
following structures listed is in physical contact with the malleus?

A. Fenestra cochleae
B. Fenestra vestibuli
C. Stapes
D. Tympanic membrane
E. Vestibular membrane

SOURCE: Dr. Leheste, Powerpoint Lecture on 02.07.2011 & Barr’s, The Human Nervous System, 9th ed. 2009, p.
319/320
 Cellular/Molecular Thread

Which of the following anatomical feature enables the organ of Corti to process different incoming sound
frequencies at the same time?

A. Variable ion composition of the perilymph


B. Variable number inner hair cell rows
C. Variable number of outer hair cell rows
D. Variable thickness of Reissner’s membrane
E. Variable width of the basilar membrane

SOURCE: Dr. Leheste, Powerpoint Lecture on 02.07.2011 & Barr’s, The Human Nervous System, 9th ed. 2009, p.
324/325
 Cellular/Molecular Thread

The so-called acoustic reflex regulates the contraction of the tensor tympani and stapedius muscle. What is the
primary physiological consequence of this interaction?

A. Facilitation of a diversified processing of pitch and timbre


B. Facilitation of reflex turning of head and eyes
C. Prevention of damage to inner and outer hair cells
D. Refinement of sound by enhancing specific frequencies
E. Release of excess fluids from the inner and/or middle ear
A. 1
B. 2
C. 3
D. 4
SOURCE: Dr. Leheste, Powerpoint Lecture on 02.07.2011 & Barr’s, The Human Nervous System,
E. 1 plus 2 9th ed. 2009, p.
SOURCE: Dr. Hallas
320&329
 Cellular/Molecular Thread Cellular/Molecular Thread
Question #: 111
98. What
What is the primary function of this cochlear duct structure in the inner ear that the isarrow
the primary function of
is pointing tothis cochlear duct structur
(human,
(human, histological preparation, stained with hematoxyli
histological preparation, stained with hematoxylin/eosin)?

A. Articulation with outer hair cell microvilli


B. Conduction of sound to the cochlear ganglion
C. Production of endolymph
D. Production of perilymph
E. Propagation of pressure waves

SOURCE: Dr. Leheste, Powerpoint Lecture on 02.07.2011 & Barr’s, The Human
Nervous System, 9th ed. 2009, p. 323-325
 Cellular/Molecular Thread A. Articulation with outer hair cell microvilli
B. Conduction of sound to the cochlear ganglion
C. Production of endolymph
D. Production of perilymph
E. Propagation of pressure waves
SOURCE: Dr. Leheste, Powerpoint Lecture on 02.07.2011
ed. 2009, p. 323-325
Cellular/Molecular Thread

NEUROANATOMY LAB
Which arrow points to the anterior commissure? (Please see attachment)
A. A
B. B
C. C
D. D
E. E
Rationale:
a. Correct. Anterior commissure

b. Incorrect; however, credit is being given due to this material not yet 24

having been covered in the course. (Interthalamic adhesion)



c. Incorrect; however, credit is being given due to this material not yet
having been covered in the course. (Pineal gland)

d. Incorrect; however, credit is being given due to this material not yet
having been covered in the course. (Mammillary body)

e. Incorrect; however, credit is being given due to this material not yet
having been covered in the course. (Tectum)


Item Description: Neuroscience Lab Question


Which arrow points to the tectum? (Please see attachment)
A. A
B. B
C. C
D. D
E. E
Rationale:
a. Incorrect. Anterior commissure
b. Incorrect. Interthalamic adhesion
c. Incorrect. Pineal gland

d. Incorrect. Mammillary body

e. Correct. Tectum


Item Description: Neuroscience Lab Question

Which arrow points to the post-central gyrus? (Please see


attachment)
A. A
B. Question
B #: 112
C. C
D. D
E. E
Rationale:
a. Incorrect. Precentral gyrus
b. Correct. Post-central gyrus
c. Incorrect. Supramarginal gyrus
d. Incorrect. Inferior frontal gyrus
e. Incorrect Superior temporal gyrus
Item Description: Neuroscience Lab Question

Which arrow points to the primary fissure? (Please see


attachment)
A. A
B. B ✓C. C
D. D
E. E
Rationale:
a- Incorrect. b- Incorrect. c- Correct. d- Incorrect. e- Incorrect.
Attachment:
Vermis
Horizontal fissure Primary fissure
Superior colliculus Cerebral aqueduct

Item Description: Neuroscience Lab Question


Question #: 85
 Which arrow points to the vermis? (Please see attachment)
✓A. A B. B C. C D. D E. E
Rationale:
a- Correct. Vermis
 b- Incorrect. Horizontal fissure c- Incorrect. Primary fissure
 d- Incorrect. Superior
colliculus e- Incorrect. Cerebral aqueduct Attachment:
Question #: 113

Item Description: Neuroscience Lab Question


Question #: 86
 Which arrow points to the inferior frontal gyrus? (Please see attachment)
A. A B. B C. C
✓D. D E. E
Rationale:
a- Incorrect. Precentral gyrus
 b- Incorrect. Post-central gyrus
 c- Incorrect. Supramarginal gyrus
 d- Correct.
Inferior frontal gyrus
 e- Incorrect. Superior temporal gyrus Attachment:
Question #: 114

Item Description: Neuroscience Lab Question


Question #: 87
 Which arrow points to the olive? (Please see attachment)
A. A B. B C. C D. D
✓E. E Rationale:
a- Incorrect. Optic chiasm
 b- Incorrect. Pons
 c- Incorrect. Middle cerebellar peduncle d- Incorrect.
Pyramid
 e- Correct. Olive
 Attachment:
Question #: 115

Item Description: Neuroscience Lab Question


Question #: 88
 Which arrow points to the middle cerebellar peduncle? (Please see attachment)
A. A
B. B ✓C. C
D. D
E. E
Rationale:
a- Incorrect. Optic chiasm
 b- Incorrect. Pons
 c- Correct. Middle cerebellar peduncle d- Incorrect. Pyramid
 e-
Incorrect. Olive
 Attachment:
Question #: 116

Item Description: Neuroscience Lab Question


Question #: 89
 Which arrow points to the optic chiasm? (Please see attachment)
✓A. A B. B C. C D. D E. E
Rationale:
a- Correct. Optic chiasm
 b- Incorrect. Pons
 c- Incorrect. Middle cerebellar peduncle d- Incorrect. Pyramid
 e-
Question
Incorrect. #: 117Attachment:
Olive


Item Description: Neuroscience Lab Question


Question #: 90
 Which arrow points to a dorsal root ganglion? (Please see attachment)
A. A B. B C. C
✓D. D E. E
Question #: 118
Rationale:
a- Incorrect. Dorsal median sulcus b- Incorrect. Gracile fasciculus
 c- Incorrect. Cuneate fasciculus
 d-
Correct. Dorsal root ganglion
e- Incorrect. Ventral root
Attachment:

Item Description: Neuroscience Lab Question


Question #: 91
 Which arrow points to the cuneate fasciculus? (Please see attachment)
A. A
B. B ✓C. C
D. D
E. E
Rationale:
a- Incorrect. Dorsal median sulcus b- Incorrect. Gracile fasciculus
 c- Correct. Cunate fasciculus
 d- Incorrect.
Dorsal root ganglion e- Incorrect. Ventral root Attachment:
Question #: 119

Item Description: Neuroscience Lab Question


Question #: 92
 Which arrow points to the pineal gland? (Please see attachment)
✓A. A
✓B. B ✓C. C
✓D. D
✓E. E Rationale:
a- Incorrect; however, credit is being given due to this material not yet having been covered in the course.
(Anterior commissure)
 b- Incorrect; however, credit is being given due to this material not yet having been
covered in the course. (Interthalamic adhesion)
c- Correct. Pineal gland
 d- Incorrect; however, credit is being given due to this material not yet having been
covered in the course. (Mammillary body)
 e- Incorrect; however, credit is being given due to this material not
yet having been covered in the course.(Tectum)
 Attachment:
Item Description: Neuroscience Lab Question
Question #: 120

Question #: 93
 Which arrow points to the thalamus? (Please see attachment)
A. A
B. B ✓C. C
D. D
E. E
Rationale:
a- Incorrect. Septum pellucidum b- Incorrect. Corpus callosum c- Correct. Thalamus
 d- Incorrect.
Hypothalamus
e- Incorrect. 4th Ventricle
Attachment:
Question #: 121

Item Description: Neuroscience Lab Question


Question #: 94
 Which arrow points to the septum pellucidum? (Please see attachment)
✓A. A ✓B. B ✓C. C ✓D. D ✓E. E
Rationale:
a- Correct. Septum pellucidum
 b- Incorrect; however, credit is being given due to this material not yet having
been covered in the course. (Corpus callosum)
 c- Incorrect; however, credit is being given due to this material
not yet having been covered in the course. (Thalamus)
 d- Incorrect; however, credit is being given due to this
material not yet having been covered in the course. (Hypothalamus)
 e- Incorrect; however, credit is being
given due to this material not yet having been covered in the course. (4th Ventricle)
 Attachment:
Item Description: Neuroscience Lab Question
Question #: 122

Question #: 95
 Which arrow points to the cerebellum? (Please see attachment)
A. A B. B C. C D. D
✓E. E Rationale:
a- Incorrect. Frontal lobe b- Incorrect. Parietal lobe c- Incorrect. Occipital lobe d- Incorrect. Temporal lobe e-
Correct. Cerebellum Attachment:
Question #: 123

Item Description: Neuroscience Lab Question


Question #: 96
 Which arrow points to the frontal lobe? (Please see attachment)
✓A. A B. B C. C D. D E. E
Rationale:
a- Correct. Frontal lobe
 b- Incorrect. Parietal lobe c- Incorrect. Occipital lobe d- Incorrect. Temporal lobe e-
Incorrect. Cerebellum Attachment:
Question #: 124

Item Description: Neuroscience Lab Question


Question #: 97
 Which arrow points to the parietal lobe? (Please see attachment)
A. A ✓B. B
C. C D. D E. E
Rationale:
a- Incorrect. Frontal lobe b- Correct. Parietal lobe
 c- Incorrect. Occipital lobe d- Incorrect. Temporal lobe e-
Incorrect. Cerebellum Attachment:
Question #: 125

Item Description: Neuroscience Lab Question

Question #: 85
Identify the structure of arrow 1: (Please see attached) ✓A. Falx cerebri
B. Tentorium cerebelli
 C. Septum pellucidum
 D. Superior sagittal sinus E. Arachnoid villi
Attachment:
Question #: 126

SOURCE: Title: SOURCE: Dr. Hallas, Neuroscience Lab #1 Cellular/Molecular Thread


Question #: 86
Identify the structure of arrow 2: (Please see attached) A. Falx cerebri
✓B. Tentorium cerebelli C. Septum pellucidum
 D. Superior sagittal sinus E. Arachnoid villi
Attachment:
Question #: 127
SOURCE: Title: SOURCE: Dr. Hallas, Neuroscience Lab #1 Cellular/Molecular Thread
Question #: 87
 Identify the structure of arrow 3: (Please see attached)
Questionpole
A. Frontal #: 128
B. Temporal pole ✓C. Occipital pole
D. Parietal pole
 E. Precentral gyrus
Attachment:

SOURCE: Title: SOURCE: Dr. Hallas, Neuroscience Lab #1 Cellular/Molecular Thread


Question #: 88
Identify the structure of arrow 4: (Please see attached) A. Post central gyrus
✓B. Precentral gyrus C. Central sulcus
 D. Lateral sulcus
 E. Longitudinal fissure
Attachment:
Question #: 129

SOURCE: SOURCE: Dr. Hallas, Neuroscience Lab #1 Cellular/Molecular Thread


Question #: 89
Identify the structure of arrow 5: (Please see attached) ✓A. Pons
B. Medulla
 C. Midbrain
 D. Thalamus
 E. Hypothalamus
Attachment:
Question #: 130
SOURCE: Title: SOURCE: Dr. Hallas, Neuroscience Lab #1 Cellular/Molecular Thread
Question #: 90
 Identify the structure of arrow 6: (Please see attached)
A. Pons ✓B.#:Medulla
Question 131
C. Midbrain
 D. Thalamus
 E. Hypothalamus
Attachment:
Question #: 132
SOURCE: SOURCE: Dr. Hallas, Neuroscience Lab #1 Cellular/Molecular Thread
Question #: 91
Question
Identify #: 133 of arrow 7: (Please see attached) A. Pons
the structure
B. Medulla C. Midbrain D. Thalamus
✓E. Hypothalamus Attachment:

SOURCE: SOURCE: Dr. Hallas, Neuroscience Lab #1 Cellular/Molecular Thread


Question #: 92
Identify the structure of arrow 8: (Please see attached) A. Pons
B. Medulla
C. Midbrain ✓D. Thalamus
E. Hypothalamus
Attachment:
Question #: 134

SOURCE: SOURCE: Dr. Hallas, Neuroscience Lab #1 Cellular/Molecular Thread


Question #: 93
Identify the structure of arrow 9: (Please see attached) A. Pons
B. Medulla ✓C. Midbrain
D. Thalamus
 E. Hypothalamus
Attachment:
Question #: 135

SOURCE: SOURCE: Dr. Hallas, Neuroscience Lab #1 Cellular/Molecular Thread

1. IDENTIFY:
A. Cuneate tubercle
B. Dorsal roots
C. Fasciculus cuneatus
Question #:gracilis
D.Fasciculus 136
E. Gracile tubercle
SOURCE: Neuroscience Laboratory
2. IDENTIFY:
A. Cuneate tubercle
B. Dorsal roots
C.Fasciculus cuneatus
D. Fasciculus gracilis
E. Gracile tubercle
SOURCE: Neuroscience Laboratory
1
Nervous System & Behavior Part I Exam #1- Version A (Written)
January 17, 2012

3. IDENTIFY:
A. Cuneate tubercle
B. Dorsal roots
C. Fasciculus cuneatus
D.Fasciculus gracilis
E. Gracile tubercle
SOURCE: Neuroscience Laboratory
4. IDENTIFY:
A. Dorsal roots
B. Fasciculus cuneatus
C.Fasciculus gracilis
D.Gracile tubercle
E. V entral roots
SOURCE: Neuroscience Laboratory
2
Nervous System & Behavior Part I Exam #1- Version A (Written)
January 17, 2012
Question #: 137

5. IDENTIFY:
A. Arachnoid granulations
B. Falx cerebri
C.Filum terminale
D.Superior sagittal sinus
E. Tentorium cerebelli
SOURCE: Neuroscience Laboratory
6. IDENTIFY:
A. Arachnoid granulations
B. Falx cerebri
C.Filum terminale
D.Superior sagittal sinus
E. Tentorium cerebelli
SOURCE: Neuroscience Laboratory
7. IDENTIFY:
A. Inferior colliculus
B. Medulla
C. Pons
D. Superior colliculus
E. Thalamus
SOURCE: Neuroscience Laboratory
3
Nervous System & Behavior Part I Exam #1- Version A (Written)
January 17, 2012
Question #: 138

8. IDENTIFY:
A. Cauda equina
B. Conus medullaris
C.Denticulate ligament
D.Filum terminale
E. Lumbosacral enlargement
SOURCE: Neuroscience Laboratory
9. IDENTIFY:
A. Cauda equina
B. Conus medullaris
C.Denticulate ligament
D.Filum terminale
E. Lumbosacral enlargement
SOURCE: Neuroscience Laboratory
4
Nervous System & Behavior Part I Exam #1- Version A (Written)
Question
January 17, #: 139
2012

10. IDENTIFY:
A. CNI
B. CNII
C.CN III
D.CNIV
E. Olfactory tract
SOURCE: Neuroscience Laboratory
11. IDENTIFY:
A. CNI
B. CNII
C.CN III
D.CNIV
E. Olfactory tract
SOURCE: Neuroscience Laboratory
12. IDENTIFY:
A. CNI
B. CNII
C.CN III
D.CNIV
E. CNV
SOURCE: Neuroscience Laboratory
5
Nervous System & Behavior Part I Exam #1- Version A (Written)
January 17, 2012
Question #: 140

13. IDENTIFY:
A. CNIV
B. CNV
C.CNVI
D.CN VII
E. Striae medullaris
SOURCE: Neuroscience Laboratory
14. IDENTIFY:
A. Inferior cerebellar peduncle
B. Inferior medullary velum
C.Middle cerebellar peduncle
D. Superior cerebellar peduncle
E. Superior medullary velum
SOURCE: Neuroscience Laboratory
15. IDENTIFY:
A. Inferior cerebellar peduncle
B. Inferior medullary velum
C.Middle cerebellar peduncle
D.Superior cerebellar peduncle
E. Superior medullary velum
SOURCE: Neuroscience Laboratory
16. IDENTIFY:
A. Facial colliculus
B. Hypoglossal trigone
C.Striae medullares
D.V agal trigone
E. V estibular area
SOURCE: Neuroscience Laboratory
6
Nervous System & Behavior Part I Exam #1- Version A (Written)
January 17, 2012
Question #: 141

17. IDENTIFY (space):


A. Fourth ventricle
B. Interventricular foramen
C.Lateral ventricle
D.Subdural space
E. Third ventricle
SOURCE: Neuroscience Laboratory
18. IDENTIFY (space):
A. Cerebral aqueduct
B. Fourth ventricle
C.Interventricular foramen
D.Lateral ventricle
E. Third ventricle
SOURCE: Neuroscience Laboratory
19. IDENTIFY: A. Flocculus
B. Lingula
 C. Nodulus
 D. Tonsil
 E. Uvula
 SOURCE: Neuroscience Laboratory
20. IDENTIFY:
A. Inferior medullary velum
B. Middle cerebellar peduncle
C.Striae medullares acoustica
D. Superior cerebellar peduncle
E. Superior medullary velum

7
Nervous System & Behavior Part I Exam #1- Version A (Written) SOURCE: Neuroscience Laboratory
Question #: 142

21. IDENTIFY:
A. Inferior colliculus
B. Pineal
C.Pulvinar
D. Superior colliculus
E. Superior medullary velum
SOURCE: Neuroscience Laboratory
22. IDENTIFY:
A. Brachium of inferior colliculus
B. Inferior cerebellar peduncle
C.Median eminence
D.Superior cerebellar peduncle
E. Superior medullary velum
SOURCE: Neuroscience Laboratory January 17, 2012.

Which arrow points to the primary fissure? (Please see attachment)


A. A
B. B
C. C
D. D
E. E
Rationale:
a. Incorrect. Vermis
b. Question
Incorrect.
#: 143Horizontal fissure
c. Correct. Primary fissure
d. Incorrect. Superior colliculus
e. Incorrect. Cerebral aqueduct
Item Description: Neuroscience Lab Question
Which arrow points to the vermis? (Please see attachment)
A. A
B. B
C. C
D. D
E. E
Rationale:
a. Correct. Vermis

b. Incorrect. Horizontal fissure
c. Incorrect. Primary fissure

d. Incorrect. Superior colliculus
e. Incorrect. Cerebral aqueduct

Item Description: Neuroscience Lab Question


Which arrow points to the olive? (Please see attachment)
A. A
B. B
C. C
D. D
E. E
Rationale:
a. Incorrect. Optic chiasm

b. Incorrect. Pons

c. Incorrect. Middle cerebellar peduncle
d. Incorrect. Pyramid

e. Correct. Olive


Item Description: Neuroscience Lab Question


Which arrow points to the middle cerebellar peduncle? (Please see attachment)
A. A
B. B
C. C
D. D
E. E
Rationale:
a. Incorrect. Optic chiasm

b. Incorrect. Pons

c. Correct. Middle cerebellar peduncle
d. Incorrect. Pyramid

e. Incorrect. Olive


Item Description: Neuroscience Lab Question


Which arrow points to the optic chiasm? (Please see attachment)
A. A
B. B
C. C
D. D
E. E
Rationale:
a. Correct. Optic chiasm

b. Incorrect. Pons

c. Incorrect. Middle cerebellar peduncle
d. Incorrect. Pyramid

e. Incorrect. Olive

Item Description: Neuroscience Lab Question
Which arrow points to a dorsal root ganglion? (Please see attachment)
A. Question
A #: 144
B. B
C. C
D. D
E. E
Rationale:
a. Incorrect. Dorsal median sulcus
b. Incorrect. Gracile fasciculus

c. Incorrect. Cuneate fasciculus

d. Correct. Dorsal root ganglion
e. Incorrect. Ventral root
Item Description: Neuroscience Lab Question
Which arrow points to the cuneate fasciculus? (Please see attachment)
A. A
B. B
C. C
D. D
E. E
Rationale:
a. Incorrect. Dorsal median sulcus
b. Incorrect. Gracile fasciculus

c. Correct. Cunate fasciculus

d. Incorrect. Dorsal root ganglion
e. Incorrect. Ventral root

Item Description: Neuroscience Lab Question

1. IDENTIFY:
A. Cuneate tubercle
B. Dorsal roots
C. Fasciculus cuneatus
D. Fasciculus gracilis
E. Gracile tubercle
SOURCE: Neuroscience Laboratory
2. IDENTIFY:
A. Cuneate tubercle
B. Dorsal roots
C. Fasciculus cuneatus
D. Fasciculus gracilis
E. Gracile tubercle
SOURCE: Neuroscience Laboratory

3. IDENTIFY:
A. Cuneate tubercle
B. Dorsal roots
C. Fasciculus cuneatus
D. Fasciculus gracilis
E. Gracile tubercle
SOURCE: Neuroscience Laboratory
4. IDENTIFY:
A. Dorsal roots
Question #: 145
B. Fasciculus cuneatus
C. Fasciculus gracilis
D. Gracile tubercle
E. Ventral roots
SOURCE: Neuroscience Laboratory

8. IDENTIFY:
A. Cauda equina
B. Conus medullaris
C. Denticulate ligament
D. Filum terminale
E. Lumbosacral enlargement
SOURCE: Neuroscience Laboratory
9. IDENTIFY:
A. Cauda equina
B. Conus medullaris
C. Denticulate ligament
D. Filum terminale
E. Lumbosacral enlargement
SOURCE: Neuroscience Laboratory

10. IDENTIFY:
A. CNI
B. CNII
C. CN III
D. CNIV
E. Olfactory tract
11. IDENTIFY:
A. CNI
B. CNII
C. CN III
D. CNIV
E. Olfactory tract
12. IDENTIFY:
A. CNI
B. CNII
C. CN III
D. CNIV
E. CNV
13. IDENTIFY:
A. CNIV
B. CNV
C. CNVI
D. CN VII
E. Striae medullaris
14. IDENTIFY:
A. Inferior cerebellar peduncle
B. Inferior medullary velum
C. Middle cerebellar peduncle
D. Superior cerebellar peduncle
E. Superior medullary velum
15. IDENTIFY:
A. Inferior cerebellar peduncle
B. Inferior medullary velum
C. Middle cerebellar peduncle
D. Superior cerebellar peduncle
E. Superior
Questionmedullary
#: 146 velum
16. IDENTIFY:
A. Facial colliculus
B. Hypoglossal trigone
C. Striae medullares
D. Vagal trigone
E. Vestibular area
21. IDENTIFY:
A. Inferior colliculus
B. Pineal
C. Pulvinar
D. Superior colliculus
E. Superior medullary velum
22. IDENTIFY:
A. Brachium of inferior colliculus
B. Inferior cerebellar peduncle
C. Median eminence
D. Superior cerebellar peduncle
E. Superior medullary velum


 Identify the structure the arrow is pointing to: (Please see attachment)
A. Conus medullaris
B. Filum terminale
C. Denticulate ligament
D. Cauda equina

E. Dorsal root
SOURCE: Dr. Hallas, Neuroscience Lab #2 and 3 Cellular/Molecular Thread

Identify the structure the arrow is pointing to: (Please see attachment)
A. CN 2
B. CN 3
C. CN 4
D. CN 5
E. CN 6

SOURCE: Dr. Hallas, Neuroscience Lab #2 and 3 Cellular/Molecular Thread


 Identify the structure the arrow is pointing to:


A. Inferior colliculus
B. Superior colliculus

C. Inferior cerebellar peduncle
D. Middle cerebellar peduncle
E. Superior cerebellar peduncle


 Identify the structure the arrow is pointing to: (Please see attachment)
A. ✓A. Facial nerve

B. Abducens nerve
C. Question #: 147 nerve

Oculomotor
D. Trochlear nerve

E. Vestibulocochlear nerve


 Identify the structure the arrow is pointing to: (Please see attachment)
A. CN 4
B. CN 5
C. CN 6
D. CN 7
E. CN 8

3. IDENTIFY:
A. CN VII
B. CN VIII
C. CNIX
D. CNX
E. CN XII
4. IDENTIFY:
A. CN VII
B. CN VIII
C. CNIX
D. CNXI
E. CN XII

1. IDENTIFY:
A. Fasciculus gracilis
B. Cuneate tubercle
C. Gracile tubercle
D. Fasciculus cuneatus
E. Dorsal roots
2. IDENTIFY:

A. Fasciculus gracilis
B. Cuneate tubercle

C. Gracile tubercle

D. Fasciculus cuneatus
E. Dorsal roots
3. The DRG labeled 3 is associated with
which spinal cord level?
A. C1

B. C2

C. C3
D. C4

E. C5
4. IDENTIFY:

A. Ventral roots

B. Question
Fasciculus
#: 148cuneatus

C. Dorsal root ganglia
D. D. Cuneate tubercle

E. Dorsal roots
9. IDENTIFY:
A. CN II
B. CN III
C. CN IV
D. CN V
E. CN VI
10. IDENTIFY:
A. CN III

B. CN IV

C. CN V
D. CN VI
E. CN VII

11. IDENTIFY:
A. CN VI

B. CN VII

C. CN VIII
D. CN IX
E. CN X
12. IDENTIFY:
A. CN VIII

B. CN IX

C. CN X
D. CN XI

E. CN XII
F.

13. IDENTIFY:
A. Lumbosacral enlargement
B. Conus medullaris
C. Filum terminale
D. Cauda equina
E. Denticulateligament
14. IDENTIFY:
A. Lumbosacral enlargement
B. Conusmedullaris
C. Filum terminale
D. Cauda equina
E. Denticulateligament

19. IDENTIFY:

A. Pineal

B. Pulvinar
C. Superior colliculus
D. Inferior colliculus
E. Superior medullary velum
20. IDENTIFY:
A. CN I

B. CN II

C. CN III
D. CN IV
E. CN V
21. IDENTIFY:

A. Question
Superior cerebellar peduncle
#: 149
B. Inferior cerebellar peduncle

C. Brachium of inferior colliculus
D. Superior medullary velum
E. Median eminence
22. IDENTIFY:

A. Superior cerebellar peduncle
B. Inferior cerebellar peduncle

C. Brachium of inferior colliculus
D. Superior medullary velum

E. Middle cerebellar peduncle
23. IDENTIFY:
A. Fasciculus gracilis
B. Cuneate tubercle
C. Graciletubercle
D. Fasciculus cuneatus
E. Dorsal roots .

Which arrow points to the vestibular nucleus? (Please see attachment)

A. A
B. B
C. C
D. D
E. E

Rationale:

a. Correct. Vestibular nucleus


b. Incorrect. Medial longitudinal fasciculus
c. Incorrect. Cochlear nucleus
d. Incorrect. Medial lemniscus
e. Incorrect. Corticospinal tract

Which arrow points to the red nucleus? (Please see attachment)


A. A
B. B
C. C
D. D
E. E
Rationale:
a. Incorrect. Superior brachium

b. Incorrect. Periaqueductal grey matter
c. Correct. Red nucleus

d. Incorrect. Substantia nigra

e. Incorrect. Corticospinal tract

Which arrow points to the inferior colliculus? (Please see attachment)


A. A
B. B
C. C
D. D
E. E
Rationale:
a. Correct. Inferior colliculus

b. Incorrect. Medial longitudinal fasciculus
c. Incorrect. Superior cerebellar peduncle
d. Question
Incorrect.
#: 150Medial lemniscus

e. Incorrect. Corticospinal tract
Which arrow points to the corticospinal tract?
A. A
B. B
C. C
D. D
E. E
Rationale:
a. Incorrect. Superior cerebellar peduncle
b. Incorrect. Medial longitudinal fasciculus
c. Incorrect. Central tegmental tract

d. Incorrect. Medial lemniscus
e. Correct. Corticospinal tract

Which arrow points to the superior cerebellar peduncle?


A. A
B. B
C. C
D. D
E. E
Rationale:
a. Correct. Superior cerebellar peduncle
b. Incorrect. Medial longitudinal fasciculus
c. Incorrect Middle cerebellar peduncle

d. Incorrect. Medial lemniscus

e. Incorrect. Corticospinal tract

Which arrow points to the facial nucleus?


A. A
B. B
C. C
D. D
E. E
Rationale:
a. Incorrect. Medial longitudinal fasciculus
b. Incorrect. Abducens nucleus

c. Correct. Facial nucleus

d. Incorrect. Medial lemniscus
e. Incorrect. Corticospinal tract
Which arrow points to the medial longitudinal fasciculus?
A. A
B. B
C. C
D. D
E. E
Rationale:
a. Incorrect. Vestibular nucleus
b. Correct. Medial longitudinal fasciculus
c. Incorrect. Inferior cerebellar peduncle
d. Incorrect. Medial lemniscus
e. Incorrect. Corticospinal tract

Which arrow points to the trigeminal nucleus?


A. A
B. B
C. C
D. D
E. Question
E #: 151
Rationale:
a. Incorrect. Vestibular nucleus
b. Incorrect. Hypoglossal nucleus
c. Correct. Trigeminal nucleus
d. Incorrect. Medial lemniscus
e. Incorrect. Corticospinal tract

Which arrow points to the spinal accessory nucleus?


A. A
B. B
C. C
D. D
E. E
Rationale:
a. Incorrect. Gracile nucleus

b. Incorrect. Cuneate fasciculus
c. Incorrect. Trigeminal nucleus
d. Correct. Accesory nucleus

e. Incorrect. Corticospinal tract
Which arrow points to the corticospinal tract?
A. A
B. B
C. C
D. D
E. E
Rationale:
a. Incorrect. Gracile fasciculus
b. Incorrect. Cuneate fasciculus
c. Incorrect. Trigeminal nucleus
d. Incorrect. Accesory nucleus
e. Correct. Corticospinal tract

Identify the structure the arrow is pointing to:


A. Hypoglossal nucleus

B. Nucleus of the vagus nerve
C. Spinal trigeminal nucleus
D. Mesencephalic nucleus

E. Nucleus cuneatus

Identify the structure the arrow is pointing to


A. Lateral vestibular nucleus
B. Nucleus ambiguous

C. Oculomotor nucleus

D. Trochlear nucleus
E. Cochlear nucleus

Identify the structure the arrow is pointing to:


A. Inferior olivary nucleus
B. Superior olivary nucleus
C. Question
Deep#:cerebellar
152 nucleus
D. Lateral vestibular nucleus
E. Inferior cerebellar peduncle

Identify the structure the arrow is pointing to:


A. Oculomotor nucleus
B. Vagal nucleus

C. Lateral vestibular nucleus
D. Facial nucleus

E. Superior cerebellar peduncle

Identify the structure the arrow is pointing to:


A. Lateral lemniscus

B. Oculomotor nerve

C. Brachium of the inferior colliculus
D. Brachium of the superior colliculus
E. Trochlear nerve

14. IDENTIFY:
A. Cochlear Nucleus
B. Hypoglossal Nucleus
C. Solitary Nucleus
D. Spinal Trigeminal Nucleus
E. Vagus Nucleus

15. IDENTIFY:
A. Cerebral Aqueduct
B. Fourth Ventricle
C. Interventricular Foramen
D. Periaqueductal Gray
E. Third Ventricle
16. IDENTIFY:
A. Inferior Cerebellar Peduncle
B. Lateral Lemniscus
C. Middle Cerebellar Peduncle
D. Superior Cerebellar Peduncle
E. Superior Medullary Velum
Question #: 153

17. IDENTIFY:
A. Brachium of Inferior Colliculus
B. Lateral Lemniscus
C. Medial Lemniscus
D. Medial Longitudinal Fasciculus
E. Superior Cerebellar Peduncle
18. IDENTIFY:
A. Brachium of Inferior Colliculus
B. Brachium of Superior Colliculus
C. Inferior Colliculus
D. Pulvinar
E. Superior Colliculus
20. IDENTIFY:
A. Inferior Colliculus
B. Red Nucleus
C. Substantia Nigra
D. Superior Cerebellar Peduncle
E. Superior Colliculus

24. IDENTIFY:

A. Nucleus gracilis
B. Nucleus cuneatus
C. Fasciculus gracilis
D. Fasciculus cuneatus
E. Pyramidal tract
25. IDENTIFY:

A. Nucleus gracilis

B. Nucleus cuneatus

C. Hypoglossal nucleus
D. Fasciculus cuneatus
E. Central gray
26. IDENTIFY:

A. Vagus nucleus

B. Cochlear nucleus

C. Hypoglossal nucleus

D. Solitary nucleus

E. Spinal trigeminal nucleus

27. IDENTIFY:
A. Fourth ventricle

B. Third ventricle

C. Cerebral aqueduct
D. Interventricular foramen

E. Periaqueductal gray

28. IDENTIFY:

A. Inferior cerebellar peduncle

B. Middle cerebellar peduncle

C. Superior cerebellar peduncle
D. Superior medullary velum

E. Lateral lemniscus
G.
29. IDENTIFY:

A. Lateral lemniscus

B. Medial lemniscus

C. Question
Medial#: longitudinal
154 fasciculus
D. Superior cerebellar peduncle

E. Brachium of inferior colliculus
30. IDENTIFY:

A. Inferior colliculus
B. Superior colliculus

C. Brachium of inferior colliculus
D. Brachium of superior colliculus
E. Pulvinar

Which arrow points to the middle frontal gyrus?


A. A
B. B
C. C
D. D
E. E
Rationale:
a. Correct. Middle frontal gyrus

b. Incorrect. Inferior parietal lobule
c. Incorrect. Supramarginal gyrus
d. Incorrect. Angular gyrus

e. Incorrect. Middle temporal gyrus

Which arrow points to the long gyrus of the insula?


A. A
B. B
C. C
D. D
E. E
Rationale:
a. Incorrect. Transverse temporal gyrus
b. Incorrect. Middle temporal gyrus

c. Correct. Insula long gyrus

d. Incorrect. Central sulcus of insula
e. Incorrect. Insula short gyri

Which arrow points to the genu of the corpus callosum?


A. A
B. B
C. C
D. D
E. E
Rationale:
a. Correct. Corpus callosum genu

b. Incorrect. Corpus callosum body

c. Incorrect. Corpus callosum splenium
d. Incorrect. Anterior commisure

e. Incorrect. Interthalamic adhesion
Which arrow points to the superior medullary velum?
A. A
B. B
C. C
D. D
E. E
Rationale:
Question #: 155
a. Incorrect. Cingulate gyrus

b. Incorrect. Septum pellucidum

c. Incorrect. Pineal gland

d. Incorrect. Calcarine sulcus

e. Correct. Superior medullary velum

Which arrow points to the cingulate gyrus?


A. A
B. B
C. C
D. D
E. E
Rationale:
a. Correct. Cingulate gyrus

b. Incorrect. Septum pellucidum

c. Incorrect. Pineal gland

d. Incorrect. Calcarine sulcus

e. Incorrect. Superior medullary velum

Which arrow points to the straight gyrus?


A. A
B. B
C. C
D. D
E. E
Rationale:
a. Incorrect. Lingual gyrus

b. Incorrect. Occipitotemporal gyrus
c. Incorrect. Uncus

d. Incorrect. Parahippocampal gyrus
e. Correct. Straight gyrus
Which arrow points to the parahippocampal gyrus?
A. A
B. B
C. C
D. D
E. E
Rationale:
a. Incorrect. Lingual gyrus

b. Incorrect. Occipitotemporal gyrus

c. Correct. The uncus is the anterior extreme
of the parahippocampal gyrus
d. Correct. Parahippocampal gyrus

e. Incorrect. Straight gyrus

Which arrow points to the central sulcus?


A. A
B. B
C. C
D. D
E. E
Rationale:
a. Incorrect. Precentral sulcus

b. Correct. Central sulcus

c. Incorrect. Post central sulcus

d. Incorrect. Intraparietal sulcus

e. Incorrect. Parietooccipital sulcus
Question #: 156
Which arrow points to the precentral sulcus?
A. A
B. B
C. C
D. D
E. E
Rationale:
a. Correct. Precentral sulcus

b. Incorrect. Central sulcus

c. Incorrect. Post central sulcus

d. Incorrect. Intraparietal sulcus

e. Incorrect. Parietooccipital sulcus
Which arrow points to the middle temporal gyrus?
A. A
B. B
C. C
D. D
E. E
Rationale:
a. Incorrect. Middle frontal gyrus
b. Incorrect. Inferior parietal lobule
c. Incorrect. Supramarginal gyrus
d. Incorrect. Angular gyrus

e. Correct. Middle temporal gyrus

Identify the structure the arrow is pointing to:


A. CN 2
B. CN 3
C. CN 4
D. CN 5
E. CN 6

Identify the structure the arrow is pointing to:


A. Superior parietal lobe
B. Angular gyrus

C. Supramarginal gyrus
D. Cuneus lobule
E. Lingula lobule

Identify the structure the arrow is pointing to:


A. Hypothalamus
B. Anterior commissure
C. Mammillary body
D. Posterior commissure
E. Crus cerebri
Question #: 157

Identify the structure the arrow is pointing to:


A. Lateral lemnicus
B. CN 3
C. CN 4
D. Brachium of the superior colliculus
E. Superior medullary velum

Identify the structure the arrow is pointing to


A. Uncus
B. Parahippocampal gyrus

C. Medial occipitotemporal gyrus
D. Lateral occipitotemporsal gyrus
E. Inferior temporal gyrus

Identify the structure the arrow is pointing to:


A. Superior parietal gyrus
B. Precuneus lobule
C. Cuneus lobule
D. Lingual lobule
E. Cingulate gyrus

1. IDENTIFY:
A. Inferior Frontal Gyrus
B. Middle Frontal Gyrus
C. Postcentral Gyrus
D. Precentral Gyrus
E. Superior Parietal Lobule
2. IDENTIFY:
A. Inferior Parietal Lobule
B. Inferior Temporal Gyrus
C. Middle Temporal Gyrus
D. Superior Marginal Gyrus
E. Superior Temporal Gyrus
Question #: 158
5. IDENTIFY:
A. Cingulate Gyrus
B. Cuneus
C. Medial Temporal Gyrus
D. Precuneus
E. Superior Frontal Gyrus
6. IDENTIFY:
A. Cingulate Gyrus
B. Cuneus
C. Medial Temporal Gyrus
D. Precuneus
E. Superior Frontal Gyrus
7. IDENTIFY:
A. Cuneus
B. Inferior Parietal Lobule
C. Lingual Gyrus
D. Precuneus
E. Superior Parietal Lobule
8. IDENTIFY:
A. Genu of Corpus Callosum
B. Hypothalamus
C. Lamina Terminalis
D. Mammillary Body
E. Rostrum of Corpus Callosum
9. IDENTIFY:
A. Hypothalamus
B. Mammillary Body
C. Massa Intermedia
D. Optic Chiasma
E. Pulvinar
10. IDENTIFY:
A. Hypothalamus
B. Mammillary Body
C. Massa Intermedia
D. Optic Chiasma
E. Thalamus

11. IDENTIFY:
A. Cingulum
B. Long Gyrus
C. Orbital Gyrus
D. Short Gyrus
E. Superior Temporal Gyrus

19. IDENTIFY:
A. Crus cerebri
B. Mammillary Body
C. Optic tract
D. Substantia Nigra
E. Tuber cinereum
7. IDENTIFY:

A. Question
Cuneus Lobule

#: 159
B. Inferior Parietal Lobule
C. Lingual Lobule

D. Precuneus Lobule
E. Superior Parietal Lobule
8. IDENTIFY:
A. Cuneus Lobule
B. Inferior Parietal Lobule
C. Lingual Lobule

D. Precuneus Lobule

E. Superior Parietal Lobule
9. IDENTIFY:

A. Hypothalamus

B. Mammillary Body
C. Massa Intermedia

D. Optic Chiasma

E. Thalamus
10. IDENTIFY:

A. Hypothalamus

B. Mammillary Body

C. Massa Intermedia
D. Optic Chiasm

E. Pulvinar
11. IDENTIFY:
A. Cingulum
B. LongGyrus
C. Orbital Gyrus 7. IDENTIFY: 8. IDENTIFY:
D. Short Gyrus A. Cuneus Lobule A. Cuneus Lobule ***
E. Superior Temporal Gyrus B. Inferior Parietal Lobule B. Inferior Parietal Lobule
C. Lingual Lobule C. Lingual Lobule
D. Precuneus Lobule *** D. Precuneus Lobule
E. Superior Parietal Lobule E. Superior Parietal Lobule

9. IDENTIFY: 10. IDENTIFY:


A. Hypothalamus A. Hypothalamus
B. Mammillary Body *** B. Mammillary Body
C. Massa Intermedia C. Massa Intermedia ***
D. Optic Chiasma D. Optic Chiasma
E. Thalamus E. Pulvinar

11. IDENTIFY:
A. Cingulum
B. Long Gyrus ***
C. Orbital Gyrus
26. IDENTIFY: D. Short Gyrus
A. Gyrus Rectus E. Superior Temporal Gyrus
B. Occipitotemporal Gyrus
C. Orbital Gyrus
D. Parahippocampal Gyrus
E. Uncus
27. IDENTIFY:
A. Gyrus Rectus
B. Occipitotemporal Gyrus
C. Orbital Gyrus
D. Parahippocampal Gyrus
E. Uncus

26. IDENTIFY:
A. Gyrus Rectus ***
B. Occipitotemporal Gyrus
C. Orbital Gyrus
Question #: 160
NEUROANATOMY
The midbrain includes the:
A. Trochlear and hypoglossal nuclei
B. Superior olivary complex
C. Spinal trigeminal nucleus
D. Vestibular nuclei and sensory decussation
E. Substantia nigra and the periaqueductal gray matter

Rationale:
a. Incorrect. Hypoglossal nuclei in medulla
b. Incorrect. Superior olivary complex in pons
c. Incorrect. Spinal trigeminal nucleus in medulla
d. Incorrect. Sensory decussation in medulla
e. Correct. These are in midbrain

Item Description: Dr. Rabin, 2014, Brainstem, Barr’s 9th ed. p 100

The tectum of the midbrain contains the:


A. Substantia nigra
B. Superior olivary complex
C. Inferior olivary complex
D. Red nuclei
E. Superior colliculi

Rationale:
a. Incorrect. Substantia nigra in tegmentum
b. Incorrect. In pons
c. Incorrect. In medulla
d. Incorrect. In tegmentum
e. Correct. The superior colliculus is in the tectum in the midbrain
Item Description: Dr. Rabin, 2014, Brainstem, Barr’s 9th ed. p 101

The cuneate fasciculus is part of the:


A. Lateral lemnisci
B. Medial lemnisci
C. Pyramidal tracts
D. Solitary tracts
E. Dorsal columns

Rationale:
a. Incorrect. Cuneate fasciculus is not part of Lateral lemnisci
b. Incorrect. Cuneate fasciculus is not part of Medial lemnisci
c. Incorrect. Cuneate fasciculus is not part of Pyramidal tracts
d. Incorrect. Cuneate fasciculus is not part of Solitary tracts
e. Correct. Cuneate fasciculus is a dorsal column
Question #: 161

Item Description: Dr. Rabin, 2014, Brainstem, Barr’s 9th ed. p 88

In the brainstem, the corticospinal tract:


A. Decussates rostral to the internal arcuate fibers
B. Runs dorsal to the spinothalamic tract
C. Runs lateral to the spinothalamic tract
D. Runs lateral to the trigeminal nuclei
E. Decussates caudal to the internal arcuate fibers

Rationale:
a. Incorrect. The sensory decussation is superior to the motor decussation
b. Incorrect. The pyramidal tract is medial and ventral in the brainstem
c. Incorrect. The pyramidal tract is medial and ventral in the brainstem
d. Incorrect. The pyramidal tract is medial and ventral in the brainstem
e. Correct. The sensory decussation is superior to the motor decussation

Item Description: Dr. Rabin, 2014, Brainstem, Barr’s 9th ed. p 89, 90

A patient has left-sided body muscle weakness but unaffected sensation. A possible location of the
lesion in the brainstem is in a part of the:
A. Right lateral pontine tegmentum
B. Right medial pontine tegmentum
C. Right medullary tegmentum
D. Right midbrain tegmentum
E. Ventral part of the brainstem

Rationale:
a. Incorrect. This symptom is related to descending motor pathways, running ventrally in brainstem. Does
not contain motor pathways
b. Incorrect. This symptom is related to descending motor pathways, running ventrally in brainstem. Does
not contain motor pathways
c. Incorrect. This symptom is related to descending motor pathways, running ventrally in brainstem. Does
not contain motor pathways
d. Incorrect. This symptom is related to descending motor pathways, running ventrally in brainstem. Does
not contain motor pathways
e. Correct. This symptom is related to descending motor pathways, running ventrally in brainstem.
Contains motor pathways

Item Description: Dr. Rabin, 2014, Brainstem, Barr’s 9th ed. pp 89-100

In the brainstem, the corticospinal tract:


A. Decussates caudal to the internal arcuate fibers
B. Decussates rostral to the internal arcuate fibers
Question
C. Runs#:dorsal
162 to the spinothalamic tract
D. Runs lateral to the spinothalamic tract
E. Runs lateral to the trigeminal nuclei

SOURCE: Dr. Rabin, 2013, Nervous System and Behavior, brainstem, corticospinal tract Cellular/Molecular
Thread

The cuneate fasciculus is part of the:


A. Dorsal columns
B. Lateral lemnisci
C. Medial lemnisci
D. Pyramidal tract
E. Solitary tract

SOURCE: Dr. Rabin, 2013, Nervous System and Behavior, brainstem, cuneate fasciculus, dorsal columns
Cellular/Molecular Thread

The rostral extreme of the dorsal columns reaches the same level in the brainstem as the:
A. Inferior olive
B. Middle cerebellar peduncles
C. Obex
D. Red nuclei
E. Tectum

Rationale: Answer choices A and C are correct. As per Dr. Rabin, “Inferior olive is visible at rostral extreme of
dorsal columns.”
SOURCE: Dr. Rabin, 2013, Nervous System and Behavior, brainstem, obex, dorsal columns
Cellular/Molecular Thread

The superior cerebellar peduncle decussate in the brainstem at the coronal level of the:
A. Abducens nuclei
B. Inferior colliculi
C. Red nuclei
D. Superior colliculi
E. Vestibular nuclei
Rationale: As per Dr. Rabin, “See Barr p 100. DSP= superior cerebellar peduncle decussate; IC= Inferior
colliculi.”
SOURCE: Dr. Rabin, 2013, Nervous System and Behavior, brainstem, superior cerebellar peduncle
Cellular/Molecular Thread

All three cerebellar peduncles are visible in horizontal section at the level of the:
A. Hypoglossal nuclei
B. Inferior olivary nuclei
Question #: 163
C. Pontine nuclei
D. Red nuclei
E. Trochlear nuclei
SOURCE: Dr. Rabin, 2013, Nervous System and Behavior, brainstem, superior cerebellar peduncle
Cellular/Molecular Thread

A unilateral lesion in the dorsal pons will affect:

A. Convergence
B. Horizontal saccades away from the side with the lesion by both eyes
C. Horizontal saccades away from the side with the lesion by the eye ipsilateral to the lesion
D. Horizontal saccades toward the side with the lesion by both eyes
E. Horizontal saccades toward the side with the lesion by the eye ipsilateral to the lesion

Rationale: Choice [D] is the most accurate and best answer. A unilateral lesion in the dorsal pons would affect
an abducens nucleus. This indirectly affects the use of the contralateral eye which is under abducens control
via the abducens nucleus>mlf>contralateral oculomotor complex>cn3 pathway. Thus an injury to one
abducens nucleus results in paralysis of conjugate gaze (both eyes) toward the side of the lesion [choice D].
Choice 'E' is not the best choice because it is related more to a cn vi injury or a ventral pons injury

SOURCE: Dr. Rabin, lecture
 Cellular/Molecular Thread.

What combination of symptoms is more likely due to a unilateral brainstem injury than a cranial nerve
injury?

A. Elevation and lateral gaze in the same eye


B. Elevation and medial movement in the same eye
C. Elevation and pupillary light reflex in the same eye
D. Elevation in one eye and medial movement in the other eye
E. Medial movement in one eye and lateral movement in the other eye

Rationale: Both D and E are correct answers.

SOURCE: Dr. Rabin, lecture Cellular/Molecular Thread


Shining light in a blind eye will cause:

A. A pupillary light reflex in neither eye


B. A pupillary light reflex in only the contralateral eye
C. A pupillary light reflex in that eye
D. The same effect as shining a light in a functioning eye of a patient with a superior dorsal midbrain

 lesion ipsilateral to the stimulated eye
E. The same effect as shining a light in a functioning eye of a patient with a superior ventral midbrain

 lesion ipsilateral to the stimulated eye

SOURCE: Dr. Rabin, lecture Cellular/Molecular Thread

Impaired leftward conjugate movement of both eyes is caused by an injury to the:

A. Left abducens nerve


B. Left abducens nucleus
C. Left basilar pons
D. Right abducens nerve
Question
E. Right#:abducens
164 nucleus

SOURCE: Dr. Rabin, lecture Cellular/Molecular Thread

In a healthy subject pure extorsion of both eyes occurs:

A. Never
B. When both inferior rectus muscles shorten
C. When both superior rectus muscles shorten
D. When tilting the head towards the left side
E. When tilting the head towards the right side

Rationale: An inferior rectus muscle cannot extort the eye purely; it extorts and depresses the eye.

SOURCE: Dr. Rabin, lecture
 Cellular/Molecular Thread


Conjugate horizontal eye movements involve:

A. An abducens nucleus and a contralateral oculomotor nucleus


B. An abducens nucleus and a contralateral trochlear nucleus
C. Both abducens nuclei
D. Both oculomotor nuclei
E. The ipsilateral abducens and oculomotor nuclei

SOURCE: Dr. Rabin, lecture Cellular/Molecular Thread

The medial rectus muscle:

A. Depresses the eye


B. Elevates the eye
C. Extorts the eye
D. Intorts the eye
E. Participates in conjugate eye movements

SOURCE: Dr. Rabin, lecture Cellular/Molecular Thread

In regard to human neuroanatomy:


A. Bigger brains in adults are consistent with higher intelligence
B. Brains are grossly asymmetrical about the midsagittal plane
C. Brains contain much more neurons than neuroglia
D. Brains do not grow in size after birth
E. Male brains are, on average, heavier than female brains

Rationale:
a. Incorrect. Bigger brains have more neuroglia, not neurons
b. Incorrect.There are minor differences and variations but both halves are grossly considered
symmetrical
c. Incorrect. Ten times more neuroglia than neurons!
d. Incorrect. Several fold (3-4 in most cases)
e. Correct. Males have more neuroglia, not neurons

Question #: 165
Item Description: Dr. Leheste, Gross Divisions of the CNS lecture notes; ppt slides; digital lecture recordings

Which sulcus/fissure separates the frontal lobe from the parietal lobe?
A. Central sulcus
B. Lateral fissure
C. Longitudinal fissure
D. Postcentral sulcus
E. Sylvian fissure

Rationale:
a. Correct. Also divides primary motor and primary sensory cortex
b. Incorrect. Divides temporal from frontal and parietal lobes
c. Incorrect. Divides left and right brain hemisphere
d. Incorrect. Lies within parietal lobe
e. Incorrect. Same as lateral fissure

Item Description: Dr. Leheste, Gross Divisions of the CNS lecture notes; ppt slides; digital lecture recordings

Which of the following areas contains portions of the primary visual cortex?
A. Insula
B. Lingual gyrus
C. Postcentral gyrus
D. Precentral gyrus
E. Superior temporal gyrus
Rationale:
a. Incorrect – The insular cortex is mainly involved in visceral and autonomic functions (more in classroom
session#23)
b. Correct. Part of occipital lobe
c. Incorrect. Primary sensory cortex
d. Incorrect. Primary motor cortex
e. Incorrect. Primary auditory cortex & Wernicke’s area posteriorly

Item Description: Dr. Leheste, Gross Divisions of the CNS lecture notes; ppt slides; digital lecture recordings

The parietal lobe is _______________to the spinal cord which is _________________to the cerebellum

A. Rostral; ventral
B. Ventral; superior
C. Posterior; anterior
D. Medial; parasagittal
E. Dorsal; dorsal
Rationale:

Question #: 166
a. correct – Even if the human CNS was a straight line as in the rat, this would be still true
b. incorrect – The cerebellum is definitely superior to the spinal cord
c. incorrect – Would work if anterior and posterior were switched
d. incorrect – All the structures here have medial and lateral aspects – choice makes therefore no sense
e. incorrect – The cerebellum would be dorsal to the spinal cord, not the other way

Item Description: Dr. Leheste, Gross Divisions of the CNS lecture notes; ppt slides; digital lecture recordings

On average, an adult male brain_____________compared to an adult female brain.

A. Has a higher neuron/neuroglia ratio


B. Has a higher intelligence capability
C. Is generally heavier
D. Provides for a higher brain weight/body weight ratio
E. Shows absolutely no gender-specific differences

Rationale: As per Dr. Leheste, “The male brain is typically heavier than the female brain because of an
increased total content of neuroglia which increases the neuroglia/neuron ratio (more neuroglia per neuron).
Choice “A ” actually states the opposite and is therefore not a valid answer choice. The only acceptable choice
therefore remains “C.”

SOURCE: Dr. Leheste 2013 Interactive Review: Gross Divisions of the CNS Cellular/Molecular

The complete human brainstem is composed of:

A. Diencephalon, pons, medulla


B. Medulla, spinal cord
C. Midbrain, pons, medulla
D. Pons, medulla
E. Spinal cord, pons, medulla

SOURCE: Dr. Leheste 2013 Interactive Review: Gross Divisions of the CNS Cellular/Molecular

Damage to which cerebral structure is responsible for the great majority of expressive aphasia cases
in right-handed individuals?

A. Left frontal lobe


B. Left insular lobe
C. Left parietal lobe
D. Right occipital lobe
E. Right temporal lobe

Question #: 167
SOURCE: Dr. Leheste 2013 Interactive Review: Gross Divisions of the CNS Cellular/Molecular

Which brain lobe is the most commonly affected in patients that suffer from expressive aphasia?

A. Left frontal lobe


B. Left temporal lobe
C. Right occipital lobe
D. Right parietal lobe
E. Right temporal lobe

SOURCE: Leheste, Powerpoint Lecture on 01.10.2012 & Barr’s The Human Nervous System, An
Anatomical Viewpoint. John A. Kierman, 9th ed., 2009.
 Cellular/Molecular Thread

Which of the primary or secondary brain vesicles contribute to the developing brainstem?

A. Diencephalon, metencephalon and myelencephalon


B. Metencephalon, mesencephalon and myelencephalon
C. Metencephalon, telencephalon and myelencephalon
D. Prosencephalon, mesencephalon and myelencephalon
E. Rhombencephalon, diencephalon and mesencephalon

Rationale:

a. Incorrect. The diencephalon develops into eyes and the thalami and is not part of the brainstem
b. Correct. As per definition
c. Incorrect. The telencephalon (pallium and subpallium) becomes cerebral cortex and basal ganglia
d. Incorrect. The prosencephalon becomes telencephalon and diencephalon
e. Incorrect. The diencephalon develops into eyes and thalami and is not part of the brainstem

Item Description: Dr. Leheste, Interactive Review of Neuroembryology, Histology The Developing Human, 9th
ed., Chapter 17; lecture notes; ppt slides; digital lecture recordings

Cells of which of the following embryonic structures/tissues are most likely to differentiate into neural
crest cells?

A. Amnion
B. Developing epidermis
C. Mesenchymal mesoderm
D. Neural plate
E. Notochord

Rationale:
a. Incorrect.
Question #: 168The amnion is mesodermal
b. Incorrect. The epidermis is ectodermal but not neuroectodermal
c. Incorrect. Neural tissue is not derived from mesodermal tissue, it is neuroectodermal instead
d. Correct. The lateral ends of the invaginating neural tube will become neural crest
e. Incorrect. Notocord tissue is mesodermal and will become the nucleus pulposus in the intervertebral
disc

Item Description: Dr. Leheste, Interactive Review of Neuroembryology, Histology The Developing Human, 9th
ed., Chapter 17; lecture notes; ppt slides; digital lecture recordings

Which tissue layer associated with the developing neural tube is the birthplace for all neurons and
macroglia?

A. Germinal
B. Mantle
C. Marginal
D. Meningeal
E. Mesodermal

Rationale:

a. Correct. Even stem cells in the intermediate/marginal layer originate in the germinal layer
b. Incorrect. Stem cells in the mantle layer are migrants from the germinal layer
c. Incorrect. Stem cells in the intermediate/marginal layer originate in the germinal layer
d. Incorrect. None of the meningeal layers have that capacity
e. Incorrect. Only microglia are mesodermal, never macroglia or neurons

Item Description: Dr. Leheste, Interactive Review of Neuroembryology, Histology The Developing Human, 9th
ed., Chapter 17; lecture notes; ppt slides; digital lecture recordings

Cells of which of the following embryonic structures/tissues induce invagination of the neural plate
through emission of the signaling protein SHH (sonic hedgehog)?

A. Neural crest
B. Neural groove
C. Neural tube
D. Notochord
E. Paraxial mesoderm

Rationale:

a. Incorrect. The neural crest responds to SHH via invagination, it does not induce it
b. Incorrect. The neural groove is neural crest responding to SHH but not inducing it
c. Incorrect. The neural tube results from this process and does not initiate it
d. Correct. The notochord is the main player in this process by producing SHH
e. Incorrect.
Question #: 169The paraxial mesoderm ‘kicks’ in later by producing other growth factors (i.e. FGF, BMPs,
TGFbeta)

Item Description: Dr. Leheste, Interactive Review of Neuroembryology lecture notes; ppt slides; digital lecture
recordings

For the experimental treatment of degenerative disk (vertebral) disease, the injection of notochord
cells is currently being evaluated. Prior to treatment, notochord cells must undergo rigorous testing
which will most likely identify them as derivatives of:

A. Ectoderm
B. Endoderm
C. Mesoderm
D. Neural crest
E. Neuroectoderm

SOURCE: Dr. Leheste 2013 Interactive Review: Neuroembryology Cellular/Molecular Thread

What will most likely constitute the long-term result of a failed rostral neuropore closure in a human
embryo approximately 4 weeks post-fertilization?

A. Absence of hindbrain structures


B. Arnold-Chiari malformation
C. Hydrocephalus
D. Meroanencephaly
E. Spina bifida occulta

SOURCE: Dr. Leheste 2013 Interactive Review: Neuroembryology Cellular/Molecular Thread

A patient presents with a rare genetic disorder affecting all cells of neuroectodermal origin. In which of
the following cell types found outside the central nervous system would one most likely expect
primary defects?

A. Cardiac Purkinje fibers


B. Melanocytes
C. Microglia
D. Oligodendrocytes
E. Skeletal myocytes

SOURCE: Dr. Leheste 2013 Interactive Review: Neuroembryology Cellular/Molecular Thread


What portion of the developing human brain encompasses the majority of the cerebral aqueduct?

Question #: 170
A. Diencephalon
B. Mesencephalon
C. Metencephalon
D. Myelencephalon
E. Telencephalon

SOURCE: Dr. Leheste 2013 Interactive Review: Neuroembryology Cellular/Molecular

Which cell type gives rise to sensory ganglia of cranial and spinal nerves as well as Schwann cells?

A. Cells of the paraxial mesoderm


B. Endodermal stem cells
C. Neural crest cells
D. Notochord cells
E. Oligodendrocytes

SOURCE: Leheste, Powerpoint Lecture on 01.09.2012 & lecture notes. Cellular/Molecular Thread

Which form of Spina bifida is characterized by both, displaced meninges as well as displaced spinal
cord?

A. Spina bifida encephalocele


B. Spina bifida meningocele
C. Spina bifida meningomyelocele
D. Spina bifida myeloschisis
E. Spina bifida occulta

SOURCE: Leheste, Powerpoint Lecture on 01.09.2012 & lecture notes. Cellular/Molecular Thread

What is the most common cause of Arnold-Chiari malformation?

A. Cerebellar herniation through the foramen magnum


B. Delayed closure of the rostral or caudal neuropores
C. Delayed development of the spinal meninges
D. Exencephaly with prevailing hindbrain structures
E. Imbalanced production/absorption of cerebrospinal fluid

Rationale: As per Dr. Leheste, “This question is asking for the “cause” of Arnold-Chiari. The correct answer
choice (Cerebellar herniation through the foramen magnum) is, according to farlex online medical dictionary
(http://medical-dictionary.thefreedictionary.com/cause) actually a “cause” and not a symptom: ... “in diseases,
an agent, event, condition or characteristic which plays an essential role in producing an occurrence of the
disease.” Using the term “cause” in lecture to describe the pathogenesis is consisted with Moore and Persaud
7th ed., p456 which is the recommended reading for this lecture. The potential ambiguity between “cause” and
“symptoms” exists throughout the body of medical literature and is not specific for this lecture. The correct
answer choice is the only one that follows a logical path and was presented as such in lecture.”

SOURCE: Leheste, Powerpoint Lecture on 01.09.2012 & lecture notes Cellular/Molecular Thread


Which dietary deficiency is most commonly associated with neural tube defects in developing human
embryos?

Question #: 171acid deficiency


A. Ascorbic
B. Beta-carotene deficiency
C. Folic acid deficiency
D. Selenium deficiency
E. Vitamin D deficiency

SOURCE: Leheste, Powerpoint Lecture on 01.09.2012 & lecture notes. Cellular/Molecular Thread

Which of the five developing secondary brain vesicles gives rise to the cerebellum?

A. Diencephalon

B. Mesencephalon

C. Metencephalon
D. Myelencephalon

E. Telencephalon


SOURCE: Leheste, Powerpoint Lecture on 01.09.2012 & lecture notes. Cellular/Molecular Thread

Which descending tract overlaps with the dorsal spinocerebellar tract?

A. Lateral corticospinal tract


B. Spinothalamic tract
C. Tectospinal tract
D. Ventral corticospinal tract
E. Vestibulospinal tract

Rationale:

a. Correct. This descending tract is located in the most lateral and dorsal portion of the lateral funiculus,
like the dorsal spinocerebellar tract.
b. Incorrect. This ascending tract spans the medial portion of the lateral and ventral funiculi
c. Incorrect. This descending tract is located in the medial portion of the ventral funiculus
d. Incorrect. This descending tract is located in the medial portion of the ventral funiculus
e. Incorrect. This descending tract is located in the ventral portion of the ventral funiculus

Item Description: Dr. Kurtzer, Power point/lecture, Barr’s The Human Nervous System, 9th edition, Chapter 5
pg 64-78, Neuroanatomy an atlas of structures, section, and systems by Duane Haines

Which of the following contains the cell bodies of second-order neurons which relay muscle
spindleinformation from the left arm?

A. Contralateral nucleus dorsalis


B. Contralateral nucleus gracilis
C. Ispilateral dorsal root ganglion
D. Ipsilateral nucleus cuneatus
E. Ipsilateral nucleus proprius
Rationale:

Question #: 172
a. Incorrect. This 2nd order nucleus receives touch and muscle spindle inputs from the ipsilateral side of
the body (right side)
b. Incorrect. This 2nd order nucleus receives touch and muscle spindle inputs from the ipsilateral side of
the lower body (right side)
c. Incorrect. This 1st order ganglia receives somatosensory inputs from the ipsilateral side of the body (left
side)
d. Correct. This 2nd order nucleus receives touch and muscle spindle inputs from the ipsilateral side of
the upper body (left side)
e. Incorrect. This 2nd order nucleus receives pain and temperature inputs from the ipsilateral side of the
body (left side)
Item Description: Dr. Kurtzer, Power point/lecture, Barr’s The Human Nervous System, 9th edition, Chapter 5
pg 64-78, Neuroanatomy an atlas of structures, section, and systems by Duane Haines

Which segments of the spinal cord contain the gray matter for controlling the lower limbs?

A. C1-C4
B. C4-T1
C. L2-S3
D. S3-S5
E. T8-L2

Rationale:

a. Incorrect. These segments contain gray matter for controlling the neck and shoulder
b. Incorrect. These segments contain gray matter for controlling the upper limb
c. Correct. These segments contain gray matter for controlling the lower limb
d. Incorrect. These segments contain gray matter for controlling the pelvic floor
e. Incorrect. These segments contain gray matter for controlling the trunk

Item Description: Dr. Kurtzer, Power point/lecture, Barr’s The Human Nervous System, 9th edition, Chapter 5
pg 64-78, Neuroanatomy an atlas of structures, section, and systems by Duane Haines

A lesion to this nucleus could block the ascending transmission of pain information from the right
arm?

A. Contralateral intermediolateral column


B. Contralateral nucleus proprius
C. Ipsilateral nucleus cuneatus
D. Ipsilateral nucleus proprius
E. Ipsilateral nucleus dorsalis

Rationale:
a. Incorrect.
Question #: 173This nucleus comprises autonomic preganglionic neurons.
b. Incorrect. This nucleus receives pain and temperature sensory inputs from the ipsilateral side of the
body (left side)
c. Incorrect. This nucleus receives touch and muscle spindle inputs from the ipsilateral side of the upper
body (right side)
d. Correct. This nucleus receives pain and temperature sensory inputs from the ipsilateral side of the
body(right side). The overall pathway crosses from one side of the spinal cord to the other, but the first
nucleus is on the same side of the body as the sensory source (ipsilateral).
e. Incorrect. This nucleus receives touch and muscle spindle inputs from the ipsilateral side of the
upperbody (right side)

Item Description: Dr. Kurtzer, Power point/lecture, Barr’s The Human Nervous System, 9th edition, Chapter 5
pg 64-78, Neuroanatomy an atlas of structures, section, and systems by Duane Haines

Which tract of the lateral funiculus transmits sensory information from muscle spindles of the
ipsilateral leg?

A. Dorsal spinocerebellar tract


B. Dorsolateral tract of Lissauer
C. Gracile fasciculus
D. Lateral corticospinal tract
E. Spinothalamic tract

Rationale:

a. Correct. This tract is contained in the lateral funiculus and transmits touch and muscle spindle
information from the ipsilateral side of the body. Note that it is named the dorsal spinocerebellar but is
part of the lateral funiculus.
b. Incorrect. This tract forms the boundry between dorsal and lateral funiculi and locally transmits pain and
temperature information from the ipsilateral side of the body
c. Incorrect. This tract is contained in the dorsal funiculus (not the lateral funiculus) and transmits touch
and muscle spindle information from the ipsilateral side of the upper body.
d. Incorrect. This tract is contained in the lateral funiculus and transmits descending motor commands to
the ipsilateral side of the body
e. Incorrect. This tract is contained in the lateral funiculus and transmits pain and temperature information
from the contralateral side of the body

Item Description: Dr. Kurtzer, Power point/lecture, Barr’s The Human Nervous System, 9th edition, Chapter 5
pg 64-78, Neuroanatomy an atlas of structures, section, and systems by Duane Haines

Which cranial nerves are located at the ponto-medullary junction?

A. Abducens, Facial, and Vestibulocochlear


B. Accessory,Glossopharyngeal, and Vestibulocochlear
C. Facial, Hypoglossal, and Vestibulocochlear
D. Glossopharyngeal, Trochlear, and Vestibulocochlear
E. Glossopharyngeal, Trochlear, and Vagus
Question #: 174

Rationale:

a. Correct. Only the Abducens (CN6), Facial (CN7), and Vestibulocochlear (CN8) are at the ponto-
medullary junction
b. Incorrect. Only the Abducens (CN6), Facial (CN7), and Vestibulocochlear (CN8) are at the ponto-
medullary junction
c. Incorrect. Only the Abducens (CN6), Facial (CN7), and Vestibulocochlear (CN8) are at the ponto-
medullary junction
d. Incorrect. Only the Abducens (CN6), Facial (CN7), and Vestibulocochlear (CN8) are at the ponto-
medullary junction
e. Incorrect. Only the Abducens (CN6), Facial (CN7), and Vestibulocochlear (CN8) are at the ponto-
medullary junction

Item Description: Dr. Kurtzer, Power point/lecture, Kiernan, Barr’s The Human Nervous System, 9th edition,
Chapter 5, 6 &10

Which portion of the cerebellar cortex is situated between the two hemispheres?

A. Dentate nucleus
B. Flocculus
C. Middle peduncle
D. Primary fissure
E. Vermis

Rationale:

a. Incorrect. This is a deep cerebellar nucleus


b. Incorrect. This lateral region of the cerebellum is not part of the anterior or posterior lobe
c. Incorrect. This is a white matter tract of the cerebellum
d. Incorrect. This longitudinal fissure separates the cerebellum’s anterior and posterior lobes
e. Correct. This is the midline portion of the cerebellar cortex of the anterior and posterior lobes

Item Description: Dr. Kurtzer, Power point/lecture, Kiernan, Barr’s The Human Nervous System, 9th edition,
Chapter 5, 6 &10

The conus medullaris is located at which vertebral segment of an adult?

A. L1
B. L3
C. L5
D. S3
E. T8
Rationale:
Question #: 175

a. Correct. The conus medullaris is the tip of the spinal cord. In the third fetal month, the tip is even with
the lowest vertebral segment, but the vertebral columns then grow more extensively than the spinal
cord. At birth the tip ends at L3 and adulthood it ends at L1
b. Incorrect. The conus medullaris is the tip of the spinal cord and ends at L1 in the adult
c. Incorrect. The conus medullaris is the tip of the spinal cord and ends at L1 in the adult
d. Incorrect. The conus medullaris is the tip of the spinal cord and ends at L1 in the adult
e. Incorrect. The conus medullaris is the tip of the spinal cord and ends at L1 in the adult

Item Description: Dr. Kurtzer, Power point/lecture, Kiernan, Barr’s The Human Nervous System, 9th edition,
Chapter 5, 6 &10

Within the cervical spinal cord, this descending motor tract has a territory largely overlapping the
corticospinal fibers which decussate in the caudal medulla:

A. Recticulospinal tract
B. Rubrospinal tract
C. Spinothalamic tract
D. Ventral spinocerebellar tract
E. Vestibulospinal tract

Rationale: As per Dr. Kurtzer, “The recticulo-spinal tract has a wide extent in the interior portion of ventral and
lateral funiculi. This territory may include some lateral and anterior corticospinal tract, but the only motor tract
that largely overlaps the corticospinal tract, i.e. ≥ 50% of its fibers overlap with corticospinal fibers, is the
rubrospinal tract.”

SOURCE: Kurtzer 2013 Internal Anatomy of the Spinal Cord, White matter tracts, Rubrospinal tract, Lecture
and assigned readings Cellular/Molecular Thread

The percentage of uncrossed corticospinal fibers within the spinal cord:

A. 0%
B. 20%
C. 50%
D. 80%
E. 100%

Rationale: Answer choice A and B are correct answers. As per Dr. Kurtzer, “Since many fibers in the anterior
corticospinal tract facts will decussate within the cord, answer (A. 0%) is reasonable and should be accepted.”
SOURCE: Dr. Kurtzer 2013 Internal Anatomy of the Spinal Cord, White matter tracts, Corticospinal tract,
Lecture and assigned readings Cellular/Molecular Thread
Question #: 176

The tract of Lissauer separates these tracts of white matter:

A. Caudate and gracile fasciculi


B. Dorsal and lateral funiculi
C. Dorsal and ventral spinocerebellar tracts
D. Lateral and ventral funiculi
E. Left and right dorsal funiculi

SOURCE: Dr. Kurtzer 2013 Internal Anatomy of the Spinal Cord, White matter tracts, Funiculi, Lecture and
assigned readings Cellular/Molecular Thread

A 47-year-old woman experiences complete paralysis of her biceps brachii. An MRI of the cervical
spinal cord would reveal a lesion of which Laminae of Rexed?

A. II
B. IV
C. VI
D. IX
E. X

Rationale: As per Dr. Kurtzer, “There certainly will be motor deficits to a lesion to lamina IV and VI due to an
inability to relay and integrate sensory information. However, the total inability to activate a muscle through
either sensory afferents or descending commands requires a lesion to the motor neurons; motor neurons are
found in lamina IX.”

SOURCE: Dr. Kurtzer 2013 Internal Anatomy of the Spinal Cord, Grey matter, Lamina of Rexed, Lecture and
assigned readings Cellular/Molecular Thread

Pre-ganglionic neurons of the sympathetic system are present in which portion of the spinal cord?

A. Central grey
B. Dorsal horn
C. Dorsolateral track of Lissauer
D. Lateral horn
E. Ventral horn

SOURCE: Dr. Kurtzer 2013 Internal Anatomy of the Spinal Cord, Grey matter, Lateral horn, Lecture and
assigned readings Cellular/Molecular Thread
What structure is the most caudal limit of the spinal cord?

Question #: 177
A. Cauda aquina
B. Conus medullaris
C. Filum terminale
D. Lumbosacral enlargement
E. S3

Rationale: As per Dr. Kurtzer, “The conus medullaris is the end of the spinal cord. Both spinal nerves and filum
terminale (a vestigial spinal nerve) extend from the spinal cord but neither are a division of the spinal cord.”

SOURCE: Dr. Kurtzer 2013 External Anatomy of the Spinal Cord, Brainstem, and Cerebellum, Spinal cord
anatomy, Conus medullaris, Lecture and assigned readings Cellular/Molecular Thread

Which cranial nerve exits the ventral surface of the pons?

A. Abducens
B. Facial
C. Hypoglossal
D. Trigeminal
E. Trochlear

Rationale: As per Dr. Kurtzer, “The Abducen, Facial, and Vestibulocochlear nerves exit the brainstem at the
ponto-medullary border. Only the Trigeminal nerve exits from the surface of the pons.”

SOURCE: Dr Kurtzer 2013 External Anatomy of the Spinal Cord, Brainstem, and Cerebellum, Brainstem
anatomy, Cranial nerves, Lecture and assigned readings Cellular/Molecular Thread

Which fiber tract connecting the brain and cerebellum has the most lateral placement?

A. Basis peduncle
B. Inferior brachium
C. Inferior cerebellar peduncle
D. Middle cerebellar peduncle
E. Superior cerebellar peduncle

SOURCE: Dr Kurtzer 2013 External Anatomy of the Spinal Cord, Brainstem, and Cerebellum, Cerebellar
anatomy, Cerebellar peduncle, Lecture and assigned readings Cellular/Molecular Thread

The spinal cord tapers to a point called the conus medullaris. What vertebrae are located nearby this
point in the normal adult?

A. C7-T1

B. L1-L2
Question #: 178
C. L5-S1

D. T6-T7

E. S3-S4


SOURCE: Dr. Kurtzer lecture, Barr’s The Human Nervous System Chapter 5 Cellular/Molecular Thread

You observe that the patient exhibits a weak stretch reflex to a tendon-tap applied just below her
patella. This could indicate injury at what level of the spinal cord?

A. C5-C6

B. C6
C. L2-L4

D. T8-T10

E. S2-S4


SOURCE: Dr. Kurtzer lecture, Barr’s The Human Nervous System Chapter 5 Cellular/Molecular Thread

A spinal herniation impinging on the cord at the level of T7-T9 would likely lead to what spatial pattern
of pain sensation?

A. Encircling the mid-trunk


B. Encircling the pelvis and buttocks
C. Radiating down the chest
D. Radiating down the front of both legs
E. Radiating down the underside of each arm

SOURCE: Dr. Kurtzer, lecture Cellular/Molecular Thread

The pyramids are formed by descending tracts from the sensorimotor cortex to spinal circuits. These
tracts from the left and right hemispheres cross at which place in the brainstem?

A. Caudal portion of the dorsal medulla


B. Caudal portion of the ventral medulla
C. Caudal portion of the ventral midbrain
D. Rostral portion of the dorsal midbrain
E. Rostral portion of the ventral medulla

SOURCE: Dr. Kurtzer lecture, Barr’s The Human Nervous System Chapter 6 Cellular/Molecular Thread

Which cranial nerve is immediately medial to the olive?

A. Abducens nerve
B. Glossopharyngeal
C. Hypoglossal nerve
D. Trochlear nerve
E. Vestibulocochlear nerve

Rationale: “Lecture #16 Spinal Cord, Brainstem, and Cerebellum”, was scheduled to be tested and this lecture
dealt with the external anatomy of these three structures. Cranial nerves are a major external feature of
brainstem anatomy and were covered in both the lecture and associated book chapter. This question dealt only
with the placement of the cranial nerves, not their function, nuclei, or patterns of connectivity. “Immediately
medial” is the most adjacent nerve which is medial. The only appropriate answer is the hypoglossal nerve.
SOURCE: Dr. Kurtzer lecture, Barr’s The Human Nervous System Chapter 6
 Cellular/Molecular
Thread

Question #: 179

You might also like